2004 anesth practice ques ebook

240
Anesthesia Anesthesia Practice Questions Practice Questions Book 1 Book 1 Edited by Rodolfo Amaya, MD Gregory A. Smith, MD Medtext Medical World, Inc. Medtext Medical World, Inc.

Upload: mary-johns

Post on 23-Dec-2015

27 views

Category:

Documents


7 download

DESCRIPTION

anes practice

TRANSCRIPT

Page 1: 2004 Anesth Practice Ques eBook

AnesthesiaAnesthesia Practice QuestionsPractice Questions

Book 1Book 1

Edited by Rodolfo Amaya, MD

Gregory A. Smith, MD Medt

ext M

edic

al W

orld

, Inc.

Medt

ext M

edic

al W

orld

, Inc.

Page 2: 2004 Anesth Practice Ques eBook

Copyright 2004 Medtext Medical World, Inc.

www.medtext.net [email protected]

c/o Editorial Office 700 N. Pacific Coast Highway

Suite #302 Redondo Beach, CA 90277

1-888-MEDTEXT (633-8398)

All rights reserved. This book is protected by copyright. No part of this book may be reproduced in any form or by any means, including photocopying, or utilized by any information storage and retrieval system without written permission from the copyright owner, see above address. Printed in the United States of America. ISBN 1-889344-65-6 Please Note: Our knowledge in the clinical sciences changes on a constant basis. Much care has been taken to make certain that the information contained within this book is correct and accurate. However, the reader is advised to consult all relevant educational and instructional literature regarding questionable data. ANESTHESIA PRACTICE QUESTIONS BOOK 1 - 2004

Page 3: 2004 Anesth Practice Ques eBook

Anesthesia Practice Questions Book 1 – 2004

Edited by Rodolfo Amaya, MD

Gregory A. Smith, MD

Page 4: 2004 Anesth Practice Ques eBook
Page 5: 2004 Anesth Practice Ques eBook

Contributors

EDITOR

Rodolfo Amaya, MD Assistant Professor and Clinical Coordinator

Department of Anesthesiology Keck School of Medicine of USC

Los Angeles, California

Gregory A. Smith, MD Private Practice

Comprehensive Pain Relief Group Redondo Beach, CA

CONTRIBUTORS

Rodolfo Amaya, MD Assistant Professor and Clinical Coordinator

Department of Anesthesiology Keck School of Medicine of USC

Los Angeles, California

J. Sudharma Ranasinghe, MD, FFARCSI Assistant Professor of Anesthesiology University of Miami School of Medicine

Jackson Memorial Medical Center Miami, Florida

M. Iqbal Shaikh, MD, PhD Department of Anesthesia and Peri-operative Care

University of California, San Francisco School of Medicine San Francisco, California

Page 6: 2004 Anesth Practice Ques eBook
Page 7: 2004 Anesth Practice Ques eBook

Acknowledgments As editor of this publication, I could not do it alone and I have to thank the contributions of my fellow authors. With their help, Medtext has provided what I believe is a very comprehensive review for the written boards. I wish every reader the best of luck. Also, I would like to thank Medtext Publishing for providing this opportunity for board review. My studies for the written boards (back when) would not have been complete had there not been these review publications. Keep up the good work! Rodolfo Amaya, MD

Page 8: 2004 Anesth Practice Ques eBook
Page 9: 2004 Anesth Practice Ques eBook

Dedication

To my wife Laleh, thank you for your encouragement, patience and understanding. I’m sorry for bringing my work home.

I love you.

Page 10: 2004 Anesth Practice Ques eBook

Questions

Page 11: 2004 Anesth Practice Ques eBook

Anesthesia Book 1 – 2004 ANSWER SHEET

There are 250 blocks below to record your answers. 1

26 51 76 101 126 151 176 201 226

2

27 52 77 102 127 152 177 202 227

3

28 53 78 103 128 153 178 203 228

4

29 54 79 104 129 154 179 204 229

5

30 55 80 105 130 155 180 205 230

6

31 56 81 106 131 156 181 206 231

7

32 57 82 107 132 157 182 207 232

8

33 58 83 108 133 158 183 208 233

9

34 59 84 109 134 159 184 209 234

10

35 60 85 110 135 160 185 210 235

11

36 61 86 111 136 161 186 211 236

12

37 62 87 112 137 162 187 212 237

13

38 63 88 113 138 163 188 213 238

14

39 64 89 114 139 164 189 214 239

15

40 65 90 115 140 165 190 215 240

16

41 66 91 116 141 166 191 216 241

17

42 67 92 117 142 167 192 217 242

18

43 68 93 118 143 168 193 218 243

19

44 69 94 119 144 169 194 219 244

20

45 70 95 120 145 170 195 220 245

21

46 71 96 121 146 171 196 221 246

22

47 72 97 122 147 172 197 222 247

23

48 73 98 123 148 173 198 223 248

24

49 74 99 124 149 174 199 224 249

25

50 75 100 125 150 175 200 225 250

Please remove from the book before beginning.

Page 12: 2004 Anesth Practice Ques eBook

When you open your eBook you will see the screen split into two sections - the left section is the navigation area, the right section contains the content of your eBook.

Once you've opened your eBook and are ready to start reading you have several options to assist you in moving from page to page. Medtext has provided a clickable navigation area that will allow you to jump from question to answer, or from section to section. Each section is titled in bold text - Book Title, Questions Section, Answers Section, etc. You can click on any item listed in the Navigation.

Page 13: 2004 Anesth Practice Ques eBook

You will see a + or - (plus or minus) next to each section. If a + (plus) is visible, click the plus to expand the list of navigational links for that section. If a - (minus) is visible, you can collapse the list by clicking on the - (minus). See the images above for details. Using these links, you can move from Question 1 to Answer 1; return to Question 2 to Answer 2, etc. If you prefer to move through the book page by page, locate the page number navigation bar at the bottom of your screen and click the arrows to move forward or back.

You can adjust the size of the text you are reading by using the "Zoom In" and "Zoom Out" buttons on the toolbar at the top of your screen in Adobe Reader. Roll your curser over each button to view the button title.

Page 14: 2004 Anesth Practice Ques eBook

Clicking on the Magnifying Glass (with the plus sign in the middle) will attach the magnifying glass to your curser and allow you to click on a specific location on the page to enlarge. You can right click while the Magnifying Glass is still attached to your curser to select a magnification level from the list. Clicking on "Actual Size" will change your view to the actual size of the printed page. Clicking on "Fit Page" - will change your view so that the entire page fits on your monitor screen. Clicking on "Fit Width" will change your view so that the width of your monitor is fully utilized. The - (minus), percentage number and + (plus) show you what percentage you are currently viewing. You can click - (minus) to make the text (page) smaller, or + (plus) to make the text (page) larger. For further assistance using Acrobat Reader software, The Acrobat Reader 6.0 User's Guide is included on this CD.

Page 15: 2004 Anesth Practice Ques eBook

Anesthesia Practice Questions: Book 1 – 2004

This material may not be duplicated without written permission from the publisher. Copyright 2004, All Rights Reserved Medtext Medical World, Inc.

15

1. A 35-year-old woman in active labor had an epidural catheter placed and was given a bolus of 10 ml of 0.5% bupivacaine containing 1:200,000 epinephrine. Twenty minutes later maternal blood pressure drops to 75/45 mmHg and heart rate to 69 bpm. The Ob/Gyn nurse informs you that the fetal heart is 90 bpm and there is a loss of beat-to-beat variability. The MOST likely explanation for fetal bradycardia and loss of beat-to-beat variability is:

A) Fetal bupivacaine toxicity B) Maternal supine hypotension syndrome C) Maternal bupivacaine toxicity D) Maternal increased sensitivity to local anesthetics E) Umbilical cord compression

2. Following successful placement of an epidural, a 25-year-old woman had an

uncomplicated labor and delivery. However during catheter removal, the catheter breaks 1 cm from the tip.

What would be the MOST appropriate action?

A) Inform the patient and take no action B) Start the patient on prophylactic antibiotics C) Call the surgical consult and request surgical exploration D) Request an additional study of the epidural space using contrast dye E) Conceal the whole information and tell the patient everything went smoothly

3. Epidural test dose contains 5 microgram per ml of epinephrine. Suppose you

administer a test dose of lidocaine containing 15 micrograms of epinephrine. This would be sufficient to produce:

A) Bradycardia B) Hypertension C) Seizure D) Segmental analgesia E) Cardiovascular collapse

Page 16: 2004 Anesth Practice Ques eBook

Anesthesia Practice Questions: Book 1 – 2004

This material may not be duplicated without written permission from the publisher. Copyright 2004, All Rights Reserved Medtext Medical World, Inc.

16

4. Which of the following regional blocks are associated with the HIGHEST serum concentration of local anesthetics? Assume you use an identical dose.

A) Thoracic epidural B) Caudal C) Brachial plexus D) Intercostal E) Epidural

5. Epidural opioids combined with local anesthetics are increasingly used for optimal

management of intraoperative and post-operative analgesia.

Which of the following statements regarding this combination are TRUE?

A) Fentanyl when combined with epidural bupivacaine will decrease the concentration of the later for satisfactory analgesia

B) Fentanyl if used alone can be as effective as local anesthetics for the second stage of labor

C) Sufentanil is not appropriate for labor analgesia because it produces unsatisfactory analgesia

D) The addition of epinephrine to fentanyl will prolong the duration of analgesia E) The main determinant of the onset of epidural opioid analgesia is water

solubility 6. Which of the following properties of bupivacaine BEST explains its cardiac toxicity

when compared to other local anesthetics?

A) Bupivacaine is more soluble in water B) Bupivacaine produces a sustained block of open calcium channels C) Bupivacaine enhances sodium-potassium exchange in the myocardium D) Bupivacaine increases the sensitivity of myocardial myocardium E) Bupivacaine blocks cardiac sodium channels for prolonged period

Page 17: 2004 Anesth Practice Ques eBook

Anesthesia Practice Questions: Book 1 – 2004

This material may not be duplicated without written permission from the publisher. Copyright 2004, All Rights Reserved Medtext Medical World, Inc.

17

7. A 60-year-old male scheduled for second and third digits tendon repair of the left hand, had a brachial plexus block placed using trans-arterial approach. Fifteen minutes later he is still complaining of sensation and pain with pin prick on the dorsum of the second and third digits.

What will be the MOST appropriate approach to completely block the unaffected site?

A) Inject 3 ml of local anesthetics lateral to biceps tendon at the flexion of the crease

B) Inject 2 ml of local anesthetics 2 cm lateral to the radial border of palmaris longus tendon

C) Inject 5 ml of local anesthetics between medial epicondyle and olecranon process proximal to arcuate ligament

D) Inject 2 ml of local anesthetics between the interval of the radial artery and flexor carpi radialis tendon

E) Re-perform the axillary block using another 10 ml of LA

8. A 60-year-old man had an interscalene block for a closed reduction of his left

humerus. Fifteen (15) minutes later he has difficulty speaking. The MOST likely explanation is:

A) Sympathetic nerve block B) Phrenic nerve block C) Subdural or subarachnoid injection D) Pneumothorax E) Recurrent laryngeal nerve block

9. Morphine can be injected intravenously, epidurally and intrathecally.

If you were to inject 5 mg of morphine intravenously and epidurally at different times in the same patient, which of the following actions of morphine will be MOST significant?

A) Greater urinary retention with intravenous administration B) Most intense analgesia with epidural administration C) Shorter duration of action with epidural administration D) Greater incidence of pruritus follows intravenous rather than neuroaxial

administration E) Shorter duration of analgesia following epidural administration

Page 18: 2004 Anesth Practice Ques eBook

Anesthesia Practice Questions: Book 1 – 2004

This material may not be duplicated without written permission from the publisher. Copyright 2004, All Rights Reserved Medtext Medical World, Inc.

18

10. Morphine and Fentanyl are two of the most commonly used opioids for epidural administration. The two differ from each other in:

A) More intense and earlier onset of pruritus B) Delayed incidence of respiratory depression C) Faster onset of analgesia D) Longer duration of action E) Higher sensitivity of fentanyl analgesia reversal by naloxone

11. A 30-year-old man is scheduled for closed reduction of his left humerus. You

perform an interscalene block using thirty (30) ml of 0.25% of bupivacaine. Thirty minutes later the patient becomes apneic.

The MOST likely differential is:

A) Pneumothorax B) Phrenic nerve block C) Recurrent laryngeal nerve block D) Vertebral artery injection E) Subarachnoid injection

12. Celiac plexus block is indicated in patients with intractable visceral pain unmanaged

by traditional medications.

A successful block would be expected to:

A) Block parasympathetic fibers to pancreas B) Enhance peristalsis with diarrhea and manifestation of hypotension C) Block of sympathetic fibers to transverse colon D) Block erection and ejaculation E) Block somatic fibers to pancreas

Page 19: 2004 Anesth Practice Ques eBook

Anesthesia Practice Questions: Book 1 – 2004

This material may not be duplicated without written permission from the publisher. Copyright 2004, All Rights Reserved Medtext Medical World, Inc.

19

13. A surgeon requests a Bier’s block for the release of left-hand carpal tunnel syndrome. Following tourniquet inflation to 300 mmHg you administer 50 ml of 0.5% lidocaine. However the surgeon decides to terminate the procedure ten (10) minutes after lidocaine administration.

What will be the MOST appropriate action at this time?

A) Administer 4 mg morphine and then deflate the tourniquet B) Deflate the tourniquet and administer ephedrine or phenylephrine to prevent

any hypotensive episodes C) Start deflating and reinflating the tourniquet several times in a minute D) Wait 20 minutes and then deflate and re-inflate immediately, and finally

deflate after 1 minute E) Wait an hour before deflating the tourniquet completely

14. Following a C-section with a successfully placed spinal block with hyperbaric 0.5%

bupivacaine, a 19-year-old female starts complaining of severe headache.

Which of the following characteristics associated with post-dural puncture headache (PDPH) will facilitate your diagnosis?

A) PDPH is more frequent in man than woman B) PDPH can never be prevented by prophylactic epidural blood patch C) There is a lower incidence of PDPH with Quincke and Pitkin spinal needles D) PDPH is less frequent if the needle bevel is parallel to the direction of dural

fibers E) PDPH is made worst in the supine position than in the sitting position

15. A 25-year-old woman is scheduled for tendon reconstruction of her second and third

fingers due to a recent dog bite. An axillary block is performed with 30 ml of 0.25% bupivacaine using a transarterial approach. However, 15 minutes later, the patient still complains of pain when the surgeon pricks the thenar eminence.

The MOST plausible explanation for continued sensitivity is inadequate block in the distribution of:

A) Median nerve B) Ulnar nerve C) Radial D) Musculocutaneous nerve E) Medial cutaneous nerve of the forearm

Page 20: 2004 Anesth Practice Ques eBook

Anesthesia Practice Questions: Book 1 – 2004

This material may not be duplicated without written permission from the publisher. Copyright 2004, All Rights Reserved Medtext Medical World, Inc.

20

16. A 45-year-old man with 10 years history of insulin dependent diabetes (IDDM) and a 3 month prior history of inferior infarct is scheduled for transmetatarsal amputation of the first and second toes for osteomyelitis.

For a successful block all the following nerves should be blocked EXCEPT:

A) Sural B) Saphenous C) Superficial peroneal D) Deep peroneal E) Tibial

17. Intrathecal administration of morphine provides prolonged post-operative analgesia.

The MOST likely site of morphine action is:

A) Medulla B) Fourth ventricle C) Spinal nerve roots D) Cerebral cortex E) Substantia gelatinosa

18. A 69-year-old man with a 15 years history of IDDM is scheduled for debridement of

a thigh abscess under local and a MAC anesthesia. The surgeon infiltrates the area around the abscess with 30 ml of 2% lidocaine containing 1:200,000 epinephrine. However, the patient continues complaining of pain on incision at the site of the abscess.

The MOST plausible explanation for the lack of anesthesia includes:

A) Increased protein binding of lidocaine at the necrotic site B) Formation of increased ionized fraction of local anesthetics at the site of

injection due to local tissue acidosis C) Formation of increased non-ionized fraction of local anesthetics D) Limited diffusion of lidocaine due to vasoconstriction produced by

epinephrine E) Increased hydrolysis of lidocaine due to acidosis at the local necrotic site

Page 21: 2004 Anesth Practice Ques eBook

Anesthesia Practice Questions: Book 1 – 2004

This material may not be duplicated without written permission from the publisher. Copyright 2004, All Rights Reserved Medtext Medical World, Inc.

21

19. An 80-year-old man with a medical history significant for CHF and a history of an MI three months ago, has had a transurethral resection of prostate (TURP) done under a spinal anesthetic with 10 mg bupivacaine (0.5%). Vital signs remained stable during the course of surgery, which lasted 60 minutes. Three minutes after the patient was transferred to a regular bed he developed nausea and vomiting and his blood pressure dropped to 65/40.

What is the MOST likely differential diagnosis?

A) An acute MI due to volume overload and development of severe CHF B) Bupivacaine induced sympathectomy leading to significant decrease in

preload due to peripheral vasodilation and venous pooling C) Dilutional hyponatremia D) Unrecognized excessive bleeding from prostate venous plexus E) Continued progression of sympathetic block

20. Prilocaine is not used in obstetrics regional anesthesia.

Which of the following side effects explains the MOST likely reason for this exclusion?

A) Prilocaine is not as safe and as potent as lidocaine B) Prilocaine can cause fetal methemoglobinemia C) Prilocaine has much shorter duration of action than lidocaine which precludes

its use D) Prilocaine is more toxic than bupivacaine and lidocaine E) Prilocaine is not metabolized by placenta

Page 22: 2004 Anesth Practice Ques eBook

Anesthesia Practice Questions: Book 1 – 2004

This material may not be duplicated without written permission from the publisher. Copyright 2004, All Rights Reserved Medtext Medical World, Inc.

22

21. A 60-year-old female underwent successful hysterectomy for fibroids ten hours ago under lumbar epidural with lidocaine and fentanyl. For the last 7 hours, the patient had been receiving morphine by epidural infusion. The patient continues to complain of her inability to move her lower extremities. Neurological examination indicates an inability to flex the thighs or extend the knees.

What will be the MOST appropriate course of action at this time?

A) Reassure the patient not to worry about anything B) Stop the epidural morphine and reassess the patient 12 hours later C) Obtain MRI of the lumbar spine and request a neurology consult D) Take the epidural catheter out and reassess the patient a few hours later E) Reverse the effects of morphine with naloxone and reassess the patient an

hour later 22. A 3-year-old child is scheduled for a repair of hypospadias. The urologist requests a

caudal for postoperative pain control.

The landmarks for the block include:

A) Posterior superior iliac spine B) Coccyx C) Greater trochanter of the femur D) Iliac crest E) Sacral cornu

23. Local anesthetics can be administered via a variety of routes.

Which of the following characteristics of local anesthetics will be the MOST desirable for a surgery lasting several hours?

A) Lipid solubility B) Ratio of non ionized to ionized forms C) High molecular weight D) Increased protein binding E) Presence of ester linkages

Page 23: 2004 Anesth Practice Ques eBook

Anesthesia Practice Questions: Book 1 – 2004

This material may not be duplicated without written permission from the publisher. Copyright 2004, All Rights Reserved Medtext Medical World, Inc.

23

24. A 20-year-old man is scheduled for closed reduction of a dislocated right humerus. Fifteen minutes later following a successful interscalene block with 40 ml of 0.25% bupivacaine the patient complains of facial flushing and sweating. Neurological examination shows unequal pupils.

The MOST plausible explanation for these findings is:

A) Horner’s syndrome due to cervical sympathetic block B) Injection into the vertebral artery with toxic manifestations of CNS symptoms C) Diffusion of local anesthetics to brain and block of cranial nerves D) Pneumothorax with symptoms strongly suggestive of hypoxic response E) Recurrent laryngeal nerve paralysis

25. Which of the following factors, when adjusted, will INCREASE the duration of

epidural block?

A) Using a local anesthetics with high protein binding B) Using increased volume of local anesthetics C) Using a local anesthetics with low pKa D) Adding sodium bicarbonate to local anesthetics E) Inserting a thoracic epidural

26. Following a successful vaginal hysterectomy under epidural anesthesia, a patient

complains of numbness and loss of dorsiflexion of the toes.

The MOST likely explanation is:

A) Epidural hematoma B) Lumbar abscess C) Saphenous nerve injury D) Sacral plexus injury E) Common peroneal nerve injury

Page 24: 2004 Anesth Practice Ques eBook

Anesthesia Practice Questions: Book 1 – 2004

This material may not be duplicated without written permission from the publisher. Copyright 2004, All Rights Reserved Medtext Medical World, Inc.

24

27. Most of the actions of morphine are reversed by naloxone.

Which of the following effect of morphine is MOST resistant to naloxone administration?

A) Nausea and vomiting B) Pruritus C) Analgesia D) Respiratory depression E) Urinary retention

28. The primary determinant of the duration of a Bier block is:

A) Ratio of non ionized to ionized fraction of local anesthetics B) Duration of tourniquet inflation C) Protein binding of local anesthetics D) Volume of local anesthetics administered E) Technique of exsanguination

29. A 60-year-old male with 25 years history of IDDM is scheduled for below the knee

amputation using peripheral nerve blocks.

Which of the following combination of nerves SHOULD be blocked for a successful anesthesia?

A) Femoral, Lateral femoral cutaneous and Sciatic nerves B) Obturator, Femoral and Sciatic nerves C) Obturator, Lateral femoral cutaneous and Sciatic nerves D) Three-in-one block E) Sciatic nerve only

Page 25: 2004 Anesth Practice Ques eBook

Anesthesia Practice Questions: Book 1 – 2004

This material may not be duplicated without written permission from the publisher. Copyright 2004, All Rights Reserved Medtext Medical World, Inc.

25

30. A 65-year-old man had a lumbar epidural placed for transurethral resection of the prostate (TURP). One hour into the procedure, he starts complaining of shortness of breath with associated abdominal pain. He is nauseated, tachycardic and hypertensive.

What is the appropriate NEXT step?

A) Inform the surgeon of possible bladder perforation and ask for termination of the procedure as soon as possible

B) Determine blood sodium concentration C) Administer Lasix D) Correct the hypertension with labetalol and ask the surgeon to use normal

saline as the irrigating fluid E) Treat the patient symptomatically and continue the surgery.

31. A 55-year-old man while undergoing TURBT for bladder tumor under hyperbaric

0.75% bupivacaine has sudden onset of a new atrial fibrillation with ventricular tachycardia.

The MOST appropriate treatment is:

A) Cardioversion B) Digoxin C) Esmolol D) Ouabain E) Verapamil

32. Manifestation of continued painless paralysis 24-hour later following placement of an

epidural is MOST consistent with which one of the following differentials?

A) Local anesthetics toxicity leading to nerve damages at the spinal nerve roots B) Adhesive arachnoiditis C) Anterior spinal artery stenosis D) Epidural hematoma E) Epidural abscess

Page 26: 2004 Anesth Practice Ques eBook

Anesthesia Practice Questions: Book 1 – 2004

This material may not be duplicated without written permission from the publisher. Copyright 2004, All Rights Reserved Medtext Medical World, Inc.

26

33. Intractable pain due to unresectable pancreatic cancer is MOST effectively treated with:

A) Bilateral neurolytic intercostals blocks at T10-T12 B) Bilateral sympathetic block with phenol C) Celiac plexus block with alcohol D) Epidural block with alcohol E) Subarachnoid block with alcohol

34. An epidural is placed in a healthy patient. During aspiration clear fluid is noted to

drip back freely from epidural catheter.

Which of the following findings CORRECTLY identifies the associated fluid?

A) Precipitation when mixed with an equal volume of thiopental B) Acidic pH C) Glucose 100 mg/dL D) Sodium 154 mEq/L E) Pressure of 200 mmHg

35. A 10-month-old, 10 kg infant is scheduled for emergent inguinal hernia repair.

During the history and physical examination you find the patient had history of a running nose with low grade fever and dry cough a week ago. You decide to proceed with general anesthesia. For maintenance you use sevoflurane-nitrous oxide and oxygen. After extubation the patient develops inspiratory stridor with retraction while breathing 50% oxygen.

The MOST appropriate next action would be:

A) Reintubation after induction with volatile agents B) Administration of corticosteroids C) Administration of nebulized racemic epinephrine D) Observation E) Treatment with glycopyrrolate to dry up secretions

Page 27: 2004 Anesth Practice Ques eBook

Anesthesia Practice Questions: Book 1 – 2004

This material may not be duplicated without written permission from the publisher. Copyright 2004, All Rights Reserved Medtext Medical World, Inc.

27

36. Arterial blood samples are most commonly utilized to assess the acid base status of patient’s blood.

Presence of a large air bubble in the syringe containing blood from a patient on 2L

O2 is consistent with which of the following errors?

PO2 PCO2 A) Increase Increase B) Decrease Increase C) Increase Decrease D) Decrease Decrease E) No Change No change

37. You are called by an ICU nurse to see a 75-year-old man with a history of aortic

stenosis scheduled for surgery the following day. The patient is complaining of chest pain and dizziness. His vitals include pulse rate of 120, BP 145/95 mmHg and respiration of 20.

The MOST appropriate therapeutic measure is administration of:

A) A liter of lactate Ringer’s solution intravenously B) Nitroglycerin intravenously to increase coronary circulation C) Midazolam to relieve his anxiety D) Supplemental oxygen E) Atenolol

38. Continuous positive airway pressure is used to improve oxygenation with an

improvement in arterial oxygen tension.

Which of the following patients with pulmonary disorder will benefit the MOST from continuous positive pressure ventilation?

A) A patient with severe kyphoscoliosis B) A chronic smoker with a history of COPD C) A patient with an acute episode of pulmonary edema following an abdominal

surgery D) A patient with partial pneumothorax E) A patient with a history of alveolar proteinosis

Page 28: 2004 Anesth Practice Ques eBook

Anesthesia Practice Questions: Book 1 – 2004

This material may not be duplicated without written permission from the publisher. Copyright 2004, All Rights Reserved Medtext Medical World, Inc.

28

39. You are asked to examine a 5-year-old child in the emergency room with a six hour history of fever, difficulty in swallowing and inspiratory stridor. The child seems to be in acute distress and prefers a sitting position.

The BEST appropriate management is:

A) Keep a close watch on the child B) An orotracheal intubation in the operating room with volatile anesthetics in

the presence of an ENT physician C) A rapid sequence induction with propofol and succinylcholine D) An awake nasotracheal fiberoptic intubation E) Treatment with ampicillin and admission to the ICU

40. A newborn delivered by c-section under spinal anesthesia, is in respiratory distress.

You are unable to pass a fine suction catheter through either nostril. The MOST appropriate next action is:

A) Place an oral airway until surgical correction can be accomplished B) Intubate the patient C) Administer phenylephrine nose drops to constrict the nasal blood vessels D) Pass a catheter through the mouth to facilitate breathing E) Do nothing at this time as the newborn will learn mouth breathing

41. A 45-year-old woman with a history significant for hypertrophic obstructive

cardiomyopathy is scheduled for umbilical hernia repair. Spinal anesthesia is induced with 12.5 mg of hyperbaric bupivacaine containing 25 µg fentanyl. Five minutes after spinal anesthesia her blood pressure decreases to 65/45 mmHg and heart rate is 60 bpm with an associated ST elevation in the lateral leads.

The MOST appropriate action at this time is intravenous administration of:

A) 500 cc of Lactate Ringers solution B) 100 µg of phenylephrine C) 10 mg of metoprolol D) 2 mg of morphine E) 0.2 mg of glycopyrrolate

Page 29: 2004 Anesth Practice Ques eBook

Anesthesia Practice Questions: Book 1 – 2004

This material may not be duplicated without written permission from the publisher. Copyright 2004, All Rights Reserved Medtext Medical World, Inc.

29

42. A 52-year-old woman is scheduled for bilateral mammoplasty. In view of history of obesity and past difficult intubation, an awake fiberoptic nasal intubation is planned. During her preparation for intubation you inject 3 ml of 2% lidocaine bilaterally 1 cm below each cornu of hyoid bone.

The MOST likely reason for this injection is:

A) Prevent laryngospasm during awake nasal intubation B) Block all sensation below the vocal cords C) Anesthetize the inferior laryngeal nerve D) Paralyze the posterior arytenoids muscle E) Anesthetize the glossopharyngeal nerve

43. A 3-year-old child underwent tympanoplasty and middle ear exploration under

general anesthesia. The child had recently recovered completely from an upper respiratory tract infection. Following extubation the child is crying with retraction of the upper thoracic muscles.

The MOST appropriate therapy is:

A) Reintubation with a smaller size endotracheal tube B) Positive pressure ventilation with a face mask C) Manage the child conservatively with nebulized racemic epinephrine (0.5 ml

of a 2.25% solution in 2.5 ml of normal saline). D) Intravenous administration of dexamethasone (0.25 – 0.5 mg/kg) E) Observation of the child in ICU

44. A 60-year-old man with a history significant for hypertension, coronary artery

disease, is under general anesthesia for a laparoscopic cholecystectomy. Half an hour later during the procedure the patient develops expiratory wheezing.

Which of the following is the MOST appropriate initial management effort to decrease wheezing?

A) Position the patient in reverse Trendelenburg position B) Administer lidocaine 100 mg via the endotracheal tube C) Apply a 5 cm positive-end-expiratory pressure (PEEP) D) Administer 0.5 ml of epinephrine with1:1000 dilution subcutaneously E) Manage the wheezing with mist therapy and racemic epinephrine solution

Page 30: 2004 Anesth Practice Ques eBook

Anesthesia Practice Questions: Book 1 – 2004

This material may not be duplicated without written permission from the publisher. Copyright 2004, All Rights Reserved Medtext Medical World, Inc.

30

45. Adverse effects of obesity are associated with increased morbidity and mortality.

Which of the following components of pulmonary volumes or function tests is altered the MOST in obesity?

A) Total lung capacity (TLC) B) Forced expiratory volume at 1 second (FEV1) C) Maximal midexpiratory flow rate (MMEFR) D) Functional residual capacity (FRC) E) Ratio of dead space to tidal volume

46. You are asked to examine a 5-kg neonate in an intensive care unit after closure of

omphalocele. A chart review discloses minimal blood loss during the surgery. His vitals at the time of examination are as follows:

Heart rate - 145 bpm Blood pressure - 50/25 mmHg Arterial blood gas on FiO2 of 0.6 PO2 65 mmHg, PCO2 40 mmHg, pH 7.26 Urinary output during surgery 1 ml/kg/hr. Presently no urine output for the last one

hour.

What is the MOST appropriate next step in the management of this patient?

A) Fluid challenge 10 ml/kg with lactate Ringers solution B) Reoperation to relieve abdominal pressure C) Diuresis with Furosemide D) Observation for few hours before making any further decision E) A chest x-ray to delineate the possible cause of acidosis and hypotension

Page 31: 2004 Anesth Practice Ques eBook

Anesthesia Practice Questions: Book 1 – 2004

This material may not be duplicated without written permission from the publisher. Copyright 2004, All Rights Reserved Medtext Medical World, Inc.

31

47. A 26-year-old man with the diagnosis of pheochromocytoma is scheduled for adrenalectomy. The patient had been on phenoxybenzamine 100 mg bid for a week. The surgeon informs you that the patient has adequate block of alpha and beta receptors.

Which one of the following statements is the MOST accurate for adequate block?

A) The physical examination should show postural hypotension, nasal stuffiness, decreased sweating and stable vital signs

B) The patient is adequately blocked based on the history provided by the surgeon

C) No further evaluation is necessary because you plan a deep opioid anesthesia D) Patient is not appropriately blocked as he should have been started on alpha-

methyltyrosine at least a week before the surgery E) Patient’s urine should be tested for catecholamine before proceeding with the

surgery 48. Which one of the following situations will present with MOST dangerous situation if

a vaporizer is filled with a wrong agent?

A) Enflurane-specific vaporizer is filled with halothane B) Halothane vaporizer is filled with enflurane C) Isoflurane vaporizer is filled with halothane D) Halothane vaporizer is filled with isoflurane E) Halothane vaporizer is filled with methoxyflurane

Page 32: 2004 Anesth Practice Ques eBook

Anesthesia Practice Questions: Book 1 – 2004

This material may not be duplicated without written permission from the publisher. Copyright 2004, All Rights Reserved Medtext Medical World, Inc.

32

49. Four hours after vaginal delivery a 2200-g male neonate vital signs are as follows:

Respiratory rate - 50/min Pulse rate - 115 bpm Arterial blood gas at room air: PO2 60 mmHg; PaCO2 44 mmHg; pH 7.33

Baby’s mother had received 75 mg of meperidine IM nearly 2 1/2 hours before

delivery.

What is the MOST appropriate step in the management of this patient?

A) Increase the FiO2 to 40% B) Intubate the patient and start mechanical ventilation C) Titrate the respiratory rate of the patient by intravenous administration of

naloxone D) Observe the patient E) Induce diuresis and acidify urine to increase elimination of meperidine

50. A 20-year-old man has sustained abdominal and anterior chest wall trauma. A left

pneumothorax has been controlled by a chest tube. Immediate laparotomy is planned for bleeding from an unknown site.

What will be the next MOST appropriate action in the management of this patient?

A) The chest tube should be clamped during general anesthesia B) High concentration of nitrous oxide with volatile anesthetics can be used C) Positive-pressure ventilation with high tidal volume and low rate should be

used D) Avoid nitrous oxide because it will increase intrapleural pressure and the size

of pneumothorax E) Place another chest tube on the right side to avoid tension pneumothorax

during the surgery

Page 33: 2004 Anesth Practice Ques eBook

Anesthesia Practice Questions: Book 1 – 2004

This material may not be duplicated without written permission from the publisher. Copyright 2004, All Rights Reserved Medtext Medical World, Inc.

33

51. Carbon monoxide (CO) poisoning has been reported in patients anesthetized on Monday morning especially when continuous high gas flows (> 10L/min) have been allowed to exist on an anesthesia machine over the weekend.

The inhaled anesthetic capable of producing the GREATEST concentrations of CO is:

A) Halothane B) Isoflurane C) Sevoflurane D) Desflurane E) Xenon

52. Which of the following BEST describe the effects of nitrous oxide alone on cerebral blood flow (CBF), cerebral metabolic rate for oxygen consumption (CMRO2), and intracranial pressure (ICP)?

A) CBF increases, CMRO2 decreases, ICP increases B) CBF decreases, CMRO2 decreases, ICP increases C) CBF increases, CMRO2 increases, ICP increases D) CBF increases, CMRO2 decreases, ICP decreases E) CBF no change, CMRO2 no change, ICP no change

53. The active metabolic byproduct of meperidine with potential toxic effects capable of inducing seizures is:

A) Laudanosine B) Meperidinic acid C) Normeperidinic acid D) Normeperidine E) Norcodeine

Page 34: 2004 Anesth Practice Ques eBook

Anesthesia Practice Questions: Book 1 – 2004

This material may not be duplicated without written permission from the publisher. Copyright 2004, All Rights Reserved Medtext Medical World, Inc.

34

54. A 44-year-old morbidly obese patient requires orotracheal intubation. An awake fiberoptic assisted tracheal intubation techniques will be used. Several peripheral nerve blocks will be utilized to anesthetize the airway.

Which of the following nerves assures that the mucosa from the epiglottis to the cord will be anesthetized?

A) Recurrent laryngeal nerve B) Glossopharyngeal nerve C) Sphenopalatine nerve D) Superior laryngeal nerve E) Lingual nerve

55. Regarding normal thermoregulation, afferent cold input to the hypothalamus is

primarily by which nerve fiber classification?

A) A-alpha B) A-beta C) A-delta D) B E) Unmyelinated C-fiber

56. Functional residual capacity (FRC) is composed of which of the following lung volumes?

A) Inspiratory reserve volume (IRV) + tidal volume (TV) B) Inspiratory reserve volume (IRV) + expiratory reserve volume (ERV) + tidal

volume (TV) C) Inspiratory reserve volume (IRV) + residual volume (RV) D) Expiratory reserve volume (ERV) + tidal volume (TV) E) Expiratory reserve volume (ERV) + residual volume (RV)

Page 35: 2004 Anesth Practice Ques eBook

Anesthesia Practice Questions: Book 1 – 2004

This material may not be duplicated without written permission from the publisher. Copyright 2004, All Rights Reserved Medtext Medical World, Inc.

35

57. Which of the following conditions are believed to augment the oculocardiac reflex?

A) Hypercarbia B) Hypocarbia C) Deep anesthetic state D) Hypertension E) Hypotension

58. A 74-year-old female with a history of atrial fibrillation and on chronic warfarin

therapy is scheduled for emergency surgery for correction of a small bowel obstruction.

What is the BEST therapy to urgently reverse the effects of warfarin?

A) Administer vitamin K B) Administer protamine C) Administer albumin D) Administer fresh frozen plasma E) Administer cryoprecipitate

59. A 54-year-old male is undergoing a craniotomy for a posterior fossa tumor. The

patient will be placed in a sitting position for the surgical procedure.

Monitoring of which of the following would be the MOST sensitive in detecting a venous air emboli?

A) Measuring oxygen saturation B) Measuring end-tidal carbon dioxide (CO2) C) Measuring end-tidal carbon monoxide (CO) D) Measuring end-tidal nitrogen (N2) E) Measuring pulmonary artery pressures

Page 36: 2004 Anesth Practice Ques eBook

Anesthesia Practice Questions: Book 1 – 2004

This material may not be duplicated without written permission from the publisher. Copyright 2004, All Rights Reserved Medtext Medical World, Inc.

36

60. Which of the following maneuvers would result in a DECREASE in anatomical dead space?

A) Supine to upright position B) Neck extension C) Use of bronchodilators D) Ventilation with a mask E) Tracheal intubation

61. Which of the following BEST describes the systolic and diastolic pressures of

peripheral arteries such as the dorsalis pedis artery when compared to the aortic root?

A) No change in systolic and diastolic pressures B) Higher systolic and diastolic pressures C) Lower systolic and diastolic pressures D) Lower systolic pressure and higher diastolic pressures E) Higher systolic pressure and lower diastolic pressure

62. Which of the following medication used for induction is associated with suppression

of adrenal function?

A) Sodium thiopental B) Propofol C) Etomidate D) Methohexital E) Ketamine

Page 37: 2004 Anesth Practice Ques eBook

Anesthesia Practice Questions: Book 1 – 2004

This material may not be duplicated without written permission from the publisher. Copyright 2004, All Rights Reserved Medtext Medical World, Inc.

37

63. A 23-year-old female is scheduled for a septorhinoplasty. This is her first surgical procedure and denies ever having anesthesia. Following propofol for induction and mivacurium for muscle relaxation she was intubated without any complications. Anesthesia consisted of desflurane/nitrous oxide/oxygen combination. Fentanyl was given as an adjuvant to the anesthesia. One and half hour later, no muscle twitch could be elicited by stimulating the ulnar nerve with a nerve stimulator.

Which of the following BEST explains the prolonged muscle paralysis?

A) Synergistic effects of desflurane B) Decrease renal clearance of mivacurium C) Hypothermia D) Atypical pseudocholinesterase E) Non-specific esterase deficiency

64. Which of the following pseudocholinesterase genotypes is associated with a normal

dibucaine number?

A) Ea Ea B) Ea Eu C) Eu Ea D) Eu Eu E) Ef Ea

65. A 66 year-old male patient with type II diabetes and end stage renal disease is

schedule for amputation of his left foot. He refuses regional anesthesia and prefers general anesthesia. Which of the following muscle relaxants is best avoided in this patient?

A) Succinylcholine B) Atracurium C) Cis-atracurium D) Rocuronium E) Pancuronium

Page 38: 2004 Anesth Practice Ques eBook

Anesthesia Practice Questions: Book 1 – 2004

This material may not be duplicated without written permission from the publisher. Copyright 2004, All Rights Reserved Medtext Medical World, Inc.

38

66. A 60-year-old male with colon cancer is schedule for a colon resection. The night before he underwent a “bowel prep”. There is concern that the patient may have developed hypokalemia.

Which of the following ECG changes would MOST be indicative of hypokalemia?

A) Peaked T waves B) Delta waves C) Widen QRS complexes D) Prominent U waves E) Loss of P waves

67. A 25-year-old male sustained a spinal cord injury at the level of T-10.

What is the LATEST that succinylcholine could be safely used?

A) Less than 24 hours post-injury B) 48 to 72 hours post-injury C) 1 week post-injury D) 4 weeks post-injury E) 4 months post-injury

68. Which of the following intravenous agents does NOT mediate its effect through the

gamma-aminobutyric acid (GABAA) receptor?

A) Methohexital B) Sodium thiopental C) Propofol D) Etomidate E) Ketamine

Page 39: 2004 Anesth Practice Ques eBook

Anesthesia Practice Questions: Book 1 – 2004

This material may not be duplicated without written permission from the publisher. Copyright 2004, All Rights Reserved Medtext Medical World, Inc.

39

69. A 19-year-old underwent a cesarean section following a prolong labor. The surgical procedure was performed under regional anesthesia. After delivery of the infant and placenta, the uterus remained hypotonic.

What would be the NEXT appropriate treatment?

A) Administer phenylephrine B) Administer neostigmine C) Administer desmopressin D) Administer oxytocin E) Administer vasopressin

70. After delivery of an infant and placenta, the uterus remained hypotonic. Oxytocin

was therefore given intravenously.

Which of the following represents a side effect due to oxytocin?

A) Hypotension B) Hypertension C) Bradycardia D) Hypoxia E) Diuresis

71. A patient is undergoing laser ablation of laryngeal papillomas. Suddenly a fire is

ignited in the endotracheal tube.

What is the appropriate NEXT step in this situation?

A) Remove the endotracheal tube B) Flood the field with saline C) Switch from oxygen to air D) Stop the administration of volatile anesthetics E) Stop ventilation

Page 40: 2004 Anesth Practice Ques eBook

Anesthesia Practice Questions: Book 1 – 2004

This material may not be duplicated without written permission from the publisher. Copyright 2004, All Rights Reserved Medtext Medical World, Inc.

40

72. A 16-year-old with a significant history of asthma is scheduled for a tonsillectomy.

All of the following medications would result in histamine release EXCEPT:

A) Morphine B) Meperidine C) Sodium thiopental D) Cis-atracurium E) Mivacurium

73. A 32-year-old muscular male was emerging from anesthesia when his endotracheal

tube was removed prematurely. Shortly after extubation the patients became hypoxic despite making respiratory efforts. After a failure to correct the hypoxemia by mask ventilation, the patient received succinylcholine and was re-intubated. After intubation oxygen saturation remained at 90% while on 100% oxygen.

His continued hypoxemia is MOST likely due to which of following?

A) Intubation of the esophagus B) Congestive heart failure C) Residual paralysis by succinylcholine D) Aspiration of gastric contents E) Pulmonary edema

74. A 16-year-old female is undergoing surgery for correction of spinal scoliosis. General

anesthesia is being provided with an opioid infusion (fentanyl) and with 0.5 MAC of sevoflurane. The patient is in a prone position and receiving rocuronium for muscle paralysis. Two hours into the surgical procedure, a sudden drop in end-tidal carbon dioxide from 35 mmHg to 18 mmHg is noticed followed by hypotension.

Which of the following has MOST likely occurred?

A) Sevoflurane induced hypotension B) Venous air embolus C) Capnography malfunction D) Spinal cord ischemia E) Malignant hyperthermia

Page 41: 2004 Anesth Practice Ques eBook

Anesthesia Practice Questions: Book 1 – 2004

This material may not be duplicated without written permission from the publisher. Copyright 2004, All Rights Reserved Medtext Medical World, Inc.

41

75. A patient had a spinal anesthetic with 5% lidocaine for a transurethral resection of the prostate (TURP) and was place in the lithotomy position. Following the procedure the patient complained of bilateral buttock pain.

What is the MOST likely etiology?

A) Femoral neuropathy B) Cauda equina syndrome C) Obturator neuropathy D) Epidural hematoma E) Transient neurologic syndrome

76. Mannitol’s PRIMARY site of action in the kidney is:

A) Proximal convoluted tubule B) Medullary portion of ascending loop of Henle C) Cortical portion of ascending loop of Henle D) Distal convoluted tubule E) Collecting duct

77. The prompt awakening that occurs after an intravenous induction dose of sodium

thiopental can be attributed to:

A) Redistribution from the brain to the blood B) Redistribution into fat tissue C) Redistribution into skeletal muscle D) Cytochrome P450 metabolism E) Renal excretion

Page 42: 2004 Anesth Practice Ques eBook

Anesthesia Practice Questions: Book 1 – 2004

This material may not be duplicated without written permission from the publisher. Copyright 2004, All Rights Reserved Medtext Medical World, Inc.

42

78. In attempting an awake fiberoptic assisted intubation, a superior laryngeal nerve block will be preformed.

Which of the following is an important landmark to perform this nerve block?

A) Chassaignac tubercle B) Base of the posterior tonsillar pillar C) Cricothyroid membrane D) Greater cornu of the hyoid cartilage E) Transverse process of C6

79. Soon after induction, intraoperative hypothermia begins to occur with the initial rapid

drop in temperature attributed to:

A) Convection to the environment B) Conduction to cold surfaces C) Radiation to the environment D) Decrease metabolism E) Redistribution of core temperature

80. An orthopedic surgeon has referred to your pain clinic a 46-year-old female patient with a complaint of severe pain involving her right lower extremity. She explains that she experienced a moderately severe right ankle sprain approximately 3 months ago that appeared to have had resolved, but shortly after, began to experience a burning aching pain that involved her lower extremity. On examination her leg is edematous and her skin appears shinny. She has difficulty flexing her foot.

With her history and signs and symptoms on examination, the MOST likely diagnosis is:

A) Complex Regional Pain Syndrome I B) Complex Regional Pain Syndrome II C) Fibromyalgia D) Peripheral neuropathy E) Raynaud’s disease

Page 43: 2004 Anesth Practice Ques eBook

Anesthesia Practice Questions: Book 1 – 2004

This material may not be duplicated without written permission from the publisher. Copyright 2004, All Rights Reserved Medtext Medical World, Inc.

43

81. A patient has been diagnosed with Complex Regional Pain Syndrome, Type I in the acute (hyperemic) stage.

Which of the following would be LESS likely to be present?

A) Edema B) Allodynia C) Hyperalgesia D) Increase skin temperature E) Osteoporosis

82. A 32-year-old patient had an above the elbow amputation of his right arm and is

experiencing phantom limb sensation one month later.

Which of the following is the MOST appropriate management?

A) Trial of opioids B) Use of nonsteroidal anti-inflammatory drugs C) Interscalene block D) Transcutaneous electrical nerve stimulation (TENS) E) No treatment

83. A patient with reflex sympathetic dystrophy experiences pain with a light touch of a

feather.

This phenomenon is called:

A) Hyperalgesia B) Hyperesthesia C) Hypoalgesia D) Allodynia E) Analgesia

Page 44: 2004 Anesth Practice Ques eBook

Anesthesia Practice Questions: Book 1 – 2004

This material may not be duplicated without written permission from the publisher. Copyright 2004, All Rights Reserved Medtext Medical World, Inc.

44

84. A patient is referred to your pain clinic by an oncologist for relief of severe abdominal pain. The patient is diagnosed with pancreatic cancer that is inoperable.

Which of the following interventions would be MOST appropriate?

A) Stellate ganglion block B) Celiac plexus block C) Superior hypogastric plexus block D) Cervical plexus block E) Lumbar sympathetic block

85. Following a celiac plexus block, the MOST common complication is:

A) Bradycardia B) Hypotension C) Renal injury D) Hematoma E) Paraplegia

86. In performing a brachial plexus block by an axillary approach for a distal radial

fracture repair, which of the following nerve MAY need supplementation?

A) Median nerve B) Radial nerve C) Ulnar nerve D) Musculocutaneous nerve E) Axillary nerve

87. After performing an axillary block of the brachial plexus, which of the following

sensory distribution would be consistent with a spared musculocutaneous nerve?

A) Lateral part of the palm and thenar eminence B) Thumb, index and middle finger, with lateral half of ring finger C) Medial aspect of the hand D) Lateral aspect of the forearm E) Medial aspect of the forearm

Page 45: 2004 Anesth Practice Ques eBook

Anesthesia Practice Questions: Book 1 – 2004

This material may not be duplicated without written permission from the publisher. Copyright 2004, All Rights Reserved Medtext Medical World, Inc.

45

88. A 32-year-old female had a total thyroidectomy two days ago. Since her surgery she has complained of hoarseness in her voice. Endoscopic exam of her vocal cords showed her left vocal cord in the paramedian position.

This is consistent with which type of nerve injury?

A) Unilateral recurrent laryngeal nerve injury B) Bilateral recurrent laryngeal nerve injury C) Unilateral vagal nerve injury D) Unilateral superior laryngeal nerve injury E) Bilateral superior laryngeal nerve injury

89. A patient with postherpetic neuralgia presents with severe unilateral back pain.

Which of the following medication is the MOST effective in the management of this patient’s pain?

A) Ibuprofen B) Morphine C) Amitriptyline D) Aspirin E) Topical lidocaine

90. A 43-year-old female with a history of diabetes is scheduled for an emergent

cholecystectomy for a gangrenous gallbladder. On physical exam, she is noted to be obese, but airway appears to be normal.

What is her ASA classification?

A) ASA physical status I B) ASA physical status I-E C) ASA physical status II D) ASA physical status II-E E) ASA physical status III

Page 46: 2004 Anesth Practice Ques eBook

Anesthesia Practice Questions: Book 1 – 2004

This material may not be duplicated without written permission from the publisher. Copyright 2004, All Rights Reserved Medtext Medical World, Inc.

46

91. All of the following neurophysiologic effects of inhaled anesthetics are true EXCEPT:

A) All of the potent inhalational agents depress CMR (Cerebral Metabolic Rate) in a linear fashion

B) Isoflurane causes iso-electric EEG at doses used clinically C) All of the potent inhalational agents increase the CBF (Cerebral Blood Flow)

in a dose dependent manner D) Halothane causes the greatest increase in the CBF per MAC (Minimum

Alveolar Concentration) multiple E) Isoflurane produces insignificant change in the CBF at 1 MAC doses

92. Factors that influence the correlation between set tidal volume and exhaled tidal

volume during intra-operative controlled ventilation include all of the following EXCEPT:

A) The flow meter settings B) The expiratory time C) The compliance of the breathing circuit D) Ventilatory rate E) Circuit leaks

93. The following statements about the cardiovascular effects of potent volatile anesthetics are true EXCEPT:

A) Halothane causes profound depression of the myocardial contractility B) At anesthetic levels >1 MAC, desflurane is associated with 10-20% increase

in heart rate C) Recent evidence indicates volatile agents exert important cardio-protective

effects during myocardial ischemia D) Compared to adenosine volatile anesthetics are only weak coronary

vasodilators E) Desflurane and sevoflurane cause dose-dependent increase in the myocardial

contractility

Page 47: 2004 Anesth Practice Ques eBook

Anesthesia Practice Questions: Book 1 – 2004

This material may not be duplicated without written permission from the publisher. Copyright 2004, All Rights Reserved Medtext Medical World, Inc.

47

94. A 20-year-old female patient is admitted for a diagnostic biopsy of an anterior mediastinal mass. She has a history of a cough for one month. There is no history of dyspnea, stridor or noisy breathing. Chest x-ray shows a massive mediastinal mass filling more than two-thirds of the chest. A computed tomographic scan confirms this finding and also shows compression of both main bronchi. The following statements are TRUE regarding the anesthetic management of this patient EXCEPT:

A) Rapid sequence induction should be planned with thiopentone and succinylcholine

B) An awake fiberoptic intubation under topical anesthesia of the oropharynx should be planned

C) Preoperatively, pulmonary flow volume loop studies should be carried out in the supine and upright positions

D) Total or near total airway obstruction may occur during induction or emergence from anesthesia

E) Large bore peripheral intravenous cannulae should be placed in the lower extremities

95. The major anatomic differences between the neonatal and adult airway include all of the following EXCEPT:

A) An infant’s larynx is situated higher in the neck B) An infant’s tongue is closer to the roof of the mouth and easily obstructs the

airway C) An infant’s epiglottis is narrower and angled away from the trachea D) An infant’s axis of the vocal folds is perpendicular to the trachea E) The narrowest portion of an infant’s larynx is the cricoid cartilage

96. Which of the following is NOT a characteristic of a VDD pacemaker

A) Is capable of pacing the ventricles only B) Provides AV synchrony C) Provides for a physiological rate increase D) Is a dual chamber device with two leads E) Can sense electrical activity in both the atrium and the ventricle

Page 48: 2004 Anesth Practice Ques eBook

Anesthesia Practice Questions: Book 1 – 2004

This material may not be duplicated without written permission from the publisher. Copyright 2004, All Rights Reserved Medtext Medical World, Inc.

48

97. The following statements about cardiovascular effects of drugs that are used during the reversal of nondepolarizing neuromuscular block are true EXCEPT:

A) Atropine induces its vagolytic effect much more rapidly than does

glycopyrrolate. B) Administration of glycopyrrolate with edrophonium may result in an initial

bradycardia C) Anticholinesterases should be used with caution in patients with autonomic

neuropathy D) When cardiac arrhythmias are a concern, atropine may be preferable to

glycopyrrolate E) To minimize cardiovascular changes, atropine is better suited with

edrophonium

98. How much epinephrine l:1000 should be added to 20 ml of local anesthetic

solution to obtain a 1:200,000 concentration of epinephrine?

A) 0.10 ml B) 0.15 ml C) 0.20 ml D) 0.50 ml E) 0.60 ml

99. A 60-year-old patient status post colectomy is in the PACU and has oxygen delivered

through nasal prongs at a flow rate of 5L/min. What is the maximum inspired concentration of oxygen (FiO2) that can be achieved by nasal prongs at a flow rate of 5L/min?

A) 0.28 B) 0.32 C) 0.36 D) 0.40 E) 0.50

Page 49: 2004 Anesth Practice Ques eBook

Anesthesia Practice Questions: Book 1 – 2004

This material may not be duplicated without written permission from the publisher. Copyright 2004, All Rights Reserved Medtext Medical World, Inc.

49

100. A 52-year-old man in PACU has difficulty in maintaining his oxygen saturation above 80% on simple face mask with an oxygen flow rate of 5L/min. The patient had a right upper lobectomy 15 minutes ago. Physical examination is unremarkable and chest is clear to auscultation. What is the next MOST appropriate step in the management of this patient?

A) Change the face mark to partial rebreathing mask B) Change the face mask to non-rebreathing mask C) Change the face mask to venturi face mask D) Change to nasal cannula with 10L oxygen flow E) Re intubate the patient

101. A 20-year-old man is undergoing a right adrenalectomy for pheochromocytoma under isoflurane–nitrous oxide and oxygen. Fifteen minutes in the procedure, his blood pressure increases acutely from 150/85 to 250/150 with presentation of premature ventricular contractions. The MOST appropriate management includes:

A) Administration of nitroprusside and propanolol B) Switching isoflurane to halothane C) Switching to a long acting muscle relaxant D) Using propanolol followed by phentolamine E) Increasing the concentrations of volatile anesthetics and opioids

102. A 20-year-old male is scheduled for bilateral inguinal hernia repair. Past history indicates the presence of ventricular septal defect. Which of the following hemodynamic phenomena is MOST likely?

A) Induction with intravenous anesthesia is delayed B) Induction with volatile anesthesia is accelerated C) Shunting is increased by positive pressure ventilation D) Shunting is increased by systematic hypotension E) Shunting is decreased by increase in systematic vascular resistance.

Page 50: 2004 Anesth Practice Ques eBook

Anesthesia Practice Questions: Book 1 – 2004

This material may not be duplicated without written permission from the publisher. Copyright 2004, All Rights Reserved Medtext Medical World, Inc.

50

103. A 75-year-old man received 15 units of packed red blood cells (14 days old) during repair of abdominal aneurysm over a course of seven hours.

Which of the following hemodynamic changes will be present at the end of surgery?

A) Thrombocytopenia leading to prolonged bleeding time and decreased plasma concentrations of labile factor V sufficient to prolong prothrombin time

B) Significant decrease in serum concentrations of calcium causing symptomatic hypocalcemia

C) Significant hyperkalemia to produce EKG changes and cardiac arrhythmia D) Significant metabolic acidosis which may need correction with sodium

bicarbonate E) No changes in the concentration of 2, 3 – diphosphoglycerate

104. Chronic obstructive lung diseases have some component of impairment in diffusion capacity. Which of the following statements is MOST accurate?

A) By increasing PAO2, PaCO2 increases B) Positive end expiratory pressure is more effective in increasing PaO2 than an

increase in FiO2 C) Increase in pulmonary compliance is more effective way of increasing PaO2 D) Increase in pulmonary blood flow is more effective way of increasing PaO E) Increase in cardiac output is more effective way of increasing PaO2

105. Assume during a laparotomy procedure you use halothane – nitrous oxide – oxygen as a maintenance agent for general anesthesia. The PaO2 drops from 325 mmHg to 225 mmHg at FiO2 of 0.5 in 1 1/2 hours. Physical examination is unchanged. What is the MOST likely reason?

A) Increased ventilation/perfusion mismatch B) Decreased cardiac output C) Sepsis D) Increased uptake of volatile agents E) Increased dead space to tidal volume ratio

Page 51: 2004 Anesth Practice Ques eBook

Anesthesia Practice Questions: Book 1 – 2004

This material may not be duplicated without written permission from the publisher. Copyright 2004, All Rights Reserved Medtext Medical World, Inc.

51

106. Six hours following parathyroidectomy a 30-year-old woman has changes in the quality of her voice. There is no inspiratory stridor. Serum level of calcium is 9mg/dl. What is the MOST appropriate management?

A) Re-intubate the patient B) Administer calcium gluconate C) Consider tracheostomy D) Observe in ICU for 24 hours E) Administer nebulized Racemic epinephrine solution

107. A 29-year-old man is scheduled for ankle surgery following a fall from stairs while on work. During pre-operative evaluation you notice bruises under the skin. The patient has a history of nose and gum bleed. These signs are MOST consistent with history of:

A) Clotting factor disorder B) Vitamin K deficiency C) Von Willebrand’s disease D) Primary hemostasis defect E) Use of antiplatelet agents

108. Which of the following is the MOST common cause of acquired platelet dysfunction?

A) Uremia B) Alcoholism C) Antiplatelet agents D) Fibrin degradation products (FDPs) E) Vitamin K deficiency

Page 52: 2004 Anesth Practice Ques eBook

Anesthesia Practice Questions: Book 1 – 2004

This material may not be duplicated without written permission from the publisher. Copyright 2004, All Rights Reserved Medtext Medical World, Inc.

52

109. In an 80-year-old ICU patient with septic shock and disseminated intravascular coagulation (DIC), which of the following drugs SHOULD be avoided?

A) Aminocaproic acid B) Tranexamic acid C) Aprotinin D) All of the above E) None of the above

110. Line isolation transformer can protect a patient from leakage current and microshock

hazards BECAUSE:

A) It can detect all faults B) It can detect small currents even in the pacing wires C) It has a three-wire grounding system D) It is an ungrounded system on the secondary side of transformer E) None of the above

111. A 78-year-old woman is undergoing total colectomy for colon cancer. His vitals are

stable. An hour later in the procedure the five lead EKG changes from sinus rhythm to ventricular tachycardia with the rate of 140 bpm. Blood pressure changes from 140/88 to 75/45 mmHg. What is the MOST appropriate next step in the management of this patient?

A) External DC cardioversion B) Get a 12-lead EKG and then plan the next step C) Administer phenylephrine intravenously D) Administer amiodarone intravenously E) Administer a calcium channel blocker

Page 53: 2004 Anesth Practice Ques eBook

Anesthesia Practice Questions: Book 1 – 2004

This material may not be duplicated without written permission from the publisher. Copyright 2004, All Rights Reserved Medtext Medical World, Inc.

53

112. A 25-year-old woman with no previous history of surgery underwent umbilical hernia repair under general anesthesia with propofol and 140 mg of succinylcholine. After an hour following completion of surgery patient had inadequate skeletal muscle strength to permit extubation of the trachea.

Which of the following is the MOST likely explanation of her weakness?

Cholinesterase Activity Dibucaine number

A) Normal 45 B) Low 45 C) Normal 80 D) Low 80 E) Low 20

113. During fetal heart rate monitoring in a 25-year-old woman, fetal heart rate decreases uniformly from 150 bpm to 120 bpm during uterine contractions.

The MOST likely explanation is?

A) Decrease in maternal blood oxygen tension B) Compression of maternal great vessels C) Compression of fetal head D) Compression of umbilical cord E) Excessive uterine activity

114. A 30-year-old woman with a transection of spinal cord at C6 level sustained after a motor vehicle accident 2 years ago is scheduled for a urology procedure. You plan a general anesthesia.

Which of the following hemodynamic parameters will show the MOST variations during the procedure?

A) Heart rate B) Regulation of body temperature C) Urine output D) Blood PaCO2 and PaO2 E) Postoperative shivering

Page 54: 2004 Anesth Practice Ques eBook

Anesthesia Practice Questions: Book 1 – 2004

This material may not be duplicated without written permission from the publisher. Copyright 2004, All Rights Reserved Medtext Medical World, Inc.

54

115. During preoperative evaluation a 60-year-old man with a history of small cell carcinoma of right upper lobe scheduled for mediastinoscopy, complains of bilateral upper and lower extremities weakness which improves with exercise.

This patient will show:

A) Increased response to anticholinesterases B) Increased sensitivity to depolarizing and nondepolarizing muscle relaxants C) Gradual decreased muscle strength with exercise D) Increased muscle strength following administration of prednisone E) Resolution of symptoms following administration of immunosuppressive

agents 116. An 80-year-old man is scheduled for excision of a 3-cm localized lesion on forehead

(basal cell carcinoma) as an outpatient surgery the following day. As his anesthesiologist you SHOULD:

A) See the patient sometime today for preanesthetic evaluation B) Call the patient and instruct him to take clear fluids for 12 hours before

surgery C) Order EKG, CXR, CBC and electrolytes before surgery D) Order oral lorazepam and instruct him to take the night before the surgery E) See the patient at the time of surgery

117. Which of the following group of patients is NOT at an increased risk for surgical complications performed at an outpatient setting?

A) Patients with a history of prolonged postoperative nausea and vomiting B) Patients with a history of immunosuppressive diseases C) Neonates born prematurely D) Presently healthy neonates treated for respiratory distress syndrome at birth E) Elderly patients with a history of hypertension and diabetes

Page 55: 2004 Anesth Practice Ques eBook

Anesthesia Practice Questions: Book 1 – 2004

This material may not be duplicated without written permission from the publisher. Copyright 2004, All Rights Reserved Medtext Medical World, Inc.

55

118. Magnesium used in the treatment of pregnancy-induced hypertension, can enhance the neuromuscular blockade during general anesthesia.

Which of the following agents may REVERSE this effect?

A) Potassium B) Calcium C) Sodium D) Neostigmine E) Lidocaine

119. Two days after thyroidectomy under general anesthesia a 27-year-old woman is found confused and disoriented. Her neck is extended with jaw rigidity, inspiratory stridor and thready pulse. What should be the next MOST appropriate course of action in the management of this patient?

A) Paralyze and intubate the patient B) Administer Ativan intravenously C) Return the patient to the operating room for reexploration of her neck D) Administer oxygen and intravenous calcium gluconate E) Face mask her with oxygen and order stat electrolytes and CBC

120. A 25-year-old man is undergoing general anesthesia for cholecystectomy. Blood gas

analysis shows PaO2 of 80 mmHg, PaCO2 of 50 mmHg and a mixed expiratory PCO2 of 25 mmHg with tidal volume of 600 ml. Which of the following statements is TRUE?

A) His physiologic dead space is 300 ml due to increased atelectasis B) His physiologic dead space is 300 ml with no changes in V/Q mismatch C) His physiologic dead space is 300 ml due to increased in V/Q mismatch D) His physiologic dead space is 450 ml due to increase in V/Q mismatch E) There is an increase in anatomic dead space with no changes in physiologic

dead space

Page 56: 2004 Anesth Practice Ques eBook

Anesthesia Practice Questions: Book 1 – 2004

This material may not be duplicated without written permission from the publisher. Copyright 2004, All Rights Reserved Medtext Medical World, Inc.

56

121. The left ventricle is perfused during diastole. Which of the following parameters BEST estimate left ventricular subendocardial

perfusion pressure?

A) The difference between mean arterial and central venous pressures B) The difference between diastolic arterial and central venous pressures C) The difference between systolic and central venous pressures D) The difference between diastolic arterial and pulmonary wedge pressures E) The difference between diastolic arterial and pulmonary artery diastolic

pressures 122. A 40-year-old 60-kg woman with a history of myasthenia gravis had thymectomy

done under general anesthesia two hours ago. General anesthesia was induced with fentanyl, propofol and rocuronium (30 mg) followed by intubation. Patient is alert, oriented, and cooperative and follows commands two hours post-surgery. Her present ventilator settings include: intermittent mandatory ventilation (IMV) at a rate of 5/min., tidal volume of 700 ml and FIO2 of 0.5. The patient is breathing over the ventilator at a rate of 20/min. MIF is – 17 cm H2O. Her heart rate is 80 bpm and blood pressure 130/75. ABG shows PaO2 of 130 mmHg, PaCO2 of 48 mmHg and pH of 7.34. What will be the next MOST appropriate course of action in the management of this patient at this time?

A) Reduce the IMV rate to 2/min. and watch for the next 4 hours B) Reverse the effects of rocuronium with neostigmine and glycopyrrolate and

then extubate after 15 minutes C) Administer 2 mg edrophonium intravenously and recheck MIF D) Keep the patient intubated till the return of full muscle strength E) Start the patient on intramuscular anticholinesterases and recheck vitals

Page 57: 2004 Anesth Practice Ques eBook

Anesthesia Practice Questions: Book 1 – 2004

This material may not be duplicated without written permission from the publisher. Copyright 2004, All Rights Reserved Medtext Medical World, Inc.

57

123. A 25-year-old man with known history of hemophilia A, is scheduled for open reduction of his left knee following a fall. His HCT is 29 and level of circulating factor VIII concentration is 5% of the normal. What is the MOST appropriate preoperative and intraoperative course of action in the management of this patient?

A) Administration of fresh frozen plasma to increase factor VIII to 50% of normal

B) Administration of fresh whole blood to increase factor VIII to 50% normal C) Administration of recombinant factor VIII to increase factor VIII to 50%

normal D) Administration of cryoprecipitate to increase factor VIII to 50% of normal E) Postponement of surgery till HCT and factor VIII levels are normalized

124. A 25-year-old man is brought to the ER by paramedics. The patient has burns over

the face, head, neck and both hands. Physical examination shows facial burns, singed facial and nasal hair and redness of

oropharyngeal and nasopharyngeal mucosa. There are no signs of stridor, grunting or hoarseness. In the management of the airway in this patient what is the MOST appropriate next step?

A) Immediate placement of humidified 100% oxygen by face mask B) Endotracheal intubation as soon as possible C) Observation and intubation if the signs of respiratory obstruction appears D) Checking the carboxyhemoglobin and methemoglobin levels in the blood and

intubation if carboxyhemoglobin level is greater than 15% E) Intubation if PaO2 level is less than 100 mmHg at FiO2 of 0.4

125. Which combination of drugs freely penetrates the blood –brain- barrier?

A) Physostigmine, scopolamine, carbon-dioxide B) Neostigmine, glycopyrrolate, carbon-dioxide C) Atropine, glycopyrrolate, bicarbonate D) Scopolamine, atropine, glycopyrrolate E) Edrophonium, atropine, bicarbonate

Page 58: 2004 Anesth Practice Ques eBook

Anesthesia Practice Questions: Book 1 – 2004

This material may not be duplicated without written permission from the publisher. Copyright 2004, All Rights Reserved Medtext Medical World, Inc.

58

126. An 80-year-old man with a history of hypertension and peripheral edema is scheduled for inguinal hernia repair. His medications include spironolactone and furosemide.

His pre-operative evaluation should include tests for:

A) Hyperkalemia B) Hypercalcemia C) Acidosis D) Hyponatremia E) Hypoglycemia

127. Which of the following clinical situations will be contraindications to the use of

metoclopramide as a premedication?

A) Intestinal obstruction, tremors, hepatic dysfunction B) Diabetic gastroparesis, esophageal reflux C) Chemotherapy induced emesis D) Patients on digoxin and insulin E) Pregnancy induced emesis

128. A 50-year-old man complains of severe pain on left chest wall. He has a history of

small cell carcinoma involving his left lung and active “shingles” in the distribution of T6-8 nerves on the right thoracic chest wall. During pulmonary function tests, which of the following parameters is MOST likely to be accurate?

A) Forced expiratory volume in one second (FEV1) B) Maximum mid-expiratory flow (MMEF) C) Negative inspiratory pressure D) Peak expiratory flow rate E) Maximum breathing capacity

Page 59: 2004 Anesth Practice Ques eBook

Anesthesia Practice Questions: Book 1 – 2004

This material may not be duplicated without written permission from the publisher. Copyright 2004, All Rights Reserved Medtext Medical World, Inc.

59

129. A 60-year-old, 150 kg man had an episode of severe cough during induction of anesthesia for a right hip replacement. Three minutes later his oxygen saturation as measured through the pulse oximeter dropped to 80 from 100. Oral suctioning noted copious secretions and vomitus in his mouth.

Physical examination reveals bilateral wheezing. Arterial blood gasses on FiO2 of

45% are: PaO2 of 52 mmHg, PaCO2 of 50 mmHg, pH 7.32. In the management of this patient what is the next MOST appropriate step?

A) Airway suctioning, intubation, mechanical ventilation with PEEP, cancellation of surgery and observation in ICU

B) Airway suctioning, intubation, saline lavage and continuation of surgery C) Intubation followed by administration of steroids and antibiotics D) Intubation, cancellation of surgery and administration of albuterol E) Airway suctioning, intubation, mechanical ventilation and continuation of

surgery

130. A 5-year-old child who aspirated a small piece of a toy into his right bronchus is anesthetized with sevoflurane in oxygen.

The ENT surgeon uses a ventilating bronchoscope to remove the foreign body.

During several attempts to remove the foreign body, it becomes more difficult to ventilate the child. The MOST appropriate emergent next step in the management of this patient is?

A) Perform emergent tracheostomy B) Push the foreign body into right upper bronchus C) Deepen the level of anesthesia with volatile anesthetics D) Paralyze the child with a muscle relaxant, start mechanical ventilation and

them attempt to take the foreign body out E) Administer racemic epinephrine and intravenous dexamethasone to decrease

subglottic edema. Proceed with foreign body extraction after this treatment

Page 60: 2004 Anesth Practice Ques eBook

Anesthesia Practice Questions: Book 1 – 2004

This material may not be duplicated without written permission from the publisher. Copyright 2004, All Rights Reserved Medtext Medical World, Inc.

60

131. A 6-year-old child is scheduled for tonsillectomy. Following induction with sevoflurane-oxygen you administer succinylcholine 1.5 mg/kg intravenously. Two minutes later it is impossible to open his mouth for intubation. What is the MOST appropriate next management plan in this situation?

A) Administer an additional intravenous dose of succinylcholine because children need higher dose

B) Cancel the surgery and monitor the child’s vitals cautiously C) Order a 12-lead EKG and draw ABG D) Determine creatine-phosphokinase levels in the blood and switch to

intravenous agents such as propofol for maintenance of anesthesia E) Administer dantrolene prophylactically and continue the surgery

132. A 30-year-old alcoholic is scheduled for emergent exploratory laparotomy for

continued abdominal bleed. He is drowsy, restless and complaining of neck pain. Physical and radiographic examinations completed so far reveal LeFort 2 and 3 fracture of maxilla, mandibular fracture, rhinorrhea and lacerated cheek. The general surgeon requests general anesthesia with complete muscle relaxation. The BEST method to secure and maintain his airway for anesthesia is:

A) Topical anesthesia of airway followed by awake oral intubation with in-line stabilization of the neck

B) Blind nasal intubation C) Awake nasal intubation using fiberoptic bronchoscope D) Rapid sequence induction followed by intubation E) Emergent tracheostomy under sedation

133. Acute respiratory distress syndrome (ARDS) is the complication of several systematic

illnesses with high morbidity and mortality. Which of the following parameters mentioned in choices A to E will show the major

pathophysiologic changes after recovery from ARDS?

A) Diffusion barrier to oxygen B) Changes in pulmonary compliance C) Changes in pulmonary capillary wedge pressure D) Changes in pulmonary shunt fraction E) Changes in oxygen dissociation curve

Page 61: 2004 Anesth Practice Ques eBook

Anesthesia Practice Questions: Book 1 – 2004

This material may not be duplicated without written permission from the publisher. Copyright 2004, All Rights Reserved Medtext Medical World, Inc.

61

134. You are evaluating a 70-year-old man in ICU who had a severely stenosed aortic valve replaced with a prosthetic aortic valve. Preoperatively patient had been hemodynamically stable and his EKG showed sinus rhythm.

His present hemodynamic profile in ICU is as follows:

Heart rate 125 bpm, now in atrial fibrillation Blood pressure 75/40 mmHg Central venous pressure 18 mmHg Pulmonary capillary wedge pressure 23 mmHg Cardiac output 2.1 L/min Systematic vascular resistance 1300 dynes/sec/cm-5

What is the MOST appropriate first step to improve his hemodynamic profile?

A) Administer dopamine B) Administer dobutamine and start on nitroglycerin drip C) Cardioversion D) Administer intraaortic balloon pump E) Administer verapamil

135. An 80-year-old man with history significant for coronary artery disease is scheduled

for surgery of right eye retinal detachment. Following induction and intubation with propofol and rocuronium general anesthesia is maintained with sevoflurane and oxygen. Suddenly his blood pressure climbs to 240/110 from 120/60 mmHg and pulse decreases from 75 bpm to 40 bpm. His EKG shows run of bigeminies and trigeminies every 4 to 5 minutes. Surgeon informs you that he just instilled 10% phenylephrine into his conjunctiva to dilate his pupil. What will be the MOST appropriate action at this time?

A) Administer propanolol and lidocaine B) Administer nitroprusside C) Administer either glycopyrrolate or atropine D) Administer phentolamine E) Wait until the action of phenylephrine dissipates

Page 62: 2004 Anesth Practice Ques eBook

Anesthesia Practice Questions: Book 1 – 2004

This material may not be duplicated without written permission from the publisher. Copyright 2004, All Rights Reserved Medtext Medical World, Inc.

62

136. While doing a neuroanesthesia case, adjustment of which of the following parameters will be the MOST important in regulating blood flow to ischemic cerebral tissue?

A) Mean perfusion pressure B) PaO2 C) PaCO2 D) Hyperthermia E) Cerebral metabolic rate (CMRO2)

137. Hypothermia at 33° C. will show all of the following physiologic changes EXCEPT:

A) Protection against cerebral ischemia B) Cardiac dysrhythmia and hypertension C) Increased drug metabolism and increased isoflurane MAC D) Prolongation of rocuronium action E) Impaired renal function

138. Which of the following clinical situations will be a contraindication to the use of

ketamine?

A) An 80-year-old man with a history of coronary artery disease, profoundly hypotensive, scheduled for emergency surgery for ruptured appendix

B) A 25-year-old asthmatic scheduled for emergency exploratory laparotomy after a motor vehicle accident and who is hypotensive

C) A 25-year-old with cardiac tamponade D) A 25-year-old with head trauma, hypotension and ICP of 10 E) A 16-year-old burn patient scheduled for extensive debridement of burn over

upper extremities and thorax

Page 63: 2004 Anesth Practice Ques eBook

Anesthesia Practice Questions: Book 1 – 2004

This material may not be duplicated without written permission from the publisher. Copyright 2004, All Rights Reserved Medtext Medical World, Inc.

63

139. A 25-year-old heroin addict is scheduled for exploratory laparotomy after a motor vehicle accident. His vitals are: blood pressure 100/60, pulse 105 and respiration 4/min. Which of the following drugs if given to this man will produce acute withdrawal?

A) Pentazocine B) Nalbuphine C) Buprenorphine D) Naloxone E) All of the above

140. According to ACLS guidelines, during cardiopulmonary resuscitation administration of

calcium chloride is recommended during:

A) Hyperkalemia and verapamil toxicity B) Electromechanical dissociation and asystole C) Digoxin and tricyclic antidepressant toxicity D) Shock and congestive heart failure E) None of the above

141. Following repair of bilateral inguinal hernia in a 70-year-old man with a history of severe COPD, you decide to use 30 mg of ketorolac (Toradol). Which of the following statements about ketorolac is TRUE?

A) Ketorolac has little potential for renal toxicity when fluid balance is adequate B) Ketorolac analgesic effects are due to its binding to opioid receptors C) Ketorolac is excreted unchanged in urine D) Ketorolac can increase bleeding due to dose-related thrombocytopenia E) Ketorolac has analgesic effects but no antipyretic or anti-inflammatory effects

Page 64: 2004 Anesth Practice Ques eBook

Anesthesia Practice Questions: Book 1 – 2004

This material may not be duplicated without written permission from the publisher. Copyright 2004, All Rights Reserved Medtext Medical World, Inc.

64

142. Midazolam (versed) is most often used pre-operatively. Compared with diazepam it is:

A) Three times more potent than diazepam B) Has longer elimination half time C) Has less clearance (ml/kg/min) D) Produces less respiratory depression E) Produces less hypotension during induction of anesthesia with opioids

143. A 75-year-old acutely ill ICU patient has serum phosphate levels of < 1 mg/dL Which of the following disorders will MOST likely be manifested in this patient?

A) Increased aspiration risk B) Increased muscle weakness C) Increased frequency of seizures D) Increased frequency of ventricular ectopic beats E) Increased GI motility and incidence of diarrhea

144. Morbidly obese patients and patients suffering from Pickwickian syndrome share

common pathophysiologic characteristics.

Which of the following characteristics will differentiate a morbidly obese patient from a patient diagnosed with Pickwickian syndrome?

A) Upper airway obstruction and carbon dioxide retention B) Decreased expiratory reserve volume C) Increased pulmonary shunt fraction D) Decreased functional residual capacity E) Increased incidence of hypertension, hypoxia and dysrhythmia

Page 65: 2004 Anesth Practice Ques eBook

Anesthesia Practice Questions: Book 1 – 2004

This material may not be duplicated without written permission from the publisher. Copyright 2004, All Rights Reserved Medtext Medical World, Inc.

65

145. All of the following adverse effects can be attributed to cyclosporine toxicity EXCEPT:

A) Abnormal hepatic enzymes B) Microcytic, hypochromic anemia C) Increased serum creatinine concentration D) ST-T changes in EKG after chronic use E) Increase in nodular density of a mass present in the lung

146. Propofol is one of the most common general anesthesia inducing agents.

All of the following mechanisms contribute to propofol induced hypotension EXCEPT:

A) Systematic vasodilation B) Central vagal stimulation C) Decreased central sympathetic tone D) Resetting discharge from aortic and carotid bodies E) Myocardial depression

147. A 50-year-old man is undergoing clipping of cerebral aneurysm under general anesthesia. The maintenance anesthetics include: nitrous-oxide-oxygen-isoflurane-narcotics-relaxant. Half an hour into the procedure the neurosurgeon informs you that the brain is tense and bulging.

Patient’s vital signs and ABG are as follows: Heart rate 100 bpm Mean arterial pressure 90 mmHg PaO2 120 mmHg PaCO2 23 mmHg pH 7.5 What is the MOST appropriate immediate course of action?

A) Hyperventilate the patient to a PaCO2 of 15 mmHg B) Administer Lasix 40 mmHg intravenously C) Administer mannitol 1 g/kg intravenously D) Administer thiopental 250 mg intravenously E) Stop nitrous-oxide and volatile anesthetics and start intravenous propofol

Page 66: 2004 Anesth Practice Ques eBook

Anesthesia Practice Questions: Book 1 – 2004

This material may not be duplicated without written permission from the publisher. Copyright 2004, All Rights Reserved Medtext Medical World, Inc.

66

148. A 65-year-old man is undergoing exploratory laparotomy for a mass in distal ileum. While exploring the distal ileum the patient suddenly becomes profoundly hypotensive, develops wheezing bilaterally and erythema over face and chest. What is the MOST appropriate next step to manage this patient?

A) Administer phenylephrine and steroids B) Administer vasopressin intravenously C) Administer diphenhydramine, cimetidine and fluids D) Infuse a liter of fluid E) Wait for the symptoms to resolve

Page 67: 2004 Anesth Practice Ques eBook

K-Type Questions

Page 68: 2004 Anesth Practice Ques eBook
Page 69: 2004 Anesth Practice Ques eBook

Anesthesia Practice Questions: Book 1 – 2004 A=1,2,3 B=1,3 C=2,4 D=4 E=All

This material may not be duplicated without written permission from the publisher. Copyright 2004, All Rights Reserved Medtext Medical World, Inc.

69

149. Inhalation anesthetics suspected of causing malignant hyperthermia INCLUDE:

1) Isoflurane 2) Sevoflurane 3) Desflurane 4) Xenon

150. A 38-year-old female patient presented with hypertension, headaches, flushing and occasional episodes of sweating. Computer tomography (CT) scan of her abdomen revealed a left adrenal mass. She is scheduled for resection of the adrenal mass.

Which of the following medications would be appropriate to use perioperatively?

1) Phenoxybenzamine 2) Phentolamine 3) Prazosin 4) Phenylephrine

151. Which of the following BEST describes the response to muscle relaxants in a patient with untreated myasthenia gravis?

1) Sensitivity to non-depolarizing muscle relaxants 2) Resistant to non-depolarizing muscle relaxants 3) Resistant to depolarizing muscle relaxants 4) Sensitivity to depolarizing muscle relaxants

152. An elderly patient with Parkinson’s disease is having a right hip arthroplasty. He gives a previous history of significant postoperative nausea and vomiting (PONV) following prior surgeries.

Which of the following medications would NOT be appropriate to administer for PONV prophylaxis?

1) Droperidol 2) Ondansetron 3) Metoclopramide 4) Dexamethasone

Page 70: 2004 Anesth Practice Ques eBook

Anesthesia Practice Questions: Book 1 – 2004 A=1,2,3 B=1,3 C=2,4 D=4 E=All

This material may not be duplicated without written permission from the publisher. Copyright 2004, All Rights Reserved Medtext Medical World, Inc.

70

153. A 38 year-old male with bipolar disorder is scheduled to have electroconvulsive therapy (ECT). Past medical history is significant for acute intermittent porphyria and anemia.

Which of the following are MOST appropriate to use for this patient?

1) Propofol 2) Atropine 3) Succinylcholine 4) Methohexital

154. During a radical nephrectomy procedure, you noticed that the T waves on the electrocardiogram have suddenly peaked. A blood specimen is immediately sent to the laboratory. Fifteen minutes later, the laboratory calls to the operating room because the potassium level is 6.4 mEq/L.

Which of the following are appropriate treatments to acutely reduce plasma potassium?

1) Hyperventilate 2) Administer glucose plus insulin 3) Administer sodium bicarbonate 4) Administer calcium chloride

155. Pregnancy is expected to DECREASE which of the following volume and capacities?

1) Expiratory reserve volume 2) Functional residual capacity 3) Residual volume 4) Vital capacity

156. Which of the following is associated with Complex Regional Pain Syndrome, Type I?

1) Involves a characteristic sensory nerve pattern 2) Always the result of nerve injury 3) Trophic changes are never present 4) Osteoporosis in late stages

Page 71: 2004 Anesth Practice Ques eBook

Anesthesia Practice Questions: Book 1 – 2004 A=1,2,3 B=1,3 C=2,4 D=4 E=All

This material may not be duplicated without written permission from the publisher. Copyright 2004, All Rights Reserved Medtext Medical World, Inc.

71

157. Which of the following represent criteria that may be used for the diagnosis of fibromyalgia?

1) Generalized pain in 3 or more sites for 3 months or longer 2) Pain relieved by a stellate ganglion block 3) Reproducible tenderness in 11 out of 18 pre specified sites 4) Sympathetic pain relief following a phentolamine infusion

158. A 60-year-old patient is diagnosed with trigeminal neuralgia. Characteristic(s)

consistent with this diagnosis include:

1) Paroxysm of pain 2) Unilateral facial involvement 3) Trigger zones may be present 4) Involves cranial nerve VII

159. A patient with CRPS type II has just received a stellate ganglion block. Evidence that

the block has been successfully placed is confirmed by the following effect(s):

1) Myosis 2) Ptosis 3) Nasal congestion 4) Hoarseness

160. In performing a stellate ganglion block, which of the following landmark(s) should be

located?

1) Cricoid cartilage 2) Greater cornu of the hyoid bone 3) Chassaignac’s tubercle 4) C-3 transverse process

Page 72: 2004 Anesth Practice Ques eBook

Anesthesia Practice Questions: Book 1 – 2004 A=1,2,3 B=1,3 C=2,4 D=4 E=All

This material may not be duplicated without written permission from the publisher. Copyright 2004, All Rights Reserved Medtext Medical World, Inc.

72

161. A patient with pancreatic cancer presents with severe abdominal pain. A celiac plexus block will be performed under fluoroscopy. Landmark(s) important to identify for performing this block include:

1) 12th ribs 2) T-12 spinous processes 3) L-1 spinous processes 4) Sacral hiatus

162. Which of the following nerve(s) require supplementation following an axillary block

of the brachial plexus?

1) Ulnar nerve 2) Intercostobrachial nerve 3) Median nerve 4) Musculocutaneous nerve

163. Epidural steroid injection has been found to be effective in all of the following

conditions EXCEPT:

1) Spondylolysis 2) Herniated disc with nerve root irritation 3) Ankylosing spondylitis 4) Spondylolisthesis

164. A patient has been diagnosed with Complex Regional Pain Syndrome (CRPS) Type II,

of hers left upper extremity. Which of the following medications are appropriate in the treatment of CRPS, Type II?

1) Clonidine 2) Amitriptyline 3) Gabapentin 4) Morphine

Page 73: 2004 Anesth Practice Ques eBook

Anesthesia Practice Questions: Book 1 – 2004 A=1,2,3 B=1,3 C=2,4 D=4 E=All

This material may not be duplicated without written permission from the publisher. Copyright 2004, All Rights Reserved Medtext Medical World, Inc.

73

165. A patient is to undergo surgery for internal fixation of his right ankle fracture. He does not wish to have general anesthesia but agrees to a spinal anesthetic.

Which of the following complications are possible due to a spinal anesthetic?

1) Hypotension 2) Hypothermia 3) Bradycardia 4) Cardiac arrest

166. A 44-year-old diabetic female is to have an amputation of her left foot because of

vascular insufficiency. She has agreed to a spinal anesthetic.

Which of the following factor(s) have the MOST influence on the spread (height of block) of the local anesthetic in the subarachnoid space?

1) Patient’s height 2) Volume injected 3) Local anesthetic concentration 4) Baricity of local anesthetic

167. It is 2 PM in the afternoon, and a patient is scheduled to undergo a surgical

procedure and you wish to perform a spinal anesthetic. In order for the patient to leave the recovery room by 5:30 PM (the closing time of the surgical center), the spinal must have receded enough to discharge the patient.

Which of the following factor(s) influence the duration of a spinal anesthetic?

1) Used of adrenergic agonist 2) Type of local anesthetic 3) Drug dose 4) Block height

Page 74: 2004 Anesth Practice Ques eBook

Anesthesia Practice Questions: Book 1 – 2004 A=1,2,3 B=1,3 C=2,4 D=4 E=All

This material may not be duplicated without written permission from the publisher. Copyright 2004, All Rights Reserved Medtext Medical World, Inc.

74

168. Perioperative factor(s) known to INCREASE the risk of postoperative nausea and vomiting is/are:

1) Sex 2) Age 3) Use of opioids 4) Tobacco use

169. A 44-year-old female patient is scheduled for hysterectomy for fibroleiomyoma. She

has a previous history of receiving anesthesia and experiencing postoperative nausea and vomiting (PONV).

Which of the following medications would be appropriated for prophylaxis against PONV?

1) Droperidol 2) Dolasetron 3) Dexamethasone 4) Ondansetron

170. The reversal of a neuromuscular blockade caused by vecuronium is dependent upon:

1) Gradual metabolism 2) Redistribution 3) Administration of cholinesterase inhibitors 4) Significant metabolism by pseudocholinesterase

171. Interactions between the neuromuscular blocking agents will have the following effects:

1) The combination of Rocuronium and Cis-atracurium are synergistic 2) Pre curarizing dose of D-Tubocurarine will shorten the duration of action of

succinylcholine 3) The potency of Rocuronium is enhanced when administered after

succinylcholine 4) The duration of action of succinylcholine is shorter when administered after

Pancuronium

Page 75: 2004 Anesth Practice Ques eBook

Anesthesia Practice Questions: Book 1 – 2004 A=1,2,3 B=1,3 C=2,4 D=4 E=All

This material may not be duplicated without written permission from the publisher. Copyright 2004, All Rights Reserved Medtext Medical World, Inc.

75

172. When monitoring the neuromuscular blockade:

1) The response of the orbicularis oculi over the eyelid is similar to that of the adductor pollicis

2) The response of the eyebrow (corrugator supercilii) is similar to that of the laryngeal adductors

3) The onset of neuromuscular blockade occurs earlier at the diaphragm 4) The dose response curve of the neuromuscular blocking drugs for the

diaphragm is shifted to the left 173. During the recovery of neuromuscular blockade with rocuronium, the TOF ratio (train

of four) at the adductor pollicis is 0.7. At this point of recovery:

1) Protection of the airway against regurgitation can be impaired 2) Most patients complain of visual symptoms 3) In the majority of patients, sustained bite is not present 4) Grip strength can be decreased

174. When monitoring the neuromuscular blockade:

1) TOF (train of four) stimulation can be repeated every 5 seconds 2) The post tetanic twitch count should not be repeated more often than every

5 minutes 3) TOF is more painful than the tetanic stimulation at 50 Hz for 5 seconds 4) The most sensitive test for detecting residual paralysis is the ability to

maintain sustained contractions to 100Hz of tetanus for 5 seconds 175. A patient with hemiplegia:

1) May develop cardiac arrest following the administration of a depolarizing muscle relaxant

2) Shows increased sensitivity to nondepolarizing muscle relaxants 3) Has a less intense block on the affected limb 4) Shows faster recovery of the neuromuscular blockade on the unaffected side

than the affected side

Page 76: 2004 Anesth Practice Ques eBook

Anesthesia Practice Questions: Book 1 – 2004 A=1,2,3 B=1,3 C=2,4 D=4 E=All

This material may not be duplicated without written permission from the publisher. Copyright 2004, All Rights Reserved Medtext Medical World, Inc.

76

176. The Mapleson D breathing system:

1) Is functionally similar to Mapleson E and F systems 2) Has a T piece near the machine end 3) Requires high fresh gas flows to prevent rebreathing of carbon dioxide 4) Is the most efficient of the Mapleson systems during spontaneous ventilation

177. Fail-safe valve:

1) Is located downstream from the nitrous oxide supply source 2) Interrupts the supply of nitrous oxide if the oxygen flow decreases 3) Is located upstream of the flow control valves 4) Prevents hypoxic gas delivery to the patient

178. Which of the following is/are TRUE in relation to the allergic reactions occurring during anesthesia?

1) Allergic reactions to local anesthetics remain uncommon despite their

frequent use 2) Muscle relaxants are the most frequently involved drugs 3) Anaphylactoid reactions are considered to activate only basophils 4) Increased concentration of tryptase (greater than 25 µg/L) suggest an

anaphylactic reaction 179. In order to optimize circle system design, the following arrangement of the major

components is/are preferred:

1) Unidirectional valves should be close to the patient 2) The fresh gas inlet should be placed between the absorber and the

expiratory valve 3) The pressure relief valve should be placed immediately before the absorber 4) The breathing bag should be located in the inspiratory limb

Page 77: 2004 Anesth Practice Ques eBook

Anesthesia Practice Questions: Book 1 – 2004 A=1,2,3 B=1,3 C=2,4 D=4 E=All

This material may not be duplicated without written permission from the publisher. Copyright 2004, All Rights Reserved Medtext Medical World, Inc.

77

180. Causes of increased peak inspiratory pressure (PIP) with little change in plateau pressure (PP) include:

1) Bronchospasm 2) Pulmonary edema 3) Increased inspiratory gas flow rate 4) Increased tidal volume

181. The causes of acute onset, inspiratory stridor in a POSTOPERATIVE patient

include:

1) Laryngospasm 2) Laryngeal edema 3) Foreign body aspiration 4) Bilateral paralysis of superior laryngeal nerve

182. During the monitoring of direct arterial blood pressure:

1) Incidence of radial artery thrombosis can be minimized by avoiding tapered catheters

2) Arterial sampling is an important source of bacterial contamination of transducer systems

3) Cerebral air embolism is a possible complication 4) Systolic pressure reading at the radial artery exceeds that of the aortic root

183. Evoked potentials:

1) Are used intraoperatively to evaluate the functional integrity of sensory and motor pathways

2) Of brain stem origin (BAEP) are less vulnerable to anesthetic agents than the evoked potentials of cortical origin (cortical component of SSEP or VEP).

3) Are minimally affected by opioids 4) Are minimally affected by N2O at concentrations of less than 50%

Page 78: 2004 Anesth Practice Ques eBook

Anesthesia Practice Questions: Book 1 – 2004 A=1,2,3 B=1,3 C=2,4 D=4 E=All

This material may not be duplicated without written permission from the publisher. Copyright 2004, All Rights Reserved Medtext Medical World, Inc.

78

184. Remifentanil hydrochloride:

1) Is a synthetic opioid 2) Is a selective µ-opioid receptor agonist 3) Has an elimination half-life of about 9 minutes 4) Is rapidly metabolized by pseudocholinesterases

185. Morphine:

1) Does not suppress myocardial contractility at clinical doses 2) When administered orally, morphine-6-glucuronide (M6G) is the primary

active compound 3) Has a bioavailability of <30% when administered orally 4) Releases less histamine when pre-treated with naloxone

186. Opioid induced muscle rigidity:

1) Is due to a direct action on muscle fibers 2) Can be decreased or prevented by pretreatment with muscle relaxants 3) Is associated with increase in creatinine kinase 4) Is mu receptor mediated

187. Morphine-6-glucuronide (M6G):

1) Is a more potent mu receptor agonist than morphine 2) Is the major metabolite of morphine 3) Has duration of action similar to that of morphine 4) Possesses a little or no analgesic activity

188. Ondansetron:

1) Is a selective antagonist at serotonin (5-HT3) receptors 2) May cause prolongation of the QT-interval 3) Is effective in the treatment of perioperative nausea and vomiting 4) Increases the lower esophageal sphincter tone

Page 79: 2004 Anesth Practice Ques eBook

Anesthesia Practice Questions: Book 1 – 2004 A=1,2,3 B=1,3 C=2,4 D=4 E=All

This material may not be duplicated without written permission from the publisher. Copyright 2004, All Rights Reserved Medtext Medical World, Inc.

79

189. Dexmedetomidine:

1) Is a highly selective alpha-2 –adrenergic agonist 2) Frequently causes dry mouth 3) Causes bradycardia 4) Increases the requirement of volatile anesthetics during general anesthesia

190. Flumazenil:

1) Has minimal intrinsic activity 2) When used to reverse diazepam, the potential for re-sedation exists 3) Can precipitate withdrawal symptoms in patients physically dependent on

benzodiazepines 4) Irreversibly binds to the benzodiazepine receptors

191. Hydroxyethyl Starch:

1) Is a synthetic colloid solution 2) Interferes with cross-matching of blood due to rouleaux formation 3) Is degraded by amylase 4) Often causes allergic reactions

192. Respiratory effects of potent volatile anesthetic agents include:

1) High concentration of volatile anesthetics may virtually obliterate the hypercarbia-induced increase in ventilatory drive

2) All volatile anesthetics decrease tidal volume 3) Most volatile agents depress the ventilatory response to hypoxia at

subanesthetic concentrations. 4) Isoflurane and sevoflurane can reverse bronchospasm with equal efficacy at

1.1 MAC

Page 80: 2004 Anesth Practice Ques eBook

Anesthesia Practice Questions: Book 1 – 2004 A=1,2,3 B=1,3 C=2,4 D=4 E=All

This material may not be duplicated without written permission from the publisher. Copyright 2004, All Rights Reserved Medtext Medical World, Inc.

80

193. During general anesthesia with nitrous oxide:

1) Megaloblastic bone marrow changes are seen in healthy patients after 2 hours of exposure

2) Causes subacute combined degeneration of the spinal cord only after several months of daily exposure

3) Causes reversible inactivation of the enzyme methionine synthase 4) Causes very low methionine synthetase activity after a few hours of

anesthesia 194. Which of the following is/are effect(s) of Nitrous Oxide?

1) Decreased uterine contractility 2) Decreased pulmonary vascular resistance 3) Decreased sympathetic nervous system activity 4) Expansion of air filled spaces

195. Factors that influence the magnitude of carbon monoxide production from volatile

anesthetics include:

1) Dryness of the carbon dioxide absorbant 2) High temperature of the carbon dioxide absorbant 3) Prolonged high fresh gas flows 4) Exposure to soda lime rather than Baralyme

196. Sevoflurane:

1) Causes formation of organic and inorganic fluoride metabolites 2) Results in formation of trifluoroacetylated liver proteins 3) Produces peak plasma fluoride concentrations that are higher than after

comparable doses of enflurane 4) Produces higher carbon monoxide concentration than desflurane on exposure

to soda lime

Page 81: 2004 Anesth Practice Ques eBook

Anesthesia Practice Questions: Book 1 – 2004 A=1,2,3 B=1,3 C=2,4 D=4 E=All

This material may not be duplicated without written permission from the publisher. Copyright 2004, All Rights Reserved Medtext Medical World, Inc.

81

197. Supine hypotension syndrome in pregnancy:

1) Is primarily a result of inferior vena caval occlusion 2) May reduce the blood flow to the uterus by as much as 45% 3) Causes approximately 1 in 10 pregnant women to become symptomatic 4) Is relieved in the Trendelenburg position

198. Physiological changes of pregnancy include:

1) Elevation of the central venous pressure 2) Decrease in the colloid oncotic pressure 3) Physiological anemia secondary to decrease in red blood cell mass 4) Threefold elevation of alkaline phosphatase

199. Transient neurological syndrome (TNS):

1) Characterized by back pain with sensory or motor deficit 2) Resolves spontaneously within several days 3) Has never been reported with bupivacaine 4) Has higher incidence with lidocaine spinal and lithotomy position

200. The following recommendations is/ are TRUE regarding neuraxial anesthesia in the

presence of antiplatelet and anticoagulation medications:

1) Clopidogrel should be discontinued for 7 days before a neuraxial anesthesia 2) Aspirin may be safely used in patients having epidural or spinal injections 3) Subcutaneous heparin does not appear to increase the risk of spinal

hematoma 4) Post surgical prophylactic dose of low molecular weight heparin (LMWH) may

be started 2-4 hours after removal of epidural catheter.

Page 82: 2004 Anesth Practice Ques eBook

Anesthesia Practice Questions: Book 1 – 2004 A=1,2,3 B=1,3 C=2,4 D=4 E=All

This material may not be duplicated without written permission from the publisher. Copyright 2004, All Rights Reserved Medtext Medical World, Inc.

82

201. Smallpox:

1) May be difficult to differentiate from chickenpox during the first 2-3 days 2) May cause death due to toxemia and overwhelming pneumonia 3) Vaccine consists of live attenuated vaccinia virus 4) Vaccine offers lifelong immunity

202. Dolasetron Mesylate:

1) May cause prolongation of QT interval in the ECG 2) Require dose adjustment in patients with hepatic or renal impairment 3) Has a major metabolite with potent serotonin (5HT3) receptor agonist

activity 4) Has an elimination half-life of 10 hours

203. Ephedrine:

1) Is an indirectly acting sympathomimetic amine 2) Has four stereoisomers 3) Is superior to other vasopressors in preserving uteroplacental blood flow 4) Has an important centrally mediated sympathetic action

204. Transdermal fentanyl preparation (Fentanyl patch):

1) Provides extended period of slow drug administration 2) In febrile patients may increase fentanyl absorption 3) Achieves therapeutic levels within 6-8 hours 4) Stops the uptake of fentanyl immediately after removal of the patch

205. Treatment of Aspiration Pneumonitis include:

1) Supplemental oxygen 2) Positive end-expiratory pressure (PEEP) 3) Inhalation of beta-2-agonists 4) Lavage with large volumes of saline administered through the endotracheal

tube

Page 83: 2004 Anesth Practice Ques eBook

Anesthesia Practice Questions: Book 1 – 2004 A=1,2,3 B=1,3 C=2,4 D=4 E=All

This material may not be duplicated without written permission from the publisher. Copyright 2004, All Rights Reserved Medtext Medical World, Inc.

83

206. Functional residual capacity (FRC):

1) Is the volume of gas in the lung at the end of a vital capacity breathe 2) Equals the residual volume plus the inspiratory reserve volume 3) Is measured by simple spirometry 4) Can be measured using body plethysmography

207. Closing capacity (CC):

1) Is a sensitive test of early small airway disease 2) If lies below the normal tidal ventilation, atelectasis will occur 3) Is measured by the use of a tracer gas such as Xenon-133 (Xe133) 4) Is decreased in obesity

208. Functional residual capacity (FRC):

1) Decreases significantly with the induction of general anesthesia 2) Reduction correlates with an increase in alveolar-arterial PO2 gradient during

anesthesia (A-a gradient) 3) Remains decreased into the postoperative period 4) Always decreases progressively during the course of anesthesia

209. Causes of increase in the volume of Zone 1 of pulmonary circulation include:

1) Deliberate hypotension 2) Positive end expiratory pressure (PEEP) 3) Pulmonary embolism 4) Increase in airway pressure

210. Moderate hypercapnia will cause:

1) Increased QT interval 2) Rise in plasma potassium 3) Increased myocardial oxygen demand 4) Decreased pulmonary vascular resistant

Page 84: 2004 Anesth Practice Ques eBook

Anesthesia Practice Questions: Book 1 – 2004 A=1,2,3 B=1,3 C=2,4 D=4 E=All

This material may not be duplicated without written permission from the publisher. Copyright 2004, All Rights Reserved Medtext Medical World, Inc.

84

211. In superior vena cava syndrome:

1) Malignancy is the underlying cause in 90% cases 2) Dyspnea and headache are the most common symptoms 3) Patient may present with changes in mental status 4) Changes in voice or stridor may occur

212. P-50 (P50):

1) Describes the position of the oxy-hemoglobin curve 2) When lower, may require higher than normal tissue perfusion to produce the

normal amount of oxygen unloading 3) Is lower in carbon monoxide poisoning 4) Is lower in methemoglobinemia

213. Bainbridge reflex:

1) Is stimulated by increases in intravascular volume 2) Causes increased heart rate 3) Sends afferent impulses through vagus nerve 4) Has its receptors within the right atrial wall

214. Baroreceptor reflex:

1) Responds to changes in blood pressure via stretch receptors present in the carotid sinus and aortic arch

2) Sends impulses along the afferent limbs of glossopharyngeal and vagus nerves

3) Typically begins to respond at pressures in excess of 170 mmHg 4) Response is increased contractility, heart rate and vascular tone

Page 85: 2004 Anesth Practice Ques eBook

Anesthesia Practice Questions: Book 1 – 2004 A=1,2,3 B=1,3 C=2,4 D=4 E=All

This material may not be duplicated without written permission from the publisher. Copyright 2004, All Rights Reserved Medtext Medical World, Inc.

85

215. Coronary blood flow:

1) Is principally driven by mean arterial pressure 2) Through the left side is maximal during peak systole 3) Exhibits some degree of autoregulation 4) Through the right side is maximal during early diastole

216. Which of the following statement(s) about the types of pacemakers and their

functions is/are TRUE?

1) A VVIR pacemaker senses electrical activity only in the ventricle 2) A VDD pace maker can pace and sense both the atrium and the ventricle 3) An AOO pacemaker is a fixed rate pace maker 4) When an AAI pacemaker senses an intrinsic electrical beat in the atrium, it

triggers the generator to fire 217. A 67-year-old man had a permanent endocardial VVI pacemaker placed 2 years ago

for complete heart block. He has a history of insulin dependent diabetes mellitus and angina on exertion. On the ECG tracing, the pacemaker spike appears at the proper time, but is not followed by the expected QRS complex.

This failure to capture may be caused by:

1) Battery failure 2) Hyperkalemia 3) Myocardial ischemia 4) Severe hyperglycemia

218. Mild hypothermia (≈ 34o C.):

1) Causes platelet dysfunction 2) Directly impairs enzymes of the coagulation cascade 3) Impairs immune function 4) Decreases wound oxygen delivery

Page 86: 2004 Anesth Practice Ques eBook

Anesthesia Practice Questions: Book 1 – 2004 A=1,2,3 B=1,3 C=2,4 D=4 E=All

This material may not be duplicated without written permission from the publisher. Copyright 2004, All Rights Reserved Medtext Medical World, Inc.

86

219. Postanesthetic shivering:

1) Increases oxygen consumption about 100% 2) Increases intraocular pressure 3) Can be treated by skin surface warming 4) Increases intracranial pressure

220. TRUE statements concerning perioperative temperature control include:

1) Operating room temperature is the most crucial factor influencing heat loss 2) One cotton blanket placed on the patient reduces heat loss by about 30% 3) The most effective perianesthetic warming system is forced air warming

blanket 4) Circulating water mattresses does not cause burns if the water temperature

is kept under 40o C. 221. Acute normovolemic hemodilution (ANH):

1) Is employed to reduce the need for allogenic red blood cells 2) Is contraindicated in the presence of malignancy 3) May improve tissue perfusion 4) Is contraindicated in the presence of wound infection

222. Potential complications associated with intraoperative use of the cell saver device

include:

1) Air and fat embolism 2) Rh isoimmunization during cesarean deliveries 3) Renal dysfunction secondary to lysis of red blood cells 4) Disseminated intravascular coagulation

Page 87: 2004 Anesth Practice Ques eBook

Anesthesia Practice Questions: Book 1 – 2004 A=1,2,3 B=1,3 C=2,4 D=4 E=All

This material may not be duplicated without written permission from the publisher. Copyright 2004, All Rights Reserved Medtext Medical World, Inc.

87

223. The following statements regarding the maternal mortality is/are TRUE:

1) Hemorrhage accounts for more than 25% of maternal deaths 2) In the United States, maternal deaths due to anesthesia related

complications have decreased within the last twenty years 3) Maternal case-fatality risk ratio for general anesthesia has shown to be 16

times that of regional anesthesia after 1985 4) The leading cause of maternal mortality is airway problems

224. The ligamentum flavum:

1) Developmentally, is a paired structure 2) May have midline gaps due to lack of fusion in the mid-line 3) Is thinnest at the cervical and high thoracic levels 4) Is composed of collagenous fibers

225. TRUE statements concerning the uterine blood flow at term include:

1) 20% of the uterine blood flow supplies the intervillous space 2) 80% of the uterine blood flow supplies the uterine myometrium 3) Uterine vessels are not sensitive to alpha-adrenergic agonists 4) Total uterine blood flow approaches 800 ml/min

226. Inhaled nitric oxide:

1) Acts by increasing cyclic guanosine monophosphate 2) Is a systemic and pulmonary vasodilator 3) May be potentiated by Sildenafil, phosphodiesterase-5 inhibitor 4) Causes systemic hypotension

Page 88: 2004 Anesth Practice Ques eBook

Anesthesia Practice Questions: Book 1 – 2004 A=1,2,3 B=1,3 C=2,4 D=4 E=All

This material may not be duplicated without written permission from the publisher. Copyright 2004, All Rights Reserved Medtext Medical World, Inc.

88

227. Potentiation of nondepolarizing neuromuscular blockade occurs in the presence of:

1) Metabolic acidosis 2) Respiratory acidosis 3) Decreased extracellular concentration of potassium 4) Cephalosporins

228. Factors that interfere with adequate reversal of nondepolarizing blockade include:

1) Respiratory acidosis (PaCO2 greater than 50 mmHg) 2) Hypothermia 3) Low extracellular potassium concentration 4) Magnesium sulfate

229. When compared with neostigmine, edrophonium:

1) Has less muscarinic effects 2) Has a quicker onset of action 3) Has fewer side effects 4) Is not broken down by acetylcholinesterase

230. In a myasthenia gravis patient on pyridostigmine therapy:

1) The sensitivity to non depolarizers may be diminished 2) The response to succinylcholine may be lengthened 3) The reversal of residual block at the end of the case may be ineffective 4) Often surgical relaxation can be provided using a potent inhaled anesthetic

only

Page 89: 2004 Anesth Practice Ques eBook

Anesthesia Practice Questions: Book 1 – 2004 A=1,2,3 B=1,3 C=2,4 D=4 E=All

This material may not be duplicated without written permission from the publisher. Copyright 2004, All Rights Reserved Medtext Medical World, Inc.

89

231. The following statement(s) is/are TRUE concerning the influence of severe renal failure on the pharmacology of nondepolarizing muscle relaxants:

1) The duration of mivacurium may be lengthened due to decrease in the

plasma cholinesterase activity 2) The elimination half life of laudanosine is unchanged 3) The duration of action of atracurium is not affected by renal failure 4) The volume of distribution of rocuronium is decreased in renal failure

232. Magnesium sulphate enhances the neuromuscular blocking properties of muscle

relaxants by:

1) Decreasing the amount of acetylcholine (Ach) released from the motor nerve terminal

2) Increasing the depolarizing action of the Ach on the postjunctional membrane

3) Decreasing the excitability of the muscle fiber itself 4) Increasing the amplitude of the end-plate potential

233. The following drugs enhance the effect of nondepolarizing muscle relaxants:

1) Local anesthetics 2) Phenytoin 3) Dantrolene 4) Carbamazepine

234. Action(s) of local anesthetics in the nerve membrane include:

1) Disruption of sodium channel activation process 2) Alteration in the threshold potential 3) Blockade of the ion-conducting pores 4) Alteration in the resting membrane potential

Page 90: 2004 Anesth Practice Ques eBook

Anesthesia Practice Questions: Book 1 – 2004 A=1,2,3 B=1,3 C=2,4 D=4 E=All

This material may not be duplicated without written permission from the publisher. Copyright 2004, All Rights Reserved Medtext Medical World, Inc.

90

235. The following statement(s) about amide and ester local anesthetics is/are TRUE:

1) Ester local anesthetics are relatively unstable in solution 2) Amide compounds undergo enzymatic degradation in the liver 3) Reports of allergic reactions to amide compounds are extremely rare 4) Amide local anesthetics are extremely stable in solution

236. The risk of developing local anesthetic toxicity is influenced by:

1) Rate of injection 2) Hypercarbia 3) Metabolic acidosis 4) Site of injection

237. Cardiovascular effects of induction dose of intravenous thiopental include:

1) Vasodilatation 2) Decreased myocardial contractility more than that of volatile anesthetics 3) Hypotension which is greater in hypertensive patient than in normotensive

patients 4) Increased heart rate more than after equivalent dose of methohexital

238. Which of the following drugs has the potential to occlude the intravenous line when

co-administered with thiopental?

1) Vecuronium 2) Atracurium 3) Sufentanil 4) Midazolam

Page 91: 2004 Anesth Practice Ques eBook

Anesthesia Practice Questions: Book 1 – 2004 A=1,2,3 B=1,3 C=2,4 D=4 E=All

This material may not be duplicated without written permission from the publisher. Copyright 2004, All Rights Reserved Medtext Medical World, Inc.

91

239. Esmolol:

1) Is a nonselective beta-receptor antagonist 2) May cause hypotension 3) Rapidly eliminated by plasma pseudocholinesterases 4) After discontinuation most drug effects are eliminated within 5 minutes

240. Signs of nitroprusside toxicity include:

1) Tachyphylaxis 2) Decreased mixed venous PO2 3) Metabolic acidosis 4) Cyanosis

241. In a cardiac transplanted patient:

1) Resting heart rate is low 2) Reflex bradycardic response is absent after carotid sinus massage 3) Cardiac output is increased primarily by increased heart rate 4) Response to laryngoscopy may be delayed or blunted

242. The following pharmacodynamic alterations may be seen in a cardiac transplanted

patient:

1) No change in heart rate with administration of atropine and glycopyrrolate 2) Increased response to epinephrine and norepinephrine 3) Absent reflex bradycardia or tachycardia in response to changes in systemic

arterial blood pressure 4) Reduced response to alpha-adrenergic agents

Page 92: 2004 Anesth Practice Ques eBook

Anesthesia Practice Questions: Book 1 – 2004 A=1,2,3 B=1,3 C=2,4 D=4 E=All

This material may not be duplicated without written permission from the publisher. Copyright 2004, All Rights Reserved Medtext Medical World, Inc.

92

243. Angiotensin II:

1) Is a direct vasoconstrictor 2) Enhances the pre junctional release of norepinephrine from the adrenergic

nerve ending 3) Increases efferent sympathetic nerve activity 4) Stimulates the secretion of aldosterone

244. Patients with high spinal cord transaction may develop:

1) Bradycardia with changes in position 2) Hyperthermia easily 3) Hypothermia easily 4) Decreased response to exogenous vasopressors

245. The following statements is/are TRUE regarding the patients requiring perioperative

corticoid supplementation:

1) Inadequate corticosteroid coverage can cause death 2) Administration of supraphysiologic doses of steroid for a short time

perioperatively causes no discernible complications. 3) Even topical application of steroid can suppress normal adrenal response for

as long as 9-12 months. 4) Plasma cortisol concentration of more than 25 µg/ml, measured during acute

stress, indicates normal pituitary–adrenal responsiveness 246. Etomidate:

1) Is an imidazole sedative-hypnotic 2) Causes compromised adrenal reserves for at least 24 hours after a single

induction dose 3) Causes minimal cardiovascular depression 4) Inhibits two essential adrenocortical

Page 93: 2004 Anesth Practice Ques eBook

Anesthesia Practice Questions: Book 1 – 2004 A=1,2,3 B=1,3 C=2,4 D=4 E=All

This material may not be duplicated without written permission from the publisher. Copyright 2004, All Rights Reserved Medtext Medical World, Inc.

93

247. Antagonist(s) used for prophylaxis and treatment of postoperative nausea and vomiting are specific for which chemoreceptor trigger zone receptor(s)?

1) Serotonin 2) Cholinergic 3) Dopamine 4) Histamine

248. Regarding thermoregulation, afferent signals from peripheral receptors that sense temperature travel primarily through which nerve fiber type(s)?

1) A - alpha fiber 2) A - delta fiber 3) A - gamma fiber 4) Unmyelinated C - fiber

249. Which of the following pharmacological factors favor the transfer of drugs across the placenta into the fetal circulation?

1) Low molecular weight 2) High degree of ionization 3) Low protein binding 4) Low lipid solubility

250. A child presents to the emergency room in respiratory distress and a history of dysphagia and drooling. You are called to the ER for possible intubation and securing of the airway. A diagnosis of epiglottitis is suspected. What other clinical characteristics are consistent with epiglottitis?

1) Insidious onset of symptoms 2) Low grade fever 3) Croupy (barking) cough 4) Usually affects children ages 2 to 6 years

Page 94: 2004 Anesth Practice Ques eBook
Page 95: 2004 Anesth Practice Ques eBook

Answers

Page 96: 2004 Anesth Practice Ques eBook
Page 97: 2004 Anesth Practice Ques eBook

Anesthesia Practice Questions: Book 1 – 2004

This material may not be duplicated without written permission from the publisher. Copyright 2004, All Rights Reserved Medtext Medical World, Inc.

97

1. E Umbilical cord compression High doses of bupivacaine can produce peripheral vasodilation, which in turn produce

peripheral venous pooling. This drops maternal blood pressure producing an acute

decrease in umbilical blood flow. Loss of fetal beat-to-beat variability results from umbilical

cord compression or severe hypotension leading to fetal hypoxemia. The treatment include

administration of supplemental oxygen to the mother, altering maternal position to the left

side in order to relieve pressure from gravid uterus on great vessels and correcting

maternal hypotension.

1. Longnecker DE, Tinker JH, Morgan GE (eds.) Principles and Practice of Anesthesiology. 2nd edition. Mosby Baltimore MD. 1998:1997. 2. A Inform the patient and take no action The presence of a small piece of epidural catheter in the epidural space is less likely to

pose any problem. Currently, the standard of care for retention of a segment of epidural

catheter in epidural space is to leave the segment in the space, inform the patient of

mishap and follow up the patient with neurological examination. However, if a continuous

spinal anesthesia is used and catheter tip gets broken or separated in the subarachnoid

space, patient should be followed for neurological symptoms and neurosurgical consult be

sought for possible retrieval of the broken segment. Use of antibiotics is not advisable.

1. Tio To, Macmurdo SD, McKenzie R: Mishap with an epidural catheter. Anesthesiology 1979;50:260-262. 3. B Hypertension The dose required to achieve adequate epidural anesthesia can cause serious side effects if

injected into the subarachnoid space or into a blood vessel, therefore a test dose is often

administered before injection. The test dose often consists of a local anesthetic and

epinephrine. The objective of test dose is to rule out intravascular or subarachnoid

injection. During administration of a test dose, 15 micrograms of epinephrine is

administered which should increase the heart rate by 20% within 60 seconds if the catheter

Page 98: 2004 Anesth Practice Ques eBook

Anesthesia Practice Questions: Book 1 – 2004

This material may not be duplicated without written permission from the publisher. Copyright 2004, All Rights Reserved Medtext Medical World, Inc.

98

is intravascular in the epidural vein. This same test dose should produce a sensory block if

it is in the subarachnoid space. Of the choices given, only choice B is the most plausible.

This response (hypertension) will be much more marked if the patient had been on beta-

blocker due to unopposed alpha effects.

1. Longnecker DE, Tinker JH, Morgan GE (eds). Principles and Practice of Anesthesiology. 2nd edition. Mosby Baltimore MD. 1998:1401. 4. D Intercostal Serum concentration depends upon the vascularity at the site of injection. The more

vascularized the tissue, the greater the systemic absorption of the local anesthetic and the

increased potential for toxicity. In descending order, the rate of developing serum

concentration of local anesthetics from highest to lowest is as follows: intravenous >

tracheal > intercostal > caudal > paracervical > epidural > brachial > local nerves >

subcutaneous.

1. Morgan GE, Mikhail MS. Clinical Anesthesiology. 2nd edition Appleton and Lange CT. 2002:235. 5. A Fentanyl when combined with epidural bupivacaine will decrease the concentration of the later for satisfactory analgesia Epidural opioids combined with local anesthetics are increasingly used for optimal

management of intraoperative and postoperative analgesia. Local anesthetic when

combined with opioids reduces the local anesthetic dose while equaling or sometimes

improving analgesia. Opioids block the pain transmission by binding at presynaptic and

postsynaptic receptor sites in the spinal cord. Inclusion of opioids can produce side effects

such as respiratory depression, nausea, pruritus and urinary retention. Of the choices given

one can argue that choice D is correct as well because the addition of epinephrine would

decrease the rate of vascular absorption therefore decreasing the systematic opioid blood

concentration and further improving the depth and duration of analgesia. However, this

combination may not significantly prolong the duration of fentanyl or sufentanil analgesia,

Page 99: 2004 Anesth Practice Ques eBook

Anesthesia Practice Questions: Book 1 – 2004

This material may not be duplicated without written permission from the publisher. Copyright 2004, All Rights Reserved Medtext Medical World, Inc.

99

since it is lipid solubility that determines the onset of action of epidural placed opioids. Both

fentanyl and sufentanil are highly lipid soluble opioids.

1. Longnecker DE, Tinker JH, Morgan GE (eds). Principles and Practice of Anesthesiology. 2nd edition. Mosby. Baltimore MD. 1998:1400. 6. E Bupivacaine blocks cardiac sodium channels for prolonged period Cardiac toxicity varies for each local anesthetic with bupivacaine being more cardiotoxic

than lidocaine. Intravenous injection of bupivacaine may results in hypotension, ventricular

tachycardia, fibrillation, and AV heart block. Bupivacaine binds to sodium channel tightly

and dissociates slowly, leading to its prolonged and exaggerated effects. It depresses the

rapid phase of depolarization in Purkinje fibers and ventricular muscles. Bupivacaine does

not enhance the Na-K exchange in the myocardium or increase the sensitivity of

myocardium adrenergic receptors to catecholamines. It has no effect on myocardial Ca-

channels and is highly lipid soluble. After accidental IV injection, the protein binding sites

(alpha1-acid glycoprotein and albumin) are quickly saturated, leaving a significant mass of

unbound drug available for diffusion into the conducting tissue of the heart. Cardiotoxic

plasma concentrations of bupivacaine are 8 to 10 µg/ml.

1. Stoelting RK, Miller RD. Basics of Anesthesia. 4th edition. Churchill Livingstone. New York NY. 2000:85-6. 7. A Inject 3 ml of local anesthetics lateral to biceps tendon at the flexion of the crease For an incomplete brachial plexus block that spares radial nerve distribution, radial nerve

block is performed as a supplement. The radial nerve is a terminal branch of posterior cord

of brachial plexus and gives rise to the lateral cutaneous nerve of the arm and posterior

cutaneous nerve of the forearm (both sensory). At the lateral epicondyle the radial nerve

branches into a superficial branch (which innervates the radial aspect of wrist and

dorsolateral aspect of 3 1/2 digits) and a deep branch (which innervates the extensor

group of forearm muscles). The supplemental block is performed by injecting 3 to 5 ml of

Page 100: 2004 Anesth Practice Ques eBook

Anesthesia Practice Questions: Book 1 – 2004

This material may not be duplicated without written permission from the publisher. Copyright 2004, All Rights Reserved Medtext Medical World, Inc.

100

local anesthetics at the lateral aspect of biceps tendon at the flexion crease. The other

nerve, which often fails to be blocked, is the musculocutaneous nerve. This nerve is

blocked by injecting 5-8 ml of local anesthetics into the belly of coracobrachialis muscle.

1. Morgan GE, Mikhail MS. Clinical Anesthesiology. 3rd edition. McGraw Hill, New York NY. 2002: 292 – 293. 8. E Recurrent laryngeal nerve block An interscalene block is performed for procedures involving the upper extremity, including

the shoulder. The interscalene block is performed by injecting 40 ml of local anesthetics

between anterior and middle scalene muscles at the level of cricoid cartilage. Complications

of interscalene block include: recurrent laryngeal nerve block which this patient has,

injection into the vertebral artery (leading to seizure), epidural and subarachnoid injections,

pneumothorax, and stellate ganglion block leading to Horner’s syndrome (miosis, ptosis,

anhydrosis, nasal congestion, vasodilation and increased skin temperature). Besides these,

infection, hematoma and nerve injury may also occur.

1. Morgan GE, Mikhail MS. Clinical Anesthesia. 3rd edition, McGraw Hill, New York NY. 2002: 288-289. 9. B Most intense analgesia with epidural administration Urinary retention after neuroaxial block with opioids is more common than after

intravenous administration. This is not dose dependent or related to systematic absorption

but rather due to interaction of opioids with receptors located in the sacral spinal cord.

Keep in mind that epidural administration of morphine produces more intense analgesia,

longer duration of action and a greater incidence of pruritus. Analgesia that follows epidural

placement of morphine reflects diffusion across the dura to gain access to mu opioid

receptors on the spinal cord as well systematic absorption into the circulation. Since

morphine is poorly lipid soluble it has slower onset of analgesia but longer duration of

action. In contrast, analgesia by epidural administration of highly lipid soluble opioids

(fentanyl, sufentanil) is primarily a reflection of systematic absorption.

1. Stoelting RK. Pharmacology and Physiology of Anesthesia. 3rd edition. Lippincott-Raven. New York NY. 1999: 79 – 83.

Page 101: 2004 Anesth Practice Ques eBook

Anesthesia Practice Questions: Book 1 – 2004

This material may not be duplicated without written permission from the publisher. Copyright 2004, All Rights Reserved Medtext Medical World, Inc.

101

10. C Faster onset of analgesia Fentanyl is a more lipid soluble opioid when compared to morphine. Fentanyl’s greater

potency and more rapid onset of action compared with morphine, reflects this higher lipid

solubility which facilitated its passage across the blood-brain barrier. The short duration of

action of fentanyl reflects its rapid redistribution to inactive tissue sites such as fat and

skeletal muscles. In contrast, morphine is poorly lipid soluble, is absorbed slowly to reach

its site of action in the CNS. Other reasons for poor penetration of morphine into the CNS

(< 0.1%) include: high degree of ionization and rapid conjugation with glucuronic acid in

liver and kidney. Once inside CNS it diffuses back into circulation slowly. This increases

the duration of action significantly when compared to fentanyl. There is no difference in

the sensitivity of various opioids to naloxone.

1. Stoelting, RK. Pharmacology and Physiology of Anesthesia. 3rd edition. Lippincott-Raven. New York NY. 1999: 92-93. 11. E Subarachnoid injection This question emphasizes the complications of interscalene block, which include:

subarachnoid (as in this case) or epidural injections, nerve blocks (phrenic and recurrent

laryngeal nerves), and stellate ganglion block (ptosis, anhydrosis, miosis, enophthalmos,

nasal congestion, vasodilation and increased skin temperature). Absorption into systematic

circulation due to proximity of vertebral artery can result in CNS toxicity (convulsions).

Besides, because of close proximity of cervical neural foramina, inadvertent injection into

epidural and subarachnoid space is not uncommon. This may produce high epidural or high

spinal anesthesia leading to apnea. Other common complications of interscalene block are:

phrenic and/or laryngeal nerves block with associated hemiparesis of the diaphragm and

laryngeal muscles. The risk of pneumothorax is remote.

1. Morgan GE, Mikhail MS, Murray MJ. Clinical Anesthesia. 3rd edition. McGraw-Hill New York NY. 2002:289.

Page 102: 2004 Anesth Practice Ques eBook

Anesthesia Practice Questions: Book 1 – 2004

This material may not be duplicated without written permission from the publisher. Copyright 2004, All Rights Reserved Medtext Medical World, Inc.

102

12. B Enhance peristalsis with diarrhea and manifestation of hypotension Celiac plexus is formed by the union of the greater (T5-T10), lesser (T10-T11), and least

(T12) splanchnic nerves and the celiac branch of right vagus. It contains both sympathetic

and parasympathetic fibers. This extensive network of ganglia and nerves is located at the

level of first lumbar vertebra in the retroperitoneal space along the aorta. Fibers from this

ganglion carry pain sensation from many of the intraperitoneal organs such as the pancreas

and liver. Celiac plexus block is a sympathetic block, commonly applied for the relief of pain

from malignancy of the pancreas, liver, or other upper abdominal organs. A successful

block is indicated by disappearance of pain in cancer patients or postural hypotension and

diarrhea in normal patients. The later symptoms are due to the preponderance of

parasympathetic response. Choice D is more of a function of sacral plexus. Celiac plexus

does not supply somatic fibers to pancreas.

1. Wildsmith JAW, Armitage EN, McClure JH. Principles and Practice of Regional Anesthesia. 3rd edition. Churchill Livingstone. 2003:301. 13. D Wait 20 minutes and then deflate and re-inflate immediately, and finally

deflate after 1 minute For short procedures involving extremities Bier block can be performed which is the

injection of local anesthetics (50 ml 0.5% lidocaine) into the venous system below an

occluding tourniquet. If surgery is completed in less than 20 minutes, the tourniquet is left

inflated to avoid sudden absorption of local anesthetics into the systematic circulation

which can produce cardiac and CNS toxicity. If the surgery is completed between 20 and 40

minutes, the cuff can be deflated and reinflated immediately, thus releasing small amount

of local anesthetics into the systematic circulation each time. Finally after 1 minute it is

deflated completely. However, one can deflate the cuff as a single maneuver after 40

minutes because most of the local anesthetics has been metabolized and chances of local

anesthetic toxicity is insignificant.

1. Stoelting RK, Miller RD. Basics of Anesthesia. 4th edition. Churchill Livingstone. 2000: 193-195.

Page 103: 2004 Anesth Practice Ques eBook

Anesthesia Practice Questions: Book 1 – 2004

This material may not be duplicated without written permission from the publisher. Copyright 2004, All Rights Reserved Medtext Medical World, Inc.

103

14. D PDPH is less frequent if the needle bevel is parallel to the direction of dural fibers Post-dural puncture headache (PDPH) is most often seen following an epidural block due to

wet tap. It is believed due to loss of CSF through the meningeal needle hole resulting in

decreased buoyant support for the brain and increased tension on meningeal vessels and

nerves. The headache is frontal or occipital and postural in nature which worsens in the

sitting or standing position. The headache may be associated with diplopia, tinnitus and

decreased hearing acuity. Non-cutting needles (Greene, Whitcare and Sprott) have lower

incidence of headache than the cutting needles (Quincke, Pitkin). There is lower incidence

of PDPH if the needle bevel is parallel to the meningeal fibers (that is the bevel points in

the lateral direction if the patient is in the sitting position). There is higher incidence of

PDPH in young, pregnant females and with the use of lower gauge needles. Treatment

include: bedrest, analgesics (Tylenol double strength), oral or intravenous hydration, and

administration of caffeine sodium benzoate (500 mg iv) and blood patch. Although,

controversial, it is likely that a prophylactic epidural blood patch may be effective in

preventing postdural puncture headache in patient with accidental dural puncture.

1. Stoelting RK, Miller RD. Basics of Anesthesia. 4th edition. Churchill Livingstone. 2000: 177. 15. D Musculocutaneous nerve The brachial plexus is formed from the anterior rami of C5 - T1. There are three

approaches (interscalene, supraclavicular and axillary blocks) to block brachial plexus based

on anatomic locations where local anesthetic solutions are placed. The anesthesia

produced with each approach is significantly different in terms of its usefulness. For

example in axillary approach of brachial plexus block, musculocutaneous and medial

antebrachial cutaneous nerves may be missed, thus this approach may produce inadequate

anesthesia of forearm. The Musculocutaneous nerve (C5-C7) leaves the axillary sheath

proximal to the point of injection into the axilla. Its principal sensory branch is, lateral

cutaneous nerve of the forearm supplying thenar eminence and motor branch to upper arm

flexors. The musculocutaneous nerve can be blocked by injecting 10 ml of local

Page 104: 2004 Anesth Practice Ques eBook

Anesthesia Practice Questions: Book 1 – 2004

This material may not be duplicated without written permission from the publisher. Copyright 2004, All Rights Reserved Medtext Medical World, Inc.

104

anesthetics 5 cm proximal to the elbow crease between biceps and brachialis muscles into

the substance of coracobrachialis muscle.

1. Stoelting RK, Miller RD. Basics of Anesthesia. 4th edition. Churchill Livingstone. 2000:189. 16. A Sural nerve Sciatic nerve is the main nerve of the lower extremity which divides into anterior and

posterior tibial nerves near popliteal fossa. Posterior tibial nerve supplies the sole of the

foot and is blocked by injecting 5 ml of local anesthetic posterior to medial malleolus,

behind posterior tibial artery. Injecting 5 ml of local anesthetics anterior to the medial

malleolus can block the saphenous nerve, a branch of femoral nerve which innervates a

strip along the medial aspect of foot. Sural nerve innervates the lateral aspect of the foot

and little toe. It is blocked by injecting 5 ml of local anesthetic solution between the lateral

malleolus and calcaneus. Deep peroneal nerve supplies the dorsum of the foot, lies

between the anterior tibial artery and the tendon of the anterior tibial muscle and

innervates the skin between the first and second toes and the short extensors of the toes.

It is blocked by injecting 5 ml of local anesthetics anterior to medial malleolus lateral to

anterior tibial artery. Injecting 5 ml of local anesthetics as a subcutaneous ridge between

medial and lateral malleolus blocks superficial peroneal nerve.

1. Stoelting RK, Miller RD. Basics of Anesthesia. 4th edition. Churchill Livingstone. New York, NY. 2000:192. 17. E Substantia gelatinosa Morphine acts at multiple sites (brain, spinal and peripheral tissues) and involves effects on

mu1, mu2, sigma and delta receptors. Morphine and related opioids selectively act on

neurons and neurotransmitters that transmit and modulate nociception. At the spinal cord

level morphine hyperpolarizes the neurons in the substantia gelatinosa via the mu2

receptors of the dorsal spinal cord thus decreasing the afferent transmission of nociceptive

impulses to the brain. Morphine also decreases the release of neurotransmitters involved

in the transmission of nociception such as substance P in the dorsal horn of spinal cord.

1. Barash PG, Cullen BF, Stoelting RK. Handbook of Clinical Anesthesia. 4th edition. Lippincott Williams & Wilkins. 2001:161.

Page 105: 2004 Anesth Practice Ques eBook

Anesthesia Practice Questions: Book 1 – 2004

This material may not be duplicated without written permission from the publisher. Copyright 2004, All Rights Reserved Medtext Medical World, Inc.

105

18. B Formation of increased ionized fraction of local anesthetics at the site of infection due to local tissue acidosis Local anesthetics exist in ionized and non ionized forms at physiologic pH. However, the

ratio of the two forms (non ionized to ionized) varies depending on the Pka of local

anesthetics and the pH of surrounding media. The non ionized form is the lipid soluble

form and is the primary determinant of local anesthetic potency. Since the non ionized

form is lipid soluble, it crosses the lipophilic nerve sheath to gain access to sodium channels

in the nerve membrane. Once inside the nerve sheath, the non ionized form of local

anesthetic is converted to the ionized form because of a lower pH. The ionized form binds

to Na channels to block nerve conduction. In this question, tissue necrosis, lactic acid

production at the infection site increases the ionized fraction of lidocaine (ionized form is

not lipid soluble) which limited the diffusion of local anesthetics through the nerve sheath

to gain access into the nerve to block impulse conduction. As a result infiltration of local

anesthetic produced a very poor block.

1. Stoelting RK, Miller RD. Basics of Anesthesia. 4th edition. Lippincott Williams & Wilkins. 2000:83. 19. B Bupivacaine induced sympathectomy leading to significant decrease in pre load due to peripheral vasodilation and venous pooling During a TURP procedure the patient is usually in the lithotomy position. The resulting

sympathectomy produced by bupivacaine after a spinal will not significantly alter the

preload since little pooling is occurring in the lower extremities while in the lithotomy

position. Transfer of the patient to a regular bed reverses this and results in significant

venous pooling in the lower extremities. This lead to profound hypotension with/without

nausea and vomiting due to decrease in venous return to the heart, decrease cardiac

output and decrease systematic vascular resistance. Administration of phenylephrine would

have been an appropriate action.

1. Stoelting RK and Miller RD. Basics of Anesthesia. 4th edition. Lippincott Williams & Wilkins. 2000:177.

Page 106: 2004 Anesth Practice Ques eBook

Anesthesia Practice Questions: Book 1 – 2004

This material may not be duplicated without written permission from the publisher. Copyright 2004, All Rights Reserved Medtext Medical World, Inc.

106

20. B Prilocaine can cause fetal methemoglobinemia Of all the amide drugs prilocaine is one of the safest drugs. It is equipotent with lidocaine

and has slightly longer duration of action. It is metabolized to O-toluidine, which can

reduce hemoglobin to methemoglobin. It is not used during labor anesthesia because “top

up” dose may be higher than the toxic dose, which may cause fetal cyanosis.

Methemoglobinemia is treated by administration of methylene blue or ascorbic acid. The

later is not used as often.

1. Wildsmith JAW, Armitage EN, McClure JH. Principles and Practice of Regional Anesthesia. 3rd edition. Churchill Livingstone. 2003:70. 21. C Obtain MRI of the lumbar spine and request a neurological consult It is extremely difficult to detect any bleeding into the epidural space, which may go

undetected. The earliest neurological symptoms and signs (back pain and pressure)

caused by hematoma are easily confused with epidural block. Time is of the essence

because, if the spinal cord compression persists for longer than 6-12 hours, catastrophic

paralysis may result due to spinal cord compression. If the sensory or motor losses

progress or outlast the expected duration of action of local anesthetics, neurological advice

should be sought and both CT and MRI be ordered. If a hematoma is diagnosed, then the

patient must have an emergent laminectomy and decompression to avoid the risk of

paralysis. Coagulopathy therefore represents a relative contraindication. The degree of

coagulopathy at which it becomes unsafe to perform regional anesthesia is highly

controversial.

1. Wildsmith JAW, Armitage EN, McClure JH. Principles and Practice of Regional Anesthesia. 3rd edition. Churchill Livingstone. 2003: 162.

Page 107: 2004 Anesth Practice Ques eBook

Anesthesia Practice Questions: Book 1 – 2004

This material may not be duplicated without written permission from the publisher. Copyright 2004, All Rights Reserved Medtext Medical World, Inc.

107

22. E Sacral cornu Caudal block provides excellent pain control and reduces the stress response to surgery.

Accurate location of sacral cornu, which represents unfused laminae of the fifth sacral

segment, is essential for performing a caudal block. The posterior superior iliac spine,

coccyx, greater trochanter of femur and iliac crest are the landmarks for a sciatic nerve

block using the posterior approach. In the posterior approach to sciatic nerve block a

straight line is drawn connecting the greater trochanter and posterior superior iliac spine.

A perpendicular line is drawn from the mid point inferiorly. A 3.5 inch, 22 gauge needle is

inserted 5 cm caudad, and 25 ml of local anesthetics is injected. This point of needle

insertion should lie on a line drawn between the coccyx and the top of greater trochanter.

1. Wildsmith JAW, Armitage EN, McClure JH. Principles and Practice of Regional Anesthesia. 4th edition. Churchill Livingstone. 2003:220. 23. D Increased protein binding Lipid solubility of local anesthetics correlates with potency, which increases as the total

number of carbon atoms in the molecule increase. Onset of action depends upon relative

concentration of non-ionized (lipid soluble) to ionized water-soluble form. Only lipid soluble

form diffuses across the neural sheath and nerve membranes. Once inside the cell only the

charged cation forms actually bind to the receptor. Local anesthetics with ester bond are

hydrolyzed by esterases. The duration of action of local anesthetics is associated with

protein binding (alpha-1 acid glycoprotein).

1. Morgan GE, Mikhail MS, Murray MJ. Clinical Anesthesia. 3rd edition. McGraw Hill. New- York. 2002: 235. 24. A Horner’s syndrome due to cervical sympathetic block Interscalene block is one of the three approaches for brachial plexus block. The

complications associated with this approach are related to the structures located in the

vicinity of block. The Horner’s syndrome due to spread of local anesthetic to cervical

sympathetic chain on the anterior vertebral body is common. The symptoms of this patient

Page 108: 2004 Anesth Practice Ques eBook

Anesthesia Practice Questions: Book 1 – 2004

This material may not be duplicated without written permission from the publisher. Copyright 2004, All Rights Reserved Medtext Medical World, Inc.

108

are due to Horner’s syndrome which consists of: miosis, anhydrosis, ptosis, enophthalmos,

flushing and sweating. Proximity of other nerves (phrenic and recurrent laryngeal) makes

them susceptible to block manifested as dyspnea and hoarseness. Other complications

include intra-arterial injection into the vertebral artery, which can produce seizure and

inadvertent injection into epidural, subarachnoid and subdural spaces manifested as apnea.

Advancing needle too far, especially in the lateral direction, can result in a pneumothorax.

1. Morgan EG, Mikhail MS, Murray MJ. Clinical Anesthesia. 3rd edition. McGraw Hill. New York NY. 2002:289. 25. A Using a local anesthetic with high protein binding Important factors one should keep in mind while performing the epidural block include

onset, duration, quality (density of block) and spread of block. The following factors will

increase the duration of epidural block:

(i) Increased protein binding

(ii) Addition of vasoconstrictors

(iii) Increased concentrations

The later also increase the quality and onset of block. The spread of block is influenced by

volume, age, pregnancy, site (greater spread with thoracic than lumbar epidural), obesity

and height.

1. Wildsmith JAW, Armitage EN, McClure JH. Principles and Practice of Regional Anesthesia. 3rd edition. Churchill Livingstone. 2003:151. 26. E Common peroneal nerve injury The most common lower extremity nerves damaged because of compression or stretching

from improper positioning or improper padding are: peroneal, sciatic, saphenous and

occasionally obturator or posterior tibial nerves. The common peroneal nerve is a branch of

the sciatic nerve which branches into posterior tibial and peroneal nerves. Peroneal nerve

Page 109: 2004 Anesth Practice Ques eBook

Anesthesia Practice Questions: Book 1 – 2004

This material may not be duplicated without written permission from the publisher. Copyright 2004, All Rights Reserved Medtext Medical World, Inc.

109

runs behind the head of fibula and around its neck. Its stimulation evokes dorsiflexion of

foot. It is the most frequently damaged nerve in the lithotomy position due to compression

between the head of fibula and the metal frame used to support the leg. Proper padding

and other precautions to decrease the incidence of this type of injury are essential. The

injury to the common peroneal nerve manifests as foot drop, loss of dorsal extension of

toes and inability to evert the foot.

1. Stoelting RK, Miller RD. Basics of Anesthesia. Churchill and Livingstone. 4th edition. 2000:206. 27. E Urinary retention

The disadvantage of epidural opioids is in the increase in unwanted effects such as nausea,

vomiting, pruritus, respiratory depression and urinary retention. Pruritus, nausea, vomiting

and respiratory depression can be reversed by smaller doses of naloxone without reversing

opioid analgesia. You may need higher doses 0.5 µg/kg to 2 µg/kg repeated at 10 minutes

interval to reverse the urinary retention. Urinary retention, seen after both regional and

systematic morphine administration, is caused by complex effects on central and peripheral

neurogenic mechanisms which results in dyssynergia between the bladder detrusor muscle

and the urethral sphincter relaxation. Also, urinary retention seems to be mediated

through several receptors. This may partly explain the resistance to naloxone

administration. Alternatively restricting the opioid dose can minimize the impact of side

effects.

1. Wildsmith JAW, Armitage EN, McClure JH. Principles and Practice of Regional Anesthesia. Churchill and Livingstone. 3rd edition. 2003:279. 28. B Duration of tourniquet inflation Following exsanguinations of arm or leg with an Esmarch bandage, the tourniquet is

inflated 300 mmHg or about 2.5 times the patient’s systolic blood pressure and 50 ml of

local anesthetics solution is administered. The duration of anesthesia depends upon the

duration of tourniquet inflation and not on other properties of local anesthetics such as

Page 110: 2004 Anesth Practice Ques eBook

Anesthesia Practice Questions: Book 1 – 2004

This material may not be duplicated without written permission from the publisher. Copyright 2004, All Rights Reserved Medtext Medical World, Inc.

110

pKa, protein binding or the class of local anesthetics (ester or amide). Since the

distribution of local anesthetic is localized, a significant amount of local anesthetic

continues to diffuse through the blood and the nerve sheath to reach the nerve endings to

block the conduction without reaching the systematic circulation. Therefore, so long as the

tourniquet remains inflated, local anesthetic will continue to block nerve conduction.

Interestingly, analgesia produced with ropivacaine will be longer lasting than with lidocaine

though both produce comparable regional anesthesia.

1. Stoelting RK, Miller RD. Basics of Anesthesia. 4th edition. Churchill and Livingstone. 2000:195. 29. A Femoral, lateral femoral cutaneous and sciatic nerves Lumbosacral plexus (L2-S3) is the main nerve supply of lower extremity. The hip and knee

joints are supplied by femoral, sciatic and obturator nerves. For unilateral operation below

the knee, femoral and sciatic nerves block are sufficient to produce surgical anesthesia

because both sensory and motor innervations come from sciatic and femoral nerves.

However, above the knee block of these two nerves (sciatic and femoral) is not sufficient to

produce surgical anesthesia. For surgeries above the knee, one needs to block additional

nerves which include: lateral femoral cutaneous nerve block or three-in-one block, which is

block of femoral, lateral femoral cutaneous and obturator nerves. This block also provides

analgesia for tourniquet application during the surgical procedure.

1. Wildsmith JAW, Armitage EN, McClure JH. Principles and Practice of Regional Anesthesia. Churchill and Livingstone. 3rd edition. 2003:219. 30. A Inform the surgeon of possible bladder perforation and ask for termination of the procedure as soon as possible The incidence of bladder perforation during a TURP is approximately 1%. An awake patient

under regional anesthesia complaining of nausea, diaphoresis and lower abdominal pain

most likely has a bladder perforation. In contrast, patients under general anesthesia may

only show hypo- or hypertension with associated bradycardia. Other complications of TURP

are hemorrhage, TURP syndrome, hypothermia, septicemia and disseminated intravascular

Page 111: 2004 Anesth Practice Ques eBook

Anesthesia Practice Questions: Book 1 – 2004

This material may not be duplicated without written permission from the publisher. Copyright 2004, All Rights Reserved Medtext Medical World, Inc.

111

coagulation. TURP syndrome is manifested as headache, restlessness, confusion,

hypotension or hypertension or seizures. These symptoms are due to circulatory fluid

overload from hypotonic non- electrolyte solution such as glycine 1.5% or a mixture of

sorbitol 2.7% and mannitol 0.54%. Other complications are hyponatremia, hyposmolarity,

fluid overload, hemolysis, and solute toxicity (glycine and sorbitol).

1. Morgan GE, Mikhail MS, Murray MJ. Clinical Anesthesiology. 3rd edition. McGraw Hill. New York NY. 2002:695. 31. E Verapamil Direct cardioversion may be used to terminate supraventricular and ventricular tachycardia.

Digitalis induced tachycardia or tachycardia due to multifocal foci are not responsive to

cardioversion. Had this patient received general anesthesia, a cardioversion would have

been an appropriate treatment. However, this patient is awake and under the

circumstances choice E, verapamil is the best choice. Verapamil is highly effective in

terminating the paroxysmal supraventricular tachycardia either due to atrial fibrillation or

flutter.

1. Stoelting RK. Pharmacology and Physiology in Anesthetics Practice. 3rd edition. Lippincott-Raven publishers. Philadelphia. 1999:341. 2. Morgan EG, Mikhail MS, Murray MJ. Clinical Anesthesiology. 3rd edition. McGraw Hill New York NY. 2002:472. 32. D Epidural hematoma The neurological symptoms and signs of epidural hematoma can be very confusing as they

are very similar to epidural block. Continued progression or prolonged duration of block

should alert the anesthesiologist and a neurological consult with further work up be sought.

Anterior spinal artery ischemia manifests itself as painless paralysis of the legs and

sphincters and generally is due to profound hypotension or surgical interruption of the

blood supply to the cord. Important signs of epidural abscess are pyrexia and leukocytosis

with associated back pain and tenderness. They generally appear 3 –4 days after the block.

MRI of spinal cord and determination of CSF protein levels are very helpful in the diagnosis.

Page 112: 2004 Anesth Practice Ques eBook

Anesthesia Practice Questions: Book 1 – 2004

This material may not be duplicated without written permission from the publisher. Copyright 2004, All Rights Reserved Medtext Medical World, Inc.

112

Staphylococcus aureus is the most common causative agent. Adhesive arachnoiditis is due

to infection or the presence of detergent in local anesthetics preparation.

1. Wildsmith JAW, Armitage EN, McClure JH. Principles and Practice of Regional Anesthesia. Churchill and Livingstone. 3rd edition. McGraw Hill New York NY. 2002:136, 162-164. 33. C Celiac plexus block with alcohol Celiac plexus block is a sympathetic block, which provides pain relief from malignancy of

upper abdominal organs particularly pancreas. The ganglion is located retroperitoneally at

the lower part of the twelfth thoracic and upper part of first lumbar vertebrae. Initially a

trial diagnostic block is performed using 20-25 ml of 0.75% lidocaine or 0.25% bupivacaine

under fluoroscopic control. If trial is successful (seen as a pain relief), neurolytic block with

50% alcohol in 1% lidocaine or 7% phenol in water is performed 24-hours later. Other

blocks mentioned in the question will not completely relieve the intra-abdominal pain.

The complications (hypotension and diarrhea) of the block are generally temporary and are

due to parasympathetic predominance. Other rare complications include: intramuscular,

intrathecal, epidural injections, sexual dysfunction, pneumothorax, bowel perforation,

kidney or liver puncture, retro-peritoneal hemorrhage or paraplegia secondary to spinal

cord ischemia.

1. Barash PG, Bruce CF, Stoelting,RK. Clinical Anesthesia. Lippincott-Raven Publishers. 3rd edition. 1997:371-372. 34. A Precipitation when mixed with an equal volume of thiopental Most of CSF is formed in the choroids plexus while a small fraction (10%) is derived from

brain substance. CSF is removed via the arachnoid villi. The CSF maintains a physiologic

stable environment. It is a colorless fluid with specific gravity of 1.003 to 1.009, pH 7.39-

7.5, contains glucose 40-80 mg/dl, protein 15-45 mg/dL, sodium 138 mEq/L, potassium 2-3

mEq/L, calcium 2-3 mEq/L magnesium 2-3 mEq/L, chloride 1-4 mEq/L. The normal

pressure of CSF ranges between 60 and 150 mmHg. The presence of CSF can be

Page 113: 2004 Anesth Practice Ques eBook

Anesthesia Practice Questions: Book 1 – 2004

This material may not be duplicated without written permission from the publisher. Copyright 2004, All Rights Reserved Medtext Medical World, Inc.

113

confirmed by the formation of precipitation when mixed with an equal volume of thiopental

or by measuring glucose.

1. Longnecker DE, Tinker JH, Morgan GE. Principles and Practice of Anesthesiology. 2nd edition. Mosby Publishing Co. 1998:1367. 35. C Administration of nebulized racemic epinephrine Onset of inspiratory stridor in a postoperative patient can be due to laryngospasm,

laryngeal edema or vocal cord dysfunction. Laryngospasm can be triggered by excessive

secretions which stimulates the superior laryngeal nerve. Laryngeal edema may be caused

by an allergic drug reaction or a traumatic intubation. Subglottic edema in children is

produced when intubated with a large size tube with no leak around the tube.

Administration of humidified oxygen is usually required. Racemic epinephrine (0.5 ml of a

2% solution diluted to a volume of 2 to 4 ml) administered by a nebulizer is indicated if

symptoms persist. Administration of glycopyrrolate or corticosteroids will not help to

resolve the laryngeal edema.

1. Morgan GE, Mikhail MS, Murray MJ, (eds.). Clinical Anesthesiology. 3rd edition. Lange Medical Books/McGraw-Hill Medical Publishing Division. New York, NY. 2002:780. 36. D Decrease Decrease Presence of a large air bubble in the blood containing syringe will decrease both PaO2 and

PaCO2 measured because both O2 and CO2 diffuse from the blood into the bubble. This is

particularly true if blood gas measurement is carried out at higher oxygen partial pressures

or when the patient is on high FiO2 during mechanical ventilation because of greater

diffusion gradient. Thus removal of air bubble, before capping the syringe and placing it in

the ice-water, is essential. This will not affect oxygen binding to hemoglobin or oxygen

saturation.

1. Miller RD (eds.). Anesthesia. 5th edition. Churchill Livingstone. New York, NY. 2000:1263.

Page 114: 2004 Anesth Practice Ques eBook

Anesthesia Practice Questions: Book 1 – 2004

This material may not be duplicated without written permission from the publisher. Copyright 2004, All Rights Reserved Medtext Medical World, Inc.

114

37. E Atenolol Critical aortic stenosis exists when aortic orifice is reduced to 0.5 – 0.7 cm2 (normal is 2.5 –

3.5 cm2). Because of the long latency period most patients with aortic stenosis develop

diastolic dysfunction as a result of an increase in ventricular mass and fibrosis. This leads to

increased myocardial oxygen demand and a decrease in myocardial oxygen supply as a

result of compression of intramyocardial coronary vessels. Maintenance of normal sinus

rhythm, heart rate, and intravascular volume is critical in patients with aortic stenosis.

Tachycardia potentiates myocardial ischemia because of impaired coronary perfusion due

to decreased filling, decreased diastolic pressure and decreased perfusion. This usually

manifest as chest pain. Therefore, the use of a beta blocker (atenolol) to reduce the heart

rate, oxygen demand and to increase filling with increased diastolic pressure will improve

the coronary perfusion and is the most appropriate action.

1. Morgan GE, Mikhail MS, Murray MJ, (eds.). Clinical Anesthesiology. 3rd edition. Lange Medical Books/McGraw-Hill Medical Publishing Division. New York NY. 2002:416. 38. C A patient with an acute episode of pulmonary edema following an abdominal surgery In the case of a patient with an acute episode of pulmonary edema, increasing

transpulmonary distending pressure, with continuous positive-pressure therapy can

increase lung volume, improve compliance, reverse ventilation/perfusion mismatching and

redistribute extravascular lung water from interstitial space toward peribronchial and

perihilar region. This improves arterial oxygenation. The treatment of hypoxemia in

patients with chronic obstructive pulmonary disease (COPD) is supportive with

supplemental oxygen. In patients with pneumothorax continuous positive pressure

ventilation in the absence of a chest tube can make a partial pneumothorax worst. CPAP

will not significantly improve arterial oxygen tension in alveolar proteinosis because of

increased diffusion impairment.

1. Morgan GE, Mikhail MS, Murray MJ, (eds.). Clinical Anesthesiology. 3rd edition. Lange Medical Books/McGraw Hill Publishing Division. New York, NY. 2000:968.

Page 115: 2004 Anesth Practice Ques eBook

Anesthesia Practice Questions: Book 1 – 2004

This material may not be duplicated without written permission from the publisher. Copyright 2004, All Rights Reserved Medtext Medical World, Inc.

115

39. B An orotracheal intubation in the operating room with volatile anesthetics in the presence of an ENT This patient has an acute epiglottitis. The most appropriate action is an orotracheal

intubation in the operating room after a slow induction with a volatile anesthetic such as

sevoflurane or halothane in the presence of an ENT physician for possible tracheostomy.

Neuromuscular blocking drugs are not administered because skeletal muscle paralysis could

result in total airway obstruction. Epiglottitis resolves in 48 to 96 hours and tracheal

extubation is performed in the operating room. Nasotracheal intubation and fiber optic

intubation is not recommended because sudden and total airway obstruction can occur.

Acute epiglottitis is a bacterial infection caused by Haemophilus influenza and treated with

ampicillin.

1. Stoelting RK, Miller RD. Basics of Anesthesia. 4th edition. Churchill Livingstone. New York NY. 2000:372. 40. A Place an oral airway until surgical correction can be accomplished This is a congenital form of choanal atresia. Nasal obstruction should be suspected in any

neonate who has good breathing efforts but in whom air entry is absent. Inability to pass a

small catheter through each naris confirms the diagnosis of unilateral or bilateral

anatomical or functional obstruction. An oral airway is placed until surgical correction can

be accomplished for anatomical obstruction or nasal suctioning is carried out for functional

obstruction. Heroin use by mother can cause congestion of the nasal mucosa and

obstruction which can be treated with phenylephrine nose drops. Orotracheal intubation or

a catheter through the mouth will not be appropriate.

1. Stoelting RK, Dierdorf SF. Anesthesia and coexisting disease. 4th edition. Churchill Livingstone. New York, NY. 2002:683-4.

Page 116: 2004 Anesth Practice Ques eBook

Anesthesia Practice Questions: Book 1 – 2004

This material may not be duplicated without written permission from the publisher. Copyright 2004, All Rights Reserved Medtext Medical World, Inc.

116

41. A 500 cc of Lactate Ringer The most appropriate course of action is intravenous administration of 500 ml of Lactate

Ringers solution. Hypertrophic obstructive cardiomyopathy leads to hypertrophy of left

ventricle and intraventricular septum. Marked left ventricular hypertrophy and septal

hypertrophy makes these patients vulnerable to myocardial ischemia and outflow

obstruction. Thus any drug or event that increases preload or afterload or decreases

myocardial contractility decreases outflow obstruction. Administration of metoprolol and

phenylephrine will be the next most appropriate action.

1. Stoelting RK, Dierdorf SF. Anesthesia and coexisting disease. 4th edition. Churchill Livingstone. New York, NY. 2002:125. 42. A Prevent laryngospasm during awake nasal intubation Superior laryngeal nerve, a branch of the vagus nerve branches into internal laryngeal and

external laryngeal nerves. The internal laryngeal nerve provides sensory innervation to the

vocal cord, epiglottis and arytenoids. It is located 1 cm below each greater cornu where it

penetrates the thyrohyoid membrane. This nerve is blocked by injection of lidocaine

through the thyrohyoid membrane below greater cornu. The external laryngeal nerve

provides motor innervation to the cricothyroid muscle which lengthens, tenses and adducts

vocal folds. Recurrent laryngeal nerve or inferior laryngeal nerve provides sensory

innervation below the vocal cords and larynx and motor innervation to all the muscles of

larynx except cricothyroid muscle. The glossopharyngeal (ninth cranial nerve) nerve

provides sensation to the posterior third of tongue and oropharynx. It is blocked by

bilateral injection of 2 ml of local anesthetics into the base of palatoglossal arch.

1. Morgan GE, Mikhail MS, Murray MJ, (eds.). Clinical Anesthesiology. 3rd edition, Lange Medical Books/McGraw-Hill, New York, NY. 2002:83.

Page 117: 2004 Anesth Practice Ques eBook

Anesthesia Practice Questions: Book 1 – 2004

This material may not be duplicated without written permission from the publisher. Copyright 2004, All Rights Reserved Medtext Medical World, Inc.

117

43. C Manage the child conservatively with oxygen, mist therapy and treat with nebulized racemic epinephrine (0.5 ml of a 2.25% solution in 2.5 ml of normal saline) Post-intubation croup is caused by glottic, laryngeal or tracheal edema. This can be

avoided if the smallest acceptable tracheal tube is utilized. Subglottic edema manifests as

a “barky cough” after tracheal extubation. Administration of humidified oxygen and

racemic epinephrine administered by a nebulizer is indicated. Intravenous administration

of dexamethasone is controversial.

1. Morgan GE, Mikhail MS, Murray MJ, (eds.). Clinical Anesthesiology. 3rd edition. Lange Medical Books/McGraw-Hill Publishing Division. New York, NY. 2002:79. 44. B Administer lidocaine 100 mg via the endotracheal tube Intraoperative wheezing most commonly results from reactive bronchospasm. The

differential diagnosis of intraoperative wheezing includes: mechanical obstruction of the

tube due to bleeding, secretions, kinking, pulmonary edema, pulmonary embolus,

aspiration of gastric contents and pneumothorax. Histamine and other inflammatory

mediators release associated with administration of numerous drugs, blood products,

contrast media, allergic or anaphylactic reactions can all produce wheezing.

Recognition of the cause of wheezing and treatment is crucial as wheezing may easily

progress to bronchospasm. Intraoperative treatment of wheezing include: increase in the

inspired concentration of oxygen, providing adequate depth of anesthesia, decompression

of abdomen if necessary, and checking the placement and patency of endotracheal tube.

This is supplemented with suctioning of endotracheal tube, use of anticholinergics drugs,

lidocaine, steroids and inhalation therapy with beta agonists.

1. Longnecker DE jr, Tinker JH, Morgan GE, (eds.). Principles and Practice of Anesthesiology. 2nd edition. Mosby. New York, NY. 1998:110.

Page 118: 2004 Anesth Practice Ques eBook

Anesthesia Practice Questions: Book 1 – 2004

This material may not be duplicated without written permission from the publisher. Copyright 2004, All Rights Reserved Medtext Medical World, Inc.

118

45. D Functional residual capacity Obesity is defined as body weight greater than 20% of ideal body weight or body mass

index of 28-35. Morbid obesity is defined as body mass index of greater than 35 or body

weight more than two times ideal body weight or greater than 100 lb. over ideal body

weight. Functional residual capacity (FRC) is decreased the most in morbidly obese

patients. Expiratory reserve volume is greatly decreased while lung residual volume

remains in the normal range. Forced expiratory flow rate and maximal mid expiratory flow

rate remains normal.

1. Duke J (ed.). Anesthesia Secrets. 2nd edition. Hanley and Belfus, Inc. Philadelphia, PA. 2000:284. 46. B Reoperation to relieve abdominal pressure Omphalocele is external herniation of abdominal viscera through the base of umbilical cord.

The incidence is 1 in 5,000 to 10,000 live births. Seventy five (75%) percent of the time it

is associated with Down’s syndrome, cardiac anomalies or Beckwith syndrome

(omphalocele, organomegaly, macroglossia and hypoglycemia). This patient had adequate

fluid resuscitation during surgery as evident from his urinary output. Following abdominal

wall closure, high intraoperative pressure interfered with abdominal organ perfusion

because the cavity is too small for viscera. In this particular case increased abdominal

pressure with decreased venous return from the lower extremities and decreased renal

perfusion lead to oliguria. A fluid challenge with lactate Ringers solution is the next most

plausible choice.

1. Barash PG, Cullen BF and Stoelting RK, (eds.). Clinical Anesthesia. 4th edition. Lippincott Williams & Wilkins. New York, NY. 2001:1185.

Page 119: 2004 Anesth Practice Ques eBook

Anesthesia Practice Questions: Book 1 – 2004

This material may not be duplicated without written permission from the publisher. Copyright 2004, All Rights Reserved Medtext Medical World, Inc.

119

47. A Postural hypotension, nasal stuffiness, decreased sweating and stable vital signs Once pheochromocytoma is diagnosed, outpatient therapy is initiated to establish alpha-

adrenergic receptor blockade (usually with phenoxybenzamine or less often with prazosin).

Stable vital signs, postural hypotension, nasal stuffiness are clues to adequate block.

Alpha-methyltyrosine inhibits tyrosine hydroxylase, which is a rate-limiting enzyme in

catecholamine biosynthesis. This medication is reserved for patients with metastatic

disease or in situations where surgery is contraindicated. The overall goal of medical

treatment before surgery is to minimized sympathetic outflow that commonly occurs with

induction and surgical stimulation

1. Morgan GE, Mikhail MS, Murray MJ, (eds.). clinical Anesthesiology. 3rd edition. Lange Medical Books. McGraw-Hill Medical Publishing Division. New York, NY. 2002:220-222. 48. A Enflurane-specific vaporizer is filled with halothane Vaporizers are agent-specific. An unintentional filling with a wrong anesthetic agent may

lead to overdose or underdose of an anesthetic agent. For example halothane with higher

vapor pressure (240 vs 175 of enflurane) will cause a 40% greater amount of anesthetic

vapor delivered to the patient. Besides, halothane is more potent than enflurane (lower

MAC). As a result filling of enflurane vaporizer with halothane could lead to delivery of

toxic dose of halothane. The converse is true if halothane vaporizer is filled with enflurane.

Same reasoning will apply if isoflurane vaporizer is filled with halothane.

1. Morgan GE, Mikhail MS, Murray MJ, (eds.). Clinical Anesthesiology. 3rd edition. Lange Medical Books. McGraw-Hill Medical Publishing Division. New York, NY. 2002:49. 49. A Increase FiO2 to 40% Opioids particularly meperidine is used most frequently for obstetrics pain relief. Neonatal

respiratory depression depends upon the total dose and the time interval between

administration and delivery. Maximal respiratory depression occurs with delivery between

1-3 hours after IM injection. However, less depression is seen if meperidine is given less

Page 120: 2004 Anesth Practice Ques eBook

Anesthesia Practice Questions: Book 1 – 2004

This material may not be duplicated without written permission from the publisher. Copyright 2004, All Rights Reserved Medtext Medical World, Inc.

120

than one hour and more than four hours after delivery. The respiratory rate of this

newborn is in the normal range (40 to 60) with slightly low pulse rate (normal 120-160),

low PaO2 (normal 70) and slightly high PaCO2 (normal 40). The most appropriate action

would be to increase FiO2 to 40% and observe the patient.

1. Longnecker DE jr, Tinker JH, Morgan GE, (eds.). Principles and Practice of Anesthesiology. 2nd edition Mosby. New York, NY. 1998:1995. 50. D Nitrous oxide should be avoided because it will increase the intrapleural pressure and the size of pneumothorax Pneumothorax is the accumulation of gas within the pleural space. The etiology of

pneumothorax include: rupture of blebs and bullae, COPD, chest trauma, surgeries

involving thoracic cage and upper abdominal wall, high volume or high pressure positive

pressure ventilation. In this question choice D, is the most appropriate because the use of

nitrous oxide during the surgery will cause preferential transfer of nitrous oxide into the air-

filled cavity (the blood:gas partition of nitrous oxide is 34 times greater than that of

nitrogen), thus making the pneumothorax worst. Chest tube should not be clamped but put

to water seal to ensure removal of air from the pleural space and continued expansion of

lung. High tidal volume will expand the pneumothorax. Signs of pneumothorax include:

wheezing, decreased breath sounds, increased peak inspiratory pressures with hypoxemia,

decreased pulmonary compliance and hypotension.

1. Stoelting RK, Miller RD. Basics of Anesthesia. 4th edition. Churchill Livingstone. New York, NY. 2000:280. 51. D Desflurane Desiccated (dehydrated) soda lime and Baralyme can degrade inhaled anesthetics to form

clinically significant concentrations of carbon monoxide (CO), which can result in

carboxyhemoglobinemia. Desflurane is capable of producing the greatest concentration of

carbon monoxide when exposed to desiccated absorbent. Highest levels of CO are formed

after prolonged contact between absorbent and inhaled anesthetics and after absorbent

disuse for at least 48 hours. Reports of CO poisoning are most common in patients

Page 121: 2004 Anesth Practice Ques eBook

Anesthesia Practice Questions: Book 1 – 2004

This material may not be duplicated without written permission from the publisher. Copyright 2004, All Rights Reserved Medtext Medical World, Inc.

121

anesthetized on Monday morning, because continuous high flows (10 L/min) from the

anesthesia machine over the weekend results in dehydration of the absorbent.

Factors that appear to increase the production of CO include:

a) The inhaled anesthetic used; desflurane produces the greatest concentration of

carbon monoxide. In order of CO production from greatest to least:

Desflurane ≥ Enflurane ≥ Isoflurane >> Halothane = Sevoflurane

b) The dryness of the absorbent - dehydrated absorbent produces more CO - when

absorbent moisture content is normal, no measurable levels of CO are produced

with desflurane, enflurane or isoflurane

c) The type of absorbent - Baralyme produces more CO than soda lime

d) The temperature - increased temperature increases CO production

e) The anesthetic concentration - high concentrations of anesthetics produces more

CO

Xenon is a noble gas that has anesthetic properties. Xenon has a MAC of 71%, making it

more potent than nitrous oxide (MAC 104%). Xenon is nonexplosive, non-pungent, and

odorless. Xenon in not known to interact with desiccated absorbent.

1. Miller R. Anesthesia. 5th edition. Churchill Livingstone. New York NY. 2000:194-5. 52. C CBF increases, CMRO2 increases, ICP increases Nitrous oxide (N2O) is an inhalational anesthetic whose effects appear to differ from other

inhalational agents. All volatile anesthetics tend to decrease cerebral metabolic rate of

oxygen (CMRO2) while increasing cerebral blood flow (CBF) and intracranial pressure

(ICP). This phenomenon is called uncoupling. Nitrous oxide on the other hand has a

different effect. When given alone it tends to cause an increase in CBF and ICP and

although somewhat controversial, the consensus is that it also increases CMRO2. When

given with a more potent volatile anesthetic, the increase in CMRO2 is not seen but a more

profound increase in CBF and ICP are seen than what would be expected with the volatile

anesthetic alone reflecting the greater vasodilating effect of N2O on cerebral circulation.

Page 122: 2004 Anesth Practice Ques eBook

Anesthesia Practice Questions: Book 1 – 2004

This material may not be duplicated without written permission from the publisher. Copyright 2004, All Rights Reserved Medtext Medical World, Inc.

122

Interesting, when N2O is given with IV anesthetics, the effects on CMRO2 or CBF are

decreased or eliminated.

The following table summarizes these effects:

ANESTHETIC

CBF

CMRO2

ICP

Halothane

↑ ↓ ↑

Isoflurane

↑ ↓ ↑

Desflurane

↑ ↓ ↑

Sevoflurane

↑ ↓ ↑

N2O alone

↑ ↑ ↑

N2O with IV Anesthetics

O O O

N2O with Volatile Anesthetics

↑ O ↑

1. Miller R. Anesthesia. 5th edition. Churchill Livingstone. New York NY. 2000:710-711.

Page 123: 2004 Anesth Practice Ques eBook

Anesthesia Practice Questions: Book 1 – 2004

This material may not be duplicated without written permission from the publisher. Copyright 2004, All Rights Reserved Medtext Medical World, Inc.

123

53. D Normeperidine Meperidine is metabolized through two hepatic routes:

1. Hydrolysis to meperidinic acid

2. N-demethylation to normeperidine which then undergoes hydrolysis to

normeperidinic acid

Excretion of the end products normeperidinic acid and normeperidine is primarily renal.

The acid metabolites, meperidinic acid and normeperidinic acid are inert and do not exert

any pharmacological effects. Normeperidine however, is pharmacologically active (half

the analgesic property of meperidine) and with CNS toxicity. Normeperidine has a longer

half-life (15-20 hours) than meperidine itself (3-6 hours). Therefore, prolonged

administration (such as patient controlled analgesia) may lead to toxicity due to

accumulation of normeperidine especially when renal impairment is present.

Normeperidine can produce signs of central nervous system excitation. These signs include

tremors, myoclonus, and grand mal seizures. Although meperidine levels may be within

standard analgesic range, normeperidine levels may sometimes be at toxic levels due to its

longer half-life. In situations where prolonged administration of an opioid is anticipated, a

drug without an active metabolite should be considered. Naloxone does not reverse

normeperidine toxicity.

Laudanosine is a active metabolite of atracurium, a non-depolarizing muscle relaxant. It too

can produce seizures with toxic levels. Norcodeine is a metabolite of codeine and not

meperidine, and is not implicated in any CNS toxicity.

1. Stoelting, RK. Pharmacology and Physiology in Anesthetic Practice. 3rd edition. Lippincott-Raven. Philadelphia.1999:91-93.

Page 124: 2004 Anesth Practice Ques eBook

Anesthesia Practice Questions: Book 1 – 2004

This material may not be duplicated without written permission from the publisher. Copyright 2004, All Rights Reserved Medtext Medical World, Inc.

124

54. D Superior laryngeal nerve The superior laryngeal nerve (a branch from the vagus nerve - Cranial nerve X)

provides sensation from the epiglottis to the vocal cord (i.e., sensation above the cords). A

superior laryngeal block will therefore provide anesthesia to this area. The block is

performed with the patient supine and the greater cornu of the hyoid bone located. The

needle is walked off the greater cornu and into the thyrohyoid membrane where 2 to 3 ml

of local anesthetic is injected. The block is repeated on the opposite side.

The recurrent laryngeal nerve provides sensation to the tracheal below the cords, while

the sphenopalatine nerve provides sensation to the mucosa of the nares. Sensation of

the posterior oral pharynx is by the lingual nerve (a branch of the mandibular division of

Cranial nerve V) and the posterior 2/3rd of the tongue is by the glossopharyngeal nerve

(Cranial nerve IX).

1. Miller R. Anesthesia. 5th edition. Churchill Livingstone. New York NY. 2000:1541-1543. 55. C A-delta Normal body core temperature is maintained within ± 0.2oC (the inter threshold range) of

its target temperature of 37oC. To accomplish this, normal thermoregulation requires three

components:

1. Afferent input from peripheral sensing sites

2. Central regulation or control by the hypothalamus

3. Efferent responses in the form of behavior or autonomic responses such as

sweating and vasodilation for temperature increases and vasoconstriction and

shivering for decreases in temperature.

Afferent cold input to the hypothalamus is via A-delta nerve fibers while warm

temperature information travel by unmyelinated C fibers. The hypothalamus regulates

temperature by comparing this afferent temperature information with threshold

Page 125: 2004 Anesth Practice Ques eBook

Anesthesia Practice Questions: Book 1 – 2004

This material may not be duplicated without written permission from the publisher. Copyright 2004, All Rights Reserved Medtext Medical World, Inc.

125

temperature for heat and cold. When the input temperature exceeds a threshold, the

appropriate response is initiated to maintain adequate body temperature.

1. Miller R. Anesthesia. 5th edition. Churchill Livingstone. New York NY. 2000:1368. 56. E Expiratory reserve volume (ERV) + residual volume (RV) The volume of gas that remains in the lung after a normal expiration is defined as the

functional residual capacity (FRC). It comprises of expiratory reserve volume (ERV) and

residual volume (RV). A reduction in FRC increases shunting and causes arterial hypoxemia.

Other capacities of interest include:

a. Inspiratory capacity (IC) composed of inspiratory reserve volume + tidal volume

b. Total lung capacity (TLC) composed of IC + FRC

c. Vital capacity (VC) composed of inspiratory reserve volume + expiratory reserve

volume + tidal volume 1. Miller R. Anesthesia. 5th edition. Churchill Livingstone. New York NY. 2000:590. 57. A Hypercarbia The oculocardiac reflex is a reflex arc that involves the trigeminal nerve (the afferent limb)

and the vagus nerve (efferent limb). This reflex can be induced by pressure on the eye

globe, traction of extraocular muscles, trauma to the eye, and even after a retrobulbar

block administration. The most common manifestation of the oculocardiac reflex is sinus

bradycardia although a wide array of other cardiac dysrhythmias can also be elicited.

Conditions that can potentiate this reflex are hypercarbia, hypoxemia, light anesthesia and

conditions that result in increase vagal tone. Hypertension and hypotension are not

associated with an increased incidence of this reflex. 1. Miller R. Anesthesia. 5th edition. Churchill Livingstone. New York NY. 2000:2180. 2. Barash P, Cullen B, Stoelting R. Clinical Anesthesia, 4th edition, J.B. Lippincott. Philadelphia. 2001:973-974.

Page 126: 2004 Anesth Practice Ques eBook

Anesthesia Practice Questions: Book 1 – 2004

This material may not be duplicated without written permission from the publisher. Copyright 2004, All Rights Reserved Medtext Medical World, Inc.

126

58. D Administer fresh frozen plasma Administration of fresh frozen plasma (FFP) is best for urgently reversing the anticoagulant

effects of warfarin (Coumadin). Recommended dose for reversal of warfarin is 5 to 8 ml/Kg

of FFP.

Administration of vitamin K, subcutaneous or intravenous, can also be use for reversing

warfarin effects, but improvement in coagulation may not be seen for hours and full

restoration of coagulability may not be seen for at least 24 hours regardless of route of

administration.

Protamine is used to reverse the effects of heparin and has no effect on the anticoagulant

warfarin. Cryoprecipitate is used for correction of factor VIII deficiency and von

Willebrand’s factor deficiency. Albumin contains no coagulation factors and is therefore

ineffective therapy for reversing warfarin anticoagulant effects.

1. Miller R. Anesthesia. 5th edition. Churchill Livingstone. New York NY. 2000:1613. 59. E Measuring pulmonary artery pressures Neurosurgical procedures involving the posterior fossa can often be complicated with the

occurrence of venous air emboli. The incidence is somewhere between 30 and 50% in

patients operated in the sitting position. Air entering the venous circulation eventually

makes its way to the pulmonary arterioles and results in intense vasoconstriction. This

results in ventilation/perfusion mismatch, interstitial pulmonary edema, and reduced

cardiac output as the pulmonary vascular resistance increases. Large volumes of air can

prevent cardiac ejection by causing an “air lock” effect.

Monitors used intraoperatively for the detection of venous air embolus include (from most

sensitive to least sensitive):

Page 127: 2004 Anesth Practice Ques eBook

Anesthesia Practice Questions: Book 1 – 2004

This material may not be duplicated without written permission from the publisher. Copyright 2004, All Rights Reserved Medtext Medical World, Inc.

127

1. Transesophageal echocardiography (TEE)

2. Precordial Doppler

3. Pulmonary artery catheter (Detection of increases in pulmonary artery pressure)

4. Capnography (Detection of decreases in end-tidal CO2)

5. Monitoring end-tidal N2 (Detection of increases in end-tidal N2)

The precordial Doppler ultrasound transducer is the most sensitive noninvasive monitor

and the earliest detector of venous air embolism while the TEE is the most sensitive

invasive monitor but is cumbersome and expensive to use. The detection of pulmonary

hypertension (increase in pulmonary artery pressures) is slightly more sensitive than

capnography. Monitoring for air (end-tidal N2) may not detect subclinical air emboli.

1. Cottrell, JE, Smith, DS. Anesthesia and Neurosurgery. 4th edition. Mosby. St. Louis. 2001:340-345. 60. E Tracheal intubation Anatomical dead space is ventilation of the nasal and oropharynx to the terminal and

respiratory bronchioles, where little gas exchange occurs. Anatomical dead space

ventilation is approximately equal to 2 ml / kg ideal body weight (or approximately 150 ml).

Anatomical dead space accounts for the majority of physiologic dead space (physiologic

dead space is anatomical dead space plus alveolar dead space).

Maneuvers that increase dead space include positioning (supine to upright position), neck

extension, and use of bronchodilators. Ventilation by mask can add approximately 100 ml.

to the anatomical dead space from the mask itself. Tracheal intubation on the other hand,

reduces anatomical dead space because ventilation now bypasses the nasal and

oropharynx area.

1. Lumb, AB. Nunn’s Applied Respiratory Physiology. 5th edition. Butterworth-Heinemann. 2000:178-179.

Page 128: 2004 Anesth Practice Ques eBook

Anesthesia Practice Questions: Book 1 – 2004

This material may not be duplicated without written permission from the publisher. Copyright 2004, All Rights Reserved Medtext Medical World, Inc.

128

61. E Higher systolic pressure and lower diastolic pressure The arterial pressure waveform is the results of blood ejected from the left ventricle into

the aorta. The flow of blood is not only from the ejection of blood from the ventricle but

propagation of the waveform by reflectance due to the distensibility and contraction of the

arterial vessels. As this arterial pressure waveform travels through peripheral arterioles a

phenomenon of distal pulse amplification occurs. Compared to the aorta, peripheral

arterial waveforms have a higher systolic pressure, a lower diastolic pressure and a wider

pulse pressure. The mean arterial pressure however remains greater in the aorta compared

to the peripheral arterial vessels.

1. Miller R. Anesthesia. 5th edition. Churchill Livingstone. New York NY. 2000:1137-1142. 2. Barash P, Cullen B, Stoelting R. Clinical Anesthesia, 4th edition, J.B. Lippincott. Philadelphia. 2001:875.

62. C Etomidate Etomidate used for induction of anesthesia causes suppression of adrenal function by

producing a dose dependent inhibition of the conversion of 11-deoxycortisol to cortisol. The

specific enzyme inhibited by etomidate appears to be 11-beta-hydroxylase. This enzyme

inhibition lasts 4 to 8 hours after an induction dose of etomidate (with documented adrenal

compromise up to 24 hours). Studies however, suggest that etomidate is safe and that this

adrenal suppression is probably clinically insignificant.

None of the other intravenous induction medications are known to cause adrenal

suppression.

1. Miller R. Anesthesia. 5th edition. Churchill Livingstone. New York NY. 2000:247-248. 2. Barash P, Cullen B, Stoelting R. Clinical Anesthesia, 4th edition, J.B. Lippincott. Philadelphia. 2001:148.

Page 129: 2004 Anesth Practice Ques eBook

Anesthesia Practice Questions: Book 1 – 2004

This material may not be duplicated without written permission from the publisher. Copyright 2004, All Rights Reserved Medtext Medical World, Inc.

129

63. D Atypical pseudocholinesterase Mivacurium is a short-acting nondepolarizing muscle relaxant with a duration of 10 to 20

minutes. Unique to mivacurium is its clearance. Mivacurium is hydrolyzed by plasma

cholinesterase (pseudocholinesterase) an enzyme that also is responsible for the

breakdown of succinylcholine. Presence of atypical pseudocholinesterase variants will result

in prolonged paralysis by succinylcholine and mivacurium.

Mivacurium is not depended on renal clearance or non-specific esterase metabolism.

Although hypothermia is known to prolong paralysis of nondepolarizing muscle relaxants

and inhaled anesthetics enhance their muscle relaxant effects, neither would be expected

to prolong mivacurium effects to the extent described.

1. Stoelting, RK. Pharmacology and Physiology in Anesthetic Practice. 3rd edition. Lippincott-Raven. Philadelphia. 1999:191-192. 2. Barash P, Cullen B, Stoelting R. Clinical Anesthesia, 4th edition, J.B. Lippincott. Philadelphia. 2001:538-540. 64. D Eu Eu Inherited variants of pseudocholinesterase can prolong the duration of action of

succinylcholine and mivacurium. Prolonged paralysis can occur with a recommended

intubation dose. These patients usually have no symptoms.

Normal pseudocholinesterase activity is inhibited by 80% by dibucaine, an amide local

anesthetic, while atypical pseudocholinesterase (homozygous) is inhibited by only 20%.

This dibucaine number (level of inhibition) can therefore be used to characterize the

different pseudocholinesterase variants. It is important to recognize that the dibucaine

number reflects the quality of plasma cholinesterase enzyme activity (i.e., ability to

hydrolyzed succinylcholine) and not the quantity of enzyme that is circulating in the

plasma.

Page 130: 2004 Anesth Practice Ques eBook

Anesthesia Practice Questions: Book 1 – 2004

This material may not be duplicated without written permission from the publisher. Copyright 2004, All Rights Reserved Medtext Medical World, Inc.

130

VARIANTS OF PLASMA CHOLINESTERASE ENZYME

GENOTYPE DIBUCAINE NUMBER (% inhibited)

DURATION (succinylcholine)

INCIDENCE

Homozygous (Eu Eu) 80 5 - 10 min normal

Heterozygous (Eu Ea) 40 - 60 20 min 1 in 480

Homozygous atypical (Ea Ea) 20 60 - 180 min 1 in 3200

Genotype Eu Eu represents the normal homozygous type of pseudocholinesterase.

Over the years two silent variants (Es) as well as a plasma cholinesterase inhibited by

fluoride (Ef) have been discovered. (A fluoride number like a dibucaine number can be

generated.)

Significant prolongation of succinylcholine and mivacurium occurs with the following

genotypes: EaEa, EfEf, EaEs, EfEa, and EsEs.

1. Stoelting, RK. Pharmacology and Physiology in Anesthetic Practice. 3rd edition. Lippincott-Raven. Philadelphia. 1999:191-192. 2. Barash P, Cullen B, Stoelting R. Clinical Anesthesia, 4th edition, J.B. Lippincott. Philadelphia. 2001:538-540. 65. E Pancuronium The major concern in using muscle relaxants in patient with renal dysfunction is that

duration of action may be prolonged; especially those depended on renal clearance.

Pancuronium is a long-acting nondepolarizing muscle relaxant that is depended on renal

clearance for termination of its effects. Approximately 80% of a single dose of pancuronium

is excreted by the kidneys unchanged. Its use in a patient with end stage renal disease is

unwise.

Page 131: 2004 Anesth Practice Ques eBook

Anesthesia Practice Questions: Book 1 – 2004

This material may not be duplicated without written permission from the publisher. Copyright 2004, All Rights Reserved Medtext Medical World, Inc.

131

Atracurium and cis-atracurium duration of action is not prolonged in renal disease. The

reason is that metabolism of these nondepolarizing muscle relaxants is not depended on

normal renal function. Instead, atracurium is depended on Hoffman elimination and ester

hydrolysis while cis-atracurium is mostly entirely depended on Hoffman elimination.

Rocuronium is primarily depended on the liver for its elimination and therefore its duration

of action is not prolonged in patients with end stage renal disease.

Succinylcholine is not depended on renal clearance. Although pseudocholinesterase is

known to be decrease in renal failure, prolonged paralysis is not clinically significant when

using succinylcholine. Concerns of exaggerated hyperkalemia in patients with renal

diseases and the use of succinylcholine has not be supported by recent controlled studies,

and therefore has been shown to be safe to use in patients with chronic renal failure.

1. Miller R. Anesthesia. 5th edition. Churchill Livingstone. New York NY. 2000:423. 2. Barash P, Cullen B, Stoelting R. Clinical Anesthesia, 4th edition, J.B. Lippincott. Philadelphia. 2001:1013-1014. 66. D Prominent U waves Gastrointestinal losses of potassium are common in patients undergoing “bowel prep” for

intestinal surgical procedures. Electrocardiogram (ECG) changes consistent with

hypokalemia include flatten T waves, prominent U waves and ST segment depression.

Although these are classic ECG description, they may not be as sensitive as indicators for

hypokalemia.

Peaked T waves, widen QRS complexes with loss of P waves are more commonly seen with

hyperkalemia. Delta waves are consistent with Wolf-Parkinson-White (WPW) syndrome.

1. Barash P, Cullen B, Stoelting R. Clinical Anesthesia, 4th edition, J.B. Lippincott.-Philadelphia. 2001:186-189.

Page 132: 2004 Anesth Practice Ques eBook

Anesthesia Practice Questions: Book 1 – 2004

This material may not be duplicated without written permission from the publisher. Copyright 2004, All Rights Reserved Medtext Medical World, Inc.

132

67. A Less than 24 hours post-injury Hyperkalemia may result from the use of succinylcholine especially in those patients with

motor deficits due to spinal cord injuries. The severity of hyperkalemia depends on the

extent of paralysis. It is probably safe to use succinylcholine within the first 48 hours. After

this time, hyperkalemia severe enough to cause ventricular fibrillation and cardiac arrest

may occur. This is due to increase sensitivity of acetylcholine receptors to succinylcholine

and upregulation of these same receptors. Peak hyperkalemia levels occur between 4

weeks and 5 months.

1. Barash P, Cullen B, Stoelting R. Clinical Anesthesia, 4th edition, J.B. Lippincott. Philadelphia. 2001:1109. 68. E Ketamine Gamma aminobutyric acid (GABA) is primarily an inhibitory neurotransmitter. It interacts

with GABAA receptors, causing its chloride channel to open, hyperpolarizing the nerve and

reducing its excitability. Sodium thiopental, methohexital, propofol, etomidate and

benzodiazepines all seem to exert there effects by enhancing the action of GABA at these

receptors. Ketamine exerts its effects via the N-methyl-D-aspartate (NMDA) receptors.

1. Barash P, Cullen B, Stoelting R. Clinical Anesthesia, 4th edition, J.B. Lippincott. Philadelphia. 2001:127-129. 69. D Administer oxytocin Uterine atony is most common cause of postpartum hemorrhage. It often occurs

immediately after delivery. Uterine message and the infusion of oxytocin are initial

treatments. Oxytocin is a hormone secreted by the posterior pituitary. Synthetic

preparations of oxytocin are often use postpartum for its ability to contract uterine smooth

muscle and control postpartum bleeding. Oxytocin is often given by infusion.

Desmopressin (DDAVP) is a synthetic analogue of antidiuretic hormone (ADH) often used in

increasing von Willebrand factor. Vasopressin (ADH) is indicated for the treatment of

diabetes insipidus. Phenylephrine is a direct alpha agonist that would not have any effect

Page 133: 2004 Anesth Practice Ques eBook

Anesthesia Practice Questions: Book 1 – 2004

This material may not be duplicated without written permission from the publisher. Copyright 2004, All Rights Reserved Medtext Medical World, Inc.

133

on uterine smooth muscle. Neostigmine is an anticholinesterase that is use to reverse the

effects of nondepolarizing muscle relaxants. 1. Stoelting, RK. Pharmacology and Physiology in Anesthetic Practice. 3rd edition. Lippincott-Raven. Philadelphia. 1999:422-424. 70. A Hypotension Uterine atony is most common cause of postpartum hemorrhage. It often occurs

immediately after delivery. Uterine message and the infusion of oxytocin are initial

treatments. Oxytocin is a hormone secreted by the posterior pituitary. Synthetic

preparations of oxytocin are often use postpartum for its ability to contract uterine smooth

muscle and control postpartum bleeding. Oxytocin is often given by infusion.

Given intravenously, oxytocin causes peripheral vasodilation of vascular smooth muscle.

The result is hypotension, with a reflex tachycardia. Patients that are hypovolemic may

experience an exaggerated hypotensive response. None of the other symptoms listed are

likely with oxytocin administration.

1. Stoelting, RK. Pharmacology and Physiology in Anesthetic Practice. 3rd edition. Lippincott-Raven. Philadelphia. 1999:422-424. 71. E Stop ventilation The use of a laser, particularly in the area of the endotracheal tube, is at high risk of

igniting an airway fire. Everyone including the surgeon and anesthesiologist should have a

plan of action should an airway fire occur. Should a fire occur the first and immediate step

is to stop ventilation and disconnect the flow of gases (oxygen) to the airway. This should

be followed by removal of the endotracheal tube. Only after the fire has been extinguished

should ventilation by mask with 100% oxygen begin. This can be followed with reintubation

of the trachea to assess the extent of damage. Switching from oxygen to air would not

necessarily extinguish the fire, nor would stopping the use of volatile anesthetic, since in

both conditions; oxygen would still be present to sustain continued combustion. Nitrous

Page 134: 2004 Anesth Practice Ques eBook

Anesthesia Practice Questions: Book 1 – 2004

This material may not be duplicated without written permission from the publisher. Copyright 2004, All Rights Reserved Medtext Medical World, Inc.

134

oxide also supports combustion just like oxygen. To reduce the risk of fire during laser

surgery, not more than 30% oxygen should be used.

1. Miller R. Anesthesia. 5th edition. Churchill Livingstone. New York NY. 2000:2210. 72. D Cis-atracurium Many medication used in the practice of anesthesia are know to cause a release in

histamine. Therefore, theoretically could induce a bronchospasm especially in those

patients with known reactive airways. Morphine and meperidine are opioids that induce

histamine release. This reaction is not seen with the synthetic opioids such as fentanyl and

alfentanil. Sodium thiopental also is known to cause histamine release, but despite this has

been used safely even in asthmatics.

Many nondepolarizing agents have been know to cause histamine release including

mivacurium, atracurium and d-tubocurarine. Cis-atracurium is an isomer of atracurium, but

unlike atracurium, does not cause the release of histamine.

1. Miller R. Anesthesia. 5th edition. Churchill Livingstone. New York NY. 2000:451-452. 73. E Pulmonary edema This is a classic presentation of negative pressure pulmonary edema. Negative pressure

pulmonary edema is most often a complication of an acute airway obstruction. The edema

occurs because mark negative intrapleural pressure is generated against a closed or

obstructed glottis driving fluid from the capillary to interstitial tissue. Negative pressure

pulmonary edema most commonly occurs after an episode of laryngospasm and requires

the continued ventilatory effort by the patient. Young athletic male are at increase risk for

developing pulmonary edema since they can generate mark negative intrapleural pressure.

Symptoms such as dyspnea, cyanosis and pink, frothy secretions can appear rapidly. Most

cases require only supportive care and resolve within 12 to 24 hours.

Page 135: 2004 Anesth Practice Ques eBook

Anesthesia Practice Questions: Book 1 – 2004

This material may not be duplicated without written permission from the publisher. Copyright 2004, All Rights Reserved Medtext Medical World, Inc.

135

Differential diagnoses include fluid overload and volume overload, both of which can

present with symptoms similar to that above. Intubation of the esophagus will present with

continuing desaturation.

1. Gravenstein N, Kirby RR. Complications in Anesthesiology. 2nd edition. Lippincott. Philadelphia. 1996:191-196. 74. B Venous air embolus Correction of spinal scoliosis provides a challenge to the anesthesiologist. This surgical

procedure involves a patent with pre-existing pulmonary and cardiac dysfunction, placing

the patient in the prone position, the possibility of significant blood loss and an increase

risk of venous air embolus (VAE). The large incisional exposure of bone and tissue increase

this risk of VAE. Sign and symptoms include sudden drop in end-tidal carbon dioxide,

hypotension, hypoxemia, and in severe cases cardiac collapse. A central venous catheter

should be placed to aspirate air should a VAE occur.

Spinal cord ischemia and malignant hyperthermia would not result in a decrease in end

tidal carbon dioxide. The amount of sevoflurane being used would not cause the significant

drop seen in the capnography.

1. Barash P, Cullen B, Stoelting R. Clinical Anesthesia, 4th edition, J.B. Lippincott. Philadelphia. 2001:1110-1111. 75. E Transient neurologic syndrome The term TNS (Transient Neurologic Syndrome) is used to describe symptoms of backache

with radiation into the buttocks or lower extremities. (An earlier used term was Transient

Radicular Irritation but that has been abandoned for TNS.) This syndrome is rarely seen

after general anesthesia and has been associated with central neuraxial anesthesia with all

local anesthetics, particularly lidocaine.

Page 136: 2004 Anesth Practice Ques eBook

Anesthesia Practice Questions: Book 1 – 2004

This material may not be duplicated without written permission from the publisher. Copyright 2004, All Rights Reserved Medtext Medical World, Inc.

136

Risk of TNS is increased with use of lidocaine, ambulatory anesthesia, lithotomy and knee

arthroscopy positions, obesity, and is unaffected by baricity, dose, type of needle, addition

of epinephrine, paresthesia, or concentration (no lower incidence with concentrations to

0.5%). TNS typically occurs 12-36 hours after resolution of spinal anesthesia and last for 2-

3 days. TNS is self limited and can be effectively treated with potent nonsteroidal anti-

inflammatory drugs.

Although femoral and obturator neuropathies are complications of lithotomy positioning,

the sensory deficit distribution does not involve the buttocks. Epidural hematoma usually

present with paralysis without sensory changes. Cauda equina syndrome is associated with

sensory and motor deficits but not with TNS.

1. Barash P, Cullen B, Stoelting R. Clinical Anesthesia, 4th edition, J.B. Lippincott. Philadelphia. 2001:709. 2. Spencer SL, et. Al. Current Issues in Spinal Anesthesia, Review Article. Anesthesiology. 2001:94:888-906. 76. B Medullary portion of ascending loop of Henle Mannitol is an osmotic diuretic often used for prophylaxis against acute renal failure and

treatment of increased intracranial pressure. It‘s primary site of action is at the medullary

portion of the ascending loop of Henle. Loop diuretics also exert their effects at this portion

of the tubules. Thiazide diuretic site of action is the cortical portion of the ascending loop of

Henle. The distal convoluted tubule is where potassium sparing diuretic (triamterene) has

their action. Aldosterone antagonists exert their effects at the collecting duct.

1. Stoelting, RK. Pharmacology and Physiology in Anesthetic Practice. 3rd edition. Lippincott-Raven. Philadelphia.1999:436,440-441.

Page 137: 2004 Anesth Practice Ques eBook

Anesthesia Practice Questions: Book 1 – 2004

This material may not be duplicated without written permission from the publisher. Copyright 2004, All Rights Reserved Medtext Medical World, Inc.

137

77. C Redistribution into skeletal muscle When an induction dose is given intravenously, it rapidly leaves the blood compartment

and enters the brain where it exerts its hypnotic effects. Uptake into the brain occurs within

30 seconds and peaks at about 5 minutes. After this initial peak in the brain, the drug

begins to redistribute into skeletal muscle. It is this redistribution of sodium thiopental from

the brain into skeletal muscle that accounts for the rapid awakening seen with its use.

1. Stoelting, RK. Pharmacology and Physiology in Anesthetic Practice. 3rd edition. Lippincott-Raven. Philadelphia.1999:436,114-115. 78. D Greater cornu of the hyoid cartilage The superior laryngeal nerve provides sensory to the area from the epiglottis to the vocal

cords (“sensation above the cord”). The nerve a branch from the vagus nerve pierces the

thyrohyoid membrane at the point of the hyoid cartilage. Bilateral blockade is necessary to

block sensation to this area.

The Chassaignac tubercle (the anterior tubercle of the transverse process of C6) is a

landmark used to perform a stellate ganglion block, while the tonsillar pillars represent the

landmarks for a glossopharyngeal block. In order to perform a translaryngeal block, the

cricothyroid membrane must be identified.

1. Miller R. Anesthesia. 5th edition. Churchill Livingstone. New York NY. 2000:1541-1543. 79. E Redistribution of core temperature The hypothalamus tightly controls the core temperature and maintains this within an inter

threshold range of 0.4o C of it set point of 37o C. When temperatures rise above this

threshold, warm autonomic responses (vasodilatation and sweating) occur while

temperatures below this inter threshold range elicit cold autonomic responses

(vasoconstriction, and shivering). Hypothermia is common during anesthesia due to

Page 138: 2004 Anesth Practice Ques eBook

Anesthesia Practice Questions: Book 1 – 2004

This material may not be duplicated without written permission from the publisher. Copyright 2004, All Rights Reserved Medtext Medical World, Inc.

138

anesthetic induced inhibition of these thermoregulatory mechanisms and the patient

exposure to a cold operating room environment.

Intraoperative hypothermia occurs in three phases. The first phase, which begins shortly

after induction and continues for 1 to 2 hours, is due to the internal redistribution of core

temperature to the periphery because of anesthetic induced peripheral vasodilatation.

During the second phase a thermal imbalance occurs because of a reduction in heat

production and loss to the environment. Radiation and convection contribute 85% of the

heat losses occurring in the operating room. Evaporation losses from the skin, respiratory

tract and open surgical wounds contribute 5 to 10% of all heat loss while conductive heat

transfer is often minimal.

The third phase is the thermal steady state and represents the equalization of heat

production with environmental heat loss. 1. Miller R. Anesthesia. 5th edition. Churchill Livingstone. New York NY. 2000:1371-1373. 80. A Complex Regional Pain Syndrome I Complex Regional Pain Syndrome I (CRPS I) is the new term replacing reflex sympathetic

dystrophy (RSD) while CRPS II replaces the term for the syndrome causalgia. CRPS I

usually follow a noxious event that does not involve nerve injury or follow a characteristic

peripheral nerve distribution. CRPS II has the same clinical signs and symptoms but the

history is significant for a nerve injury.

CRPS I presents with a triad of sensory, autonomic and motor signs and symptoms.

Sensory symptoms include pain described as burning and aching. Allodynia and

hyperalgesia is often present. Autonomic signs and symptoms almost always include

edema, skin color changes (erythema or cyanotic), and even changes in skin temperature

(higher or lower) when compared to the other limb. Motor signs and symptoms include

muscle weakness, spasm, and decrease range of motion. Other associated signs can

include trophic changes like increase or decrease nail or hair growth.

Page 139: 2004 Anesth Practice Ques eBook

Anesthesia Practice Questions: Book 1 – 2004

This material may not be duplicated without written permission from the publisher. Copyright 2004, All Rights Reserved Medtext Medical World, Inc.

139

Fibromyalgia, peripheral neuropathy and Raynaud’s Syndrome do not include many of

these characteristics, especially allodynia, hyperalgesia, or edema.

1. Srinivasa RN, et.al., Complex Regional Pain Syndrome I (Reflex Sympathetic Dystrophy), Anesthesiology, 2002:96:1254-1260. 81. E Osteoporosis Complex Regional Pain Syndrome, Type I (CRPS I) or Reflex Sympathetic Dystrophy is

characterized by constant, severe burning pain most often involving the upper limbs and

follows a noxious events without nerve damage. Three characteristic phases have often

been described and include an acute hyperemic stage (I), dystrophic stage (II) and

atrophic stage (III). Edema, allodynia, hyperalgesia and skin temperature changes are

often seen in all three phases. Osteoporosis is more characteristic with the atrophic (III)

stage of the disease.

1. Raj PP. Pain Medicine - A Comprehensive Review. Mosby. St. Louis. 1996:473-474. 82. E No treatment Phantom limb sensation is experienced by almost everyone who has a limb amputated

and must be distinguished from phantom limb pain. Unlike phantom limb pain, phantom

limb sensation is not painful and therefore does not require treatment. The sensation is

very vivid and often is described as a “limb” occupying the same space as the amputated

limb. Over time the sensation fades.

1. Nikolajsen L, et. al., Phantom Limb Pain, British Journal of Anaesthesia, 2001:87,107-116.

Page 140: 2004 Anesth Practice Ques eBook

Anesthesia Practice Questions: Book 1 – 2004

This material may not be duplicated without written permission from the publisher. Copyright 2004, All Rights Reserved Medtext Medical World, Inc.

140

83. D Allodynia Allodynia is pain due to a stimulus that is not normally painful such as the stroke of a

feather. This sensation is very characteristic in reflex sympathetic dystrophy.

Hyperalgesia is an increase response to a stimulus that is normally painful, while

hyperesthesia is an exaggerated pain response to a noxious stimulus such as pressure or

heat. Hypoalgesia is decrease pain to a stimulus that is normally painful, while analgesia is

the absence of pain to a stimulus that is normally painful.

1. Cousins MJ, Bridenbaugh PO. Neural Blockade in Clinical Anesthesia and Management of Pain. 3rd edition. Lippincott. Philadelphia. 1998:701-703. 84. B Celiac plexus block The celiac plexus is the largest of the great plexuses of the sympathetic nervous system.

The cardiac plexus innervates primarily thoracic structures, the celiac plexus innervates

abdominal organs, and the hypogastric plexus supplies pelvic organs. All three contain

visceral afferent and efferent fibers. In addition, they contain parasympathetic fibers that

pass through these ganglia after originating in cranial or sacral areas of the nervous

system.

Celiac block is indicated in patients with pain arising from the abdominal viscera, especially

pancreatic cancer. Relief of pain is almost immediate. Celiac plexus block performed with a

neurolytic agent can provide relief of pain for up to 4 months. The celiac plexus is generally

clustered at the level of the body of L1, posterior to the vena cava on the right, just lateral

to the aorta on the left, and posterior to the pancreas.

1. Warfield, CA, Bajwa, ZH. Principles and Practice of Pain Medicine, 2nd edition, McGraw-Hill. New York NY. 2004:699-703.

Page 141: 2004 Anesth Practice Ques eBook

Anesthesia Practice Questions: Book 1 – 2004

This material may not be duplicated without written permission from the publisher. Copyright 2004, All Rights Reserved Medtext Medical World, Inc.

141

85. B Hypotension Hypotension is the most common complication of celiac plexus blockade. It can be reduced

by volume preloaded before performing the blockade. Hypotension along with pain relief is

also reliable signs of a successful celiac plexus block. Another common complication is

diarrhea due to increased peristalsis of the gut produced by the shift in the balance of the

parasympathetic and sympathetic innervations. This may occur within the first 12 hours

after the blockade and may be a source of relief to patients on chronic opioid therapy for

cancer pain. The most serious complication is the development of paralysis from

unrecognized subarachnoid injection of a neurolytic drug. Radiographic confirmation of

needle location is advisable before injection of any neurolytic drug.

1. Barash P, Cullen B, Stoelting R. Clinical Anesthesia, 4th edition, J.B. Lippincott. Philadelphia. 2001:734-735. 86. D Musculocutaneous nerve The brachial plexus is derived from the anterior primary rami of the C-5, C-6, C-7, C-8 and

T-1 nerves. As the nerve roots leave the intervertebral foramina, they converge, forming

trunks, divisions, cords, and then finally terminal nerves.

The axillary approach to the brachial plexus is the most popular because of ease of block,

reliability of hand and forearm anesthesia, and safety. This block is ideally suited for

outpatients. Axillary block is unsuitable for surgical procedures on the upper arm or

shoulder. At the level of the axillary, the musculocutaneous nerve has already left the

brachial plexus and travels within the coracobrachialis muscle. Supplementation of this

nerve is important since it provides sensory to the lateral portion of the forearm.

Supplementation of the axillary nerve is not required since anesthesia of that nerve is not

required for this procedure.

1. Miller R. Anesthesia. 5th edition. Churchill Livingstone. New York NY. 2000:1524-1527.

Page 142: 2004 Anesth Practice Ques eBook

Anesthesia Practice Questions: Book 1 – 2004

This material may not be duplicated without written permission from the publisher. Copyright 2004, All Rights Reserved Medtext Medical World, Inc.

142

87. D Lateral aspect of the forearm The musculocutaneous nerve provides sensation to the lateral aspect of the forearm. The

ulnar nerve provides sensory to the little finger and medial aspect ring finger with medial

aspect of the hand. The median nerve supplies sensory to the lateral part of the palm, the

thenar eminence, thumb, index and middle finger and the lateral half of the ring finger.

The medial cutaneous nerve provides sensory to the medial aspect of the forearm.

The musculocutaneous nerve is often spared after a block of the brachial plexus via an

axillary approach. Its supplementation is therefore important to facilitate surgery involving

the distal portions of the arm. Supplementation involved identifying the coracobrachialis

muscle and injecting local anesthetic into the body of the muscle.

1. Miller R. Anesthesia. 5th edition. Churchill Livingstone. New York NY. 2000:1524-1527 88. A Unilateral recurrent laryngeal nerve injury Recurrent laryngeal nerve injury continues to be one of the most common complications of

thyroid surgery. Unilateral recurrent laryngeal nerve causes hoarseness but no airway

compromise. The affects vocal cord is usually in the paramedian position. Bilateral

recurrent laryngeal nerve results in inspiratory stridor and airway compromise. Both vocal

cords are in the paramedian position. Unilateral vagal nerve injury (i.e., unilateral superior

laryngeal nerve and recurrent laryngeal nerve injury) leads to affected cord being

motionless and bowed in the lateral position. There is no airway compromise.

Unilateral superior laryngeal nerve injury causes the affected cord to appear shorter and

the voice is hoarse. Bilateral superior laryngeal nerve injury causes both cords to shorten

and the voice is also hoarse. Neither results in airway compromise.

1. Gravenstein N, Kirby RR. Complications in Anesthesiology. 2nd edition. Lippincott. Philadelphia. 1996:380.

Page 143: 2004 Anesth Practice Ques eBook

Anesthesia Practice Questions: Book 1 – 2004

This material may not be duplicated without written permission from the publisher. Copyright 2004, All Rights Reserved Medtext Medical World, Inc.

143

89. C Amitriptyline The pain associated with acute zoster and postherpetic neuralgia is neuropathic and results

from injury of the peripheral nerves and altered central nervous system signal processing.

Aspirin and other mild analgesic drugs are commonly used in patients with postherpetic

neuralgia, but their value is limited. Ibuprofen is ineffective. Neuropathic pain is also less

responsive to opioids such as morphine. Topical formulation of aspirin and lidocaine has

shown varied results, but are occasionally used.

Tricyclic antidepressant drugs are the most effective therapy for postherpetic neuralgia. Be-

cause of their ability to block the reuptake of norepinephrine and serotonin, these drugs

may relieve pain by increasing the inhibition of spinal neurons involved in pain perception.

They are especially effective for neuropathic pain.

1. Kost RG, et. al., Postherpetic Neuralgia - Pathogenesis, Treatment, and Prevention, New England Journal of Medicine, 1996; 335(1):32-42. 90. D ASA physical status II-E Assignment of physical status is based on the physical condition of the patient and is

independent of the planned operation. It is important to recognize that physical status

classification does not represent anesthetic risk. However, perioperative complications are

more frequent in patient with poor physical status.

Physical Status Classification Description

I A normal healthy patient.

II A patient with mild systemic disease that results in no functional limitations (Hypertension, diabetes, morbid obesity, extremes of age).

III A patient with severe systemic disease that results in functional limitations (poorly controlled hypertension, diabetes with vascular complications, angina pectoris).

IV A patient with sever systemic disease that is a constant treat to life (congestive heart failure, unstable angina, advanced pulmonary, renal or hepatic dysfunction).

V A moribund patient who is not expected to survive without the operation (ruptured abdominal aneurysm, head injury with elevated ICP).

VI A declared brain dead patient whose organ are being removed for donor purposes.

E EMERGENCY OPERATION 1. Stoelting RK, Miller RD. Basics of Anesthesia. 4th edition. Churchill Livingstone. New York NY. 2000:113-114.

Page 144: 2004 Anesth Practice Ques eBook

Anesthesia Practice Questions: Book 1 – 2004

This material may not be duplicated without written permission from the publisher. Copyright 2004, All Rights Reserved Medtext Medical World, Inc.

144

91. A All of the potent inhalational agents depress CMR (Cerebral Metabolic Rate) in a linear fashion All of the potent inhalational agents depress CMR (Cerebral Metabolic Rate) to varying

degrees in a non-linear fashion.

Isoflurane abolishes EEG activity at doses used clinically and can usually be tolerated from

a hemodynamic standpoint. Halothane produces isoelectric EEG only at 4.5%

concentration.

All of the potent agents increase CBF in a dose-dependent manner. Halothane is a very

potent cerebral vasodilator and causes the greatest increase in CBF per MAC multiple. In

human studies, isoflurane produces insignificant or no change in CBF.

1. Ebert TJ, Scmid PG: Inhalational anesthesia. In Barash PG, Cullen BF, Stoelting RK (eds): Clinical Anesthesia. 4th edition. Lippincott Williams & Wilkins, Philadelphia, PA. 2001:390 2. Newberg LA, Milder JH, Michenfelder JD: The cerebral metabolic effect of isoflurane at and above concentrations that suppress critical electric activity. Anesthesiology 1983; 59:23-28 3. Algottson L, Messeter K, Nordstrom CH et al: Cerebral blood flow and oxygen consumption during isoflurane and halothane anesthesia in man. Acta Anaesthesiol Scand 1988;32:15-20 92. B The expiratory time The inspiratory time influences the correlation between the set tidal volume and exhaled

tidal volume, not the expiratory time Gas flow from the anesthesia machine into the

breathing circuit is continuous. During the inspiratory phase of mechanical ventilation, the

ventilator relief valve is closed and the patient receives volume from the bellows as well as

from the flow meters. The factors that influence the correlation between set tidal volume

and exhaled tidal volume include the flow meter settings, the inspiratory time, the

compliance of the breathing circuit, external leakage and the location of the tidal volume

sensor.

1. Andrews JJ, Brockwell RC. Delivery systems for inhaled anesthetics. In Barash PG, Cullen BF, Stoelting RK (eds.). Clinical Anesthesia 4th edition. Lippincott Williams & Wilkins, Philadelphia, PA. 2001:584

Page 145: 2004 Anesth Practice Ques eBook

Anesthesia Practice Questions: Book 1 – 2004

This material may not be duplicated without written permission from the publisher. Copyright 2004, All Rights Reserved Medtext Medical World, Inc.

145

93. E Desflurane and sevoflurane cause dose-dependent increase in the myocardial contractility Halothane is notable for its decrease in cardiac output- an effect that contributes

importantly to its blood pressure lowering effect.

In volunteers, sevoflurane and halothane up to about 1 MAC results in minimal, if any,

changes in steady-state heart rate. In contrast isoflurane at 1 MAC and desflurane at >1

MAC have been associated with increase in heart rate of 10-20%.

There is a new body of literature describing the potential for organ-protective effect

(particularly cardioprotective effects) of the potent inhaled agents. This appears to be

related to a reduced loading of calcium into the myocardial cells during ischemia.

Although volatile anesthetics impair coronary autoregulation to some degree, these agents

do not produce the profound degree of coronary vasodilatation and inhibition of

autoregulation caused by adenosine or dipyridamole. Volatile anesthetics are only weak

coronary vasodilators.

All modern volatile anesthetics, including desflurane and sevoflurane, depress contractile

function in normal myocardium in vitro and in vivo.

1. Pagel PS, Farber NE, Warltier DC. Cardiovascular Pharmacology. In Miller RD (ed.): Anesthesia. 5th edition. Churchill Livingstone, Philadelphia, PA. 2000:96-116 2. Ebert TJ, Schmid PG in Inhalation Anesthesia. In Barash PG, Cullen BF, Stoelting RK (eds): Clinical Anesthesia. 4th edition. Lippincott Williams & Wilkins, Philadelphia, PA. 2001:392-398 3. Novalija E, Fujita S, Kampine JP et al: Sevoflurane mimics ischemic preconditionng effects on coronary flow and nitric oxide release in isolated hearts. Anesthesiology 1999;91:701-712 4. Conzen PF, Habazettl H, VollmarB et al: Coronary microcirculation during halothane, enflurane, isoflurane, and adenosine in dogs. Anesthesiology 1992;76:261-70

Page 146: 2004 Anesth Practice Ques eBook

Anesthesia Practice Questions: Book 1 – 2004

This material may not be duplicated without written permission from the publisher. Copyright 2004, All Rights Reserved Medtext Medical World, Inc.

146

94. A Rapid sequence induction should be planned with thiopentone and succinylcholine Anterior mediastinal masses are uncommon. By nature of their anatomical location, they

produce three problems: compression of the heart, compression of the large vessels

(principally the superior vena cava) and compression of the trachea and main bronchus. A

patient can be asymptomatic and yet have airway compression, which only manifests at

induction of anesthesia when voluntary control of the airway is lost. Unexpected and often

total airway obstruction occurs on induction of general anesthesia and conventional

solutions, e.g. tracheostomy will not be helpful because the obstruction is usually

intrathoracic and close to or below the carina.

Preoperative maximal inspiratory and expiratory flow volume curves helps to quantify the

degree of impairment and will also differentiate extrathoracic from intrathoracic

obstruction.

1. Goh MH, Liu XY, Goh YS. Anterior mediastinal masses: an anesthetic challenge. Case report. Anesthesia 1999;54:670-682 95. D An infant’s axis of the vocal folds is perpendicular to the trachea The five major anatomic differences between the neonatal and adult airway are:

1. Tongue- An infant’s tongue is relatively large in proportion to rest of the oral

cavity.

2. Position of larynx- An infant’s larynx is higher in the neck (C3-4) than is an

adult’s.

3. Epiglottis – An adult’s epiglottis is broad, and its axis is parallel to that of the

trachea. An infant’s epiglottis is narrower and angled away from the axis of the

trachea.

4. Vocal folds – An infant’s vocal folds have a lower attachment anteriorly than

posteriorly. (Adult’s axis of the vocal folds is perpendicular to the trachea).

5. Subglottis – The narrowest portion of an infant’s larynx is the cricoid cartilage;

in an adult, it is the rima glottidis.

1. Wheeler M, Cote CJ, Todres ID. Pediatric Airway. In Corte CJ, Tordes ID, Ryan JF, Goudsouzian NG (Eds)A Practice of Anesthesia for Infants and Children. 3rd ED. W.B Saunders, Philadelphia, PA. 2001:79-80

Page 147: 2004 Anesth Practice Ques eBook

Anesthesia Practice Questions: Book 1 – 2004

This material may not be duplicated without written permission from the publisher. Copyright 2004, All Rights Reserved Medtext Medical World, Inc.

147

96. D Is a dual chamber device with two leads VDD, a relatively recent development in pacemaker technology, is a single lead, atrial

sensing ventricular pacemaker. It is capable of pacing the ventricles only but can sense

electrical activity in both the chambers. A VDD pacemaker provides AV synchrony and

provides for a physiological rate increase. Because it senses the atrial activity, it can

increase the rate of ventricular pacing as needed to keep up with an increase in SA node

depolarization. It is not an appropriate pacemaker for a patient with atrial fibrillation or

flutter.

1. Gregorators G, Abraham J, Epstein AE, et al. ACC/AHA guidelines. Update for implantation of cardiac pacemakers and antiarrhythmia devices. Summery article: A report of the American College of Cardiology/American Heart Association Task Force on Practice Guidelines (ACC/AHA/NASPE committee to update the 1998 Pacemaker Guidelines) J Am Coll Cardiol. 2002;40(9): 1703-19. 97. D When cardiac arrhythmias are a concern, atropine may be preferable to glycopyrrolate

In order to block the muscarinic effects of edrophonium and neostigmine, anticholinergic

agents (atropine or glycopyrrolate) are routinely added to these reversal agents. Atropine

induces its vagolytic effect much more rapidly than does glycopyrrolate. Therefore, to

minimize the cardiovascular changes, atropine is better suited with rapidly acting

edrophonium, and glycopyrrolate is better suited with the slower acting neostigmine. When

glycopyrrolate is administered with edrophonium, there may be an initial bradycardia unless

it is administered at least 1 minute earlier. On the other hand, administration of atropine

with neostigmine will induce initial tachycardia. Since arrhythmias may occur,

anticholinesterase should be used with caution in patients with autonomic neuropathy.

When cardiac arrhythmias are a concern, glycopyrrolate may be preferable to atropine, and

should be given with anticholinesterases over 2-5 minutes.

1. Savarese JJ, Caldwell JE, Lien CA, Miller RD. Pharmacology of muscle relaxants and their antagonists. In Miller RD (ed.): Anesthesia. 5th edition. Churchill Livingstone, Philadelphia, PA. 2000:469

Page 148: 2004 Anesth Practice Ques eBook

Anesthesia Practice Questions: Book 1 – 2004

This material may not be duplicated without written permission from the publisher. Copyright 2004, All Rights Reserved Medtext Medical World, Inc.

148

98. A 0.10 ml 1;200,000 = 5 µg/ cc

Because 1 in 200,000 = 1 G in 200,000 cc or 1000mg in 200,000 or 1000micg in 200cc

or 5 micg /cc

Therefore in 20 cc you need 100micg.

1: 1000 epinephrine has 1G in 1000cc or 1000mg in 1000cc or 1000 micg in 1 cc

As shown above we need only 100 micg;

Therefore, take 0.1 cc and add that to 20 cc

0.1 cc of 1:1000 epinephrine has 100µg of epinephrine which is diluted in 20 cc to give 5µg

/cc.

1. CB, Strichartz GR. Local Anesthetics In Miller RD (ed.): Anesthesia. 5th edition. Churchill Livingstone, Philadelphia, PA. 2000:509 99. D 0.40 Supplemental oxygen can be administered via the low- and high-flow system to increase

the delivery of oxygen in hypoxemic patient. Inspired oxygen concentrations (FiO2)

achieved with nasal cannula depends upon:

• Flow rate of oxygen (L/min)

• Patient tidal volume

• Inspiratory flow rate

• Breathing frequency

• Volume of nasopharynx

Inhaled oxygen concentration is increased to 4% for each 1 L/min increment of oxygen

delivery. Beyond 6L/min (FiO2 45%) inhaled concentration of oxygen does not increase

because nasopharynx volume is fixed.

1. Stoelting RK, Miller RD. Basics of Anesthesia. 4th edition. Churchill Livingstone. New York, NY. 2000:421.

Page 149: 2004 Anesth Practice Ques eBook

Anesthesia Practice Questions: Book 1 – 2004

This material may not be duplicated without written permission from the publisher. Copyright 2004, All Rights Reserved Medtext Medical World, Inc.

149

100. B Change the face mask to nonbreathing mask A simple face mask does not include a valve or oxygen reservoir bag. The minimum

oxygen flow rate should be at least 5L/min to prevent rebreathing of CO2. At oxygen flow

rate of 8L/min, the mask can provide inhaled oxygen concentrations of 60%. The partial

rebreathing face mask is a valveless system with a reservoir bag. With oxygen flow rate of

10L/min the inhaled oxygen concentration can be between 50-65%. The nonbreathing

face mask includes unidirectional valve plus an oxygen reservoir bag. Inhaled

concentrations of oxygen can be increased to 100%. Venturi or air-entrainment mask

entrains large volume of room air to mix with the oxygen flow through the injector. The

resultant mixture of gases has inhaled inspired oxygen between 25-40%.

1. Stoelting RK, Miller RD. basics of Anesthesia. 4th edition. Churchill Livingstone. New York, NY. 2000:422. 101. A Administration of Nitroprusside and Propanolol Pheochromocytoma is a catecholamine – secreting tumor. More than 95% are found in the

abdominal cavity, 10% originate outside adrenal medulla, 10% are malignant, and 10% are

bilateral. The hallmark of pheochromocytoma is paroxysmal hypertension associated with

diaphoresis, headache, tremulousness, palpitation, and weight loss. The triad of

diaphoresis, tachycardia, and headache in a hypertensive patient is highly suggestive of

pheochromocytoma. The mainstay of pharmacologic therapy is alpha blockade with

phenoxybenzamine or prazosin followed by beta blockade with propanolol. In the above

example manipulation of adrenal produced surge of sympathetic activity and release of

catecholamine as evident by an acute increase in blood pressure and PVC’s. The best

treatment is nitroprusside which dilates peripheral vessels and propanolol which blocks the

action of catecholamine.

1. Stoelting RK, Dierdorf SF. Anesthedia and Coexisting Disease 4th edition. Churchill Livingstone, New York, NY. 2002:430-434.

Page 150: 2004 Anesth Practice Ques eBook

Anesthesia Practice Questions: Book 1 – 2004

This material may not be duplicated without written permission from the publisher. Copyright 2004, All Rights Reserved Medtext Medical World, Inc.

150

102. B Induction with Volatile Anesthetic is accelerated Ventricular septal defect (VSD) is characterized by a left to right intracardiac shunt that

results in high pulmonary blood flow. High pulmonary flow will accelerate induction with

volatile agents because greater volume of blood entering lung due to added shunt volume

take up anesthetic agents to deliver to systematic circulation. In contrast, the amount of

drug entering the circulation via intravenous route will be diluted thus delaying the

induction. Positive pressure ventilation will decrease L to R shunt due to increase

intrathoracic pressure. A decrease in systematic resistance, leading to hypotension will

decrease L to R shunt because of low shunting pressure; while an increase in systematic

vascular resistance with hypertension will increase L to R shunt because of increase in

shunting pressure on the left side.

1. Stoelting RK, Miller RD. Basics of Anesthesia 4th edition. Churchill Livingstone, New York, NY. 2002: 258. 103. A Thrombocytopenia leading to prolonged bleeding time and decreased plasma concentrations of labile factor V sufficient to prolong prothrombin time If packed red cells are used to replace massive blood loss, then based on simple dilution

after the loss of one blood volume, the percentage of the original coagulation factor levels

and platelets will likely be 30 to 40 percent. Unlike platelets which decline rapidly during

the first 24 hours of storage, labile factor V and VIII decline to levels of 50% after two

weeks of storage and then decrease slowly thereafter. Thus slow massive transfusion will

produce coagulopathy due to platelets and coagulation factors dilution. Symptomatic

hypocalcemia or hyperkalemia will occur if transfusion is sufficiently rapid.

1. Longnecker DE Jr., Tinker JH, Morgan GE. Principle and Practice of Anesthesiology 2nd edition. Mosby, New York, NY. 1998: 935, 2422.

Page 151: 2004 Anesth Practice Ques eBook

Anesthesia Practice Questions: Book 1 – 2004

This material may not be duplicated without written permission from the publisher. Copyright 2004, All Rights Reserved Medtext Medical World, Inc.

151

104. A By Increasing PAO2, PaCO2 Increases Impaired oxygen diffusion between capillaries wall and alveolar wall prevents complete

equilibration of alveolar gas with pulmonary capillary blood. Many factors affect diffusion

of oxygen through this barrier. For example increase distance between alveoli and

erythrocytes (such as in pulmonary fibrosis) will decrease (impair) diffusion; high cardiac

output shortens the transit time of the red blood cells through capillaries because of

increase in flow, atelectasis will impair diffusion because of increased diffusion barrier. On

the other hand, increase FiO2 with high PAO2 will increase O2 gradient and facilitate

diffusion; continuous positive airway pressure distends alveoli and facilitates diffusion.

Although multiple factors can affect PaO2, the most effective way of increasing PaO2 is by

increasing PAO2.

1. Marini JJ, Wheeler AP. Critical Care Medicine 2nd edition. Williams & Wilkins, Baltimore. 197:378. 105. B Decrease Cardiac Output A dose – dependent reduction of arterial pressure is due to direct myocardial depression

effects of halothane. Halothane at 2 MAC can decrease blood pressure and cardiac output

by 50%. The hypotensive effects on blood pressure are potentiated due to blunting of

inhibitory responses in the carotid and aortic pressure receptors. Bradycardia or junctional

rhythm may result due to slowing of sinuatrial node conduction. Although volatile

anesthetics do increase ventilation perfusion mismatch, this is less likely based on the

history.

1. Morgan GE Jr., Mikhail MS, Murray MJ, (eds). Clinical Anesthesiology 3rd edition. Lange Medical Books/McGraw Hill Publishing Division, New York, NY, 2000:139-140.

Page 152: 2004 Anesth Practice Ques eBook

Anesthesia Practice Questions: Book 1 – 2004

This material may not be duplicated without written permission from the publisher. Copyright 2004, All Rights Reserved Medtext Medical World, Inc.

152

106. D Observe in ICU for 24-hours The post-operative complications of parathyroidectomy include hemorrhage, hematoma,

and damage to recurrent laryngeal nerve. Unilateral damage to recurrent laryngeal nerve

manifests itself as hoarseness. Bilateral damage to recurrent laryngeal nerve causes airway

compromise. Hypoparathyroidism from parathyroidectomy leading to hypocalcemia rarely

manifest itself as early as 1 to 3 hours. Typically hypocalcemia does not appear until 24 to

72 hours post-operatively. Laryngeal muscles are very sensitive to hypocalcemia which is

manifested as laryngospasm and inspiratory stridor.

1. Stoelting RK, Miller RD. Basics of Anesthesia 4th edition. Churchill Livingstone, New York, NY. 2002:312-313. 107. D Primary Hemostasis Defect Hemostasis following trauma or surgery depends on three process: (1) localized spasm due

to local myogenic reflexes and the release of humoral factors from platelets, (2) formation

of a platelet plug (primary hemostasis), and (3) coagulation of blood (secondary

hemostasis). A defect in any of the sites can cause bleeding disorder. Formation of the

platelet plug involves: (1) adhesion, (2) release of platelet granules, and (3) aggregation.

The presence of petechiae suggests thrombocytopenia, functional abnormality of platelets

or defects in the integrity of the vascular walls.

Clotting factor disorders include hemophilia. These patients present with deeper tissue

bleeding. Vitamin K is necessary for synthesis of clotting factor II, VII, IX, and X.

Antiplatelet agents, such as aspirin, inhibit synthesis of thromboxane A2 for life of platelets.

1. Morgan GE Jr., Mikhail MS, Murray MJ, (eds). Clinical Anesthesiology 3rd edition. Lange Medical Books/McGraw Hill Publishing Division, New York, NY, 2000:717.

Page 153: 2004 Anesth Practice Ques eBook

Anesthesia Practice Questions: Book 1 – 2004

This material may not be duplicated without written permission from the publisher. Copyright 2004, All Rights Reserved Medtext Medical World, Inc.

153

108. C Antiplatelet Agents Uremia is seen in patients with end-stage renal disease when GFR decreases below 25

ml/min. Uremia produce platelet defect which is due to accumulation of toxic metabolites

that reduce platelet aggregation. Dialysis provides temporary benefit. Desmopressin

(DDAVP) also improves platelet function. Long term alcohol consumption leads to platelet

dysfunction. Antiplatelet agents (aspirin) are the most common cause of platelet

dysfunction. These drugs inhibit synthesis of thromboxane A2. Fibrin degradation products

produced from fibrin, inhibit crosslinking of fibrin strands and coat surface of platelets

which inhibit platelet aggregation. Vitamin K is needed for synthesis of clotting factors II,

VII, IX, and X.

1. Stoelting RK, Miller RD. Basics of Anesthesia. 4th edition. Churchill Livingstone. New York, NY. 2000:244. 109. D All of the above

Aminocaproic acid Tranexamic acid Aprotinin

Conversion of plasminogen to plasmin by plasminogen activator enzymes is responsible for

fibrinolysis. Fibrinolysis leads to the dissolution of fibrin clots and is necessary to restore

normal blood flow in condition such as DIC. DIC stimulates fibrinolysis as a defense

mechanism to lyse blood clots. Aminocaproic (EACA) tranexamic acid (TXA) and aprotinin

are antifibrinolytic. EACA and TXA bind to both plasminogen and plasmin molecules and

prevent plasmin from degrading fibrinogen and fibrin. This leads to reduced lysis and

reduced formation of fibrinogen degradation products. Aprotinin is an inhibitor of plasmin

and kallikrein.

1. Morgan GE Jr., Mikhail MS, Murray MJ, (eds). Clinical Anesthesiology 3rd edition. Lange Medical Books/McGraw Hill Publishing Division, New York, NY, 2000:721.

Page 154: 2004 Anesth Practice Ques eBook

Anesthesia Practice Questions: Book 1 – 2004

This material may not be duplicated without written permission from the publisher. Copyright 2004, All Rights Reserved Medtext Medical World, Inc.

154

110. D It is an ungrounded system on the secondary side of transformer Leakage current is present in all electrical equipment as a result of defective insulation,

capacitive coupling or induction between internal electrical circuits. The magnitude of such

leaks is imperceptible (less than 1 mA). However (100 uA) can cause ventricular fibrillation

if it reaches the heart directly. Line isolation monitor is a device that continuously monitors

the integrity of the isolated power system and protects a patient from macro shock not

microshock (100 uA current).

1. Morgan GE, Mikhail MS, Murray MJ (eds.). Clinical Anesthesiology. 3rd edition. Lange Medical Books/McGraw-Hill Medical Publishing Division. New York, NY. 2002:23 111. A External DC cardioversion In the presence of hemodynamically compromised ventricular tachycardia immediate

synchronized DC cardioversion is required. Ventricular tachycardia not associated with

hypotension is best managed by intravenous administration of amiodarone or

procainamide. Ventricular tachycardia is defined as the presence of at least three

consecutive wide QRS complexes on EKG occurring at an effective rate of 120 bpm or

higher. Amiodarone nor a calcium channel blockers or phenylephrine will be an

appropriate choice.

1. Barash PG, Cullen BF, Stoelting RK (eds.). Handbook of Clinical Anesthesia. 4th edition. Lippincott Williams & Wilkins. Baltimore MD. 2001:847. 112. E Low - 20 Dibucaine is a local anesthetic which inhibits normal pseudocholinesterase activity by 80%,

the homozygous atypical enzyme by 20% and heterozygous atypical enzyme by 40-50%.

The enzyme activity is measured in terms of hydrolysis of acetylcholine by

pseudocholinesterase. Dibucaine number is the percentage inhibition of

pseudocholinesterase activity and is proportional to pseudocholinesterase function (normal

or atypical). Thus, the presence of an atypical form of enzyme in circulation will lead to

Page 155: 2004 Anesth Practice Ques eBook

Anesthesia Practice Questions: Book 1 – 2004

This material may not be duplicated without written permission from the publisher. Copyright 2004, All Rights Reserved Medtext Medical World, Inc.

155

prolonged muscle relaxation with succinylcholine because of slow hydrolysis. In this

example, prolonged effect of succinylcholine with inadequate muscle strength after an hour

most likely indicates homozygous atypical enzyme.

1. Morgan GE, Mikhail MS, Murray MJ (eds.). Clinical Anesthesiology. 3rd edition. Lange Medical Books/ McGraw-Hill Medical Publishing Division. New York, NY. 2002:183. 113. C Compression of fetal head Compression of fetal head or stretching of the neck during uterine contractions leads to

uniform deceleration (usually 10 – 40 beats/min.) of fetal heart rate. These are thought to

be mediated via vagus nerve and are not indicative of fetal distress. Since the question

says uniform changes in fetal heart rate during uterine contractions, the best choice is C.

Late or Type II decelerations follow the peak of uterine contractions and begin 10-30

seconds after the onset of the uterine contraction. These decelerations are thought to be

due to the effect of a decrease in arterial oxygen tension and maternal hypotension leading

to uteroplacental insufficiency. These are associated with fetal distress. Variable or Type

III decelerations are variable in onset, duration and magnitude and are thought to be due

to umbilical cord compression. These decelerations are generally benign unless associated

with fetal bradycardia (HR slower than 70 bpm) or prolonged beyond 30 seconds.

1. Morgan GE, Mikhail MS, and Murray MJ, (eds.). Clinical Anesthesiology. 3rd edition. Lange Medical Books/McGraw-Hill Medical Publishing Division. New York, NY. 2002:841. 114. A Heart rate This is a case of autonomic hyperreflexia (AH). Autonomic hyperreflexia should be

expected in patients with spinal cord lesions above T6 levels, which can be precipitated by

surgical manipulations. AH is seen after the resolution of spinal shock and the return of

spinal reflexes 1-3 weeks after injury. In AH, sympathetic nervous system below the level

of spinal cord transection is functionally isolated from inhibitory influences of the brain. Any

cutaneous or visceral stimulus below the level of the lesion produces intense sympathetic

discharge with hypertension and vasoconstriction. A baroreceptor parasympathetic

Page 156: 2004 Anesth Practice Ques eBook

Anesthesia Practice Questions: Book 1 – 2004

This material may not be duplicated without written permission from the publisher. Copyright 2004, All Rights Reserved Medtext Medical World, Inc.

156

mediated compensatory reflex above the lesion produces reflex bradycardia and

vasodilation. Deep general anesthesia and regional anesthesia (spinal) prevents AH.

Changes in heart rate and chronic vasodilation leading to hypothermia are most common in

these patients.

1. Morgan GE, Mikhail MS, Murray MJ, (eds.). Clinical Anesthesiology. 3rd edition. Lange Medical Books/McGraw-Hill Medical Publishing Division. New York , NY. 2002:590. 115. B Increased sensitivity to depolarizing and nondepolarizing muscle relaxants This patient has myasthenic syndrome (Eaton-Lambert syndrome). The syndrome is due to

the presence of antibodies which are directed to presynaptic calcium channels that

markedly reduces the quanta of acetylcholine released from the presynaptic sites. Repeated

efforts, exercise and drugs such as 4-aminopyridine that stimulate the presynaptic release

of acetylcholine improve skeletal muscle strength. There is poor response to

anticholinesterases and increased sensitivity to succinylcholine and nondepolarizing muscle

relaxants. Therefore, muscle relaxants should be used in very small amounts. In general

volatile agents are sufficient to provide muscle relaxation for both intubation and surgical

procedures. Immunosuppressive agents will not affect the muscle strength.

1. Stoelting RK, Dierfdorf SF (eds.). Anesthesia and Coexisting Disease. 4th edition. Churchill Livingstone. New York, NY. 2002:527. 116. A See the patient sometime today for pre-anesthetic evaluation Generally anesthesiologists see the patient prior to the day of scheduled outpatient surgery

unless some unusual circumstances prevent this evaluation. The objective is to perform

history, physical examination, order preoperative tests and assess the suitability for an

outpatient surgery. In this elderly patient it may be appropriate to order preoperative EKG,

CBC and basic electrolytes including BUN, creatinine and glucose to assess the cardiac and

renal functional status (most commonly affected systems in the elderly patients). It is not

necessary to obtain routine radiographs in the absence of positive findings on the history or

Page 157: 2004 Anesth Practice Ques eBook

Anesthesia Practice Questions: Book 1 – 2004

This material may not be duplicated without written permission from the publisher. Copyright 2004, All Rights Reserved Medtext Medical World, Inc.

157

physical examination. There is nothing in history or physical examination to suggest that

an outpatient procedure could be hazardous.

1. Stoelting RK, Miller RD, (eds.). Basics of Anesthesia. 4th edition. Churchill Livingstone. New York, NY. 2002:392-393. 117. E Elderly patients with a history of hypertension and diabetes Age is not a factor in the selection of patients for outpatient surgery. In elderly patients,

acceptability for outpatient surgery is influenced by physical status of the patient, prior

history and the ability to be cared for by a competent adult at home. Patients not likely to

be candidate for outpatient surgeries are: patients with a history of prolonged nausea,

vomiting and pain, or presence of poorly controlled systematic illness; patients prone to

hospital acquired infections, neonates born prematurely and infants with past history of

respiratory distress syndrome or a family history of sudden infant death syndrome.

1. Stoelting RK, Miller RD, (eds.). Basics of Anesthesia. 4th edition. Churchill Livingstone. New York, NY 2002:392-394. 118. B Calcium Hypermagnesemia (>2.5 mg/dl) is generally iatrogenic (treatment of gestational

hypertension). However, it can also occur due to excessive intake (antacid and laxative

abuse), renal impairment, adrenal insufficiency, hypothyroidism, rhabdomyolysis and

lithium administration. Symptomatic hypermagnesemia presents with neurologic,

neuromuscular manifestations consisting of hyporeflexia, sedation, skeletal muscle

weakness, impaired pre-junctional release of acetylcholine and decrease in motor end-plate

sensitivity to acetylcholine. The cardiovascular symptoms of hypermagnesemia consist of

vasodilation, bradycardia, myocardial depression and respiratory arrest (24 mg/dl). For

symptomatic hypermagnesemia, calcium is used to reverse magnesium effects and

mechanical ventilatory support is continued until the effects dissipate.

1. Stoelting RK, miller RD, (eds.). Basics of Anesthesia. 4th edition. Churchill Livingstone. New York, NY. 2002: 236.

Page 158: 2004 Anesth Practice Ques eBook

Anesthesia Practice Questions: Book 1 – 2004

This material may not be duplicated without written permission from the publisher. Copyright 2004, All Rights Reserved Medtext Medical World, Inc.

158

119. D Administer oxygen and intravenous calcium gluconate Hypoparathyroidism from unintentional removal of the parathyroid glands can cause acute

hypocalcemia. A decrease in plasma calcium concentration (less than 4.5 mEq/L) is the

most diagnostic indicator of hypoparathyroidism. This is manifested with muscle cramps,

weakness, increased neuromuscular irritability, changes in mental status, hypotension and

EKG changes (prolonged QT interval). Symptoms of hypocalcemia generally develop in 24-

48 hours after surgery. Other possible complications of thyroidectomy include recurrent

laryngeal nerve palsy with unilateral palsy causes hoarseness or aphonia, and bilateral

palsy resulting in stridor. The history and physical examination of this patient strongly

suggests hypocalcemia (multisystem symptoms) which should be treated with intravenous

administration of 10% CaCl2 (3-5 ml) or calcium gluconate solution (10 ml).

1. Morgan GE, Mikhail MS, and Murray MJ, (eds.). Clinical Anesthesiology. 3rd edition. Lange Medical Books/McGraw-Hill Medical Publishing Division. New York, NY. 2002: 742-744. 120. C His physiologic dead space is 300 ml due to increase in V/Q mismatch Physiologic dead space is composed of anatomical dead space as well as alveolar dead

space (that is alveoli that are not perfused). In the upright position dead space is about

150 ml in adults. Physiologic dead space increases during general anesthesia due to

increase in V/Q mismatch. Use Bohr’s equation to calculate dead space.

VD/VT = (PACO2 – PECO2) / PACO2

Where PACO2 is alveolar CO2 tension and PECO2 is the mixed expired CO2 tension. The

arterial CO2 tension (PaCO2) can be used to approximate the alveolar concentration.

1. Morgan GE, Mikhail MS, and Murray MJ, (eds.). Clinical Anesthesia. 3rd edition. Lange Medical Books/McGraw-Hill. New York, NY. 2002:490.

Page 159: 2004 Anesth Practice Ques eBook

Anesthesia Practice Questions: Book 1 – 2004

This material may not be duplicated without written permission from the publisher. Copyright 2004, All Rights Reserved Medtext Medical World, Inc.

159

121. D The difference between diastolic arterial and pulmonary wedge pressures The main determinants of coronary perfusion pressure are aortic diastolic pressure (DP)

and left ventricular end-diastolic pressure (LVEDP). Wedge pressure is measured to

evaluate the preloading conditions of the left ventricle which correlates with left ventricular

end-diastolic pressure (LVEDP). This wedge pressure should be measured at the end of

left ventricular diastole. On venous pressure tracing this corresponds to the end of A-wave

before the appearance of C-wave, taken at end expiration. Therefore in the question

asked, D is the most appropriate choice. Central venous pressure is used as an index of

right ventricular preload, although it may approximate left ventricular preload in most

normal individuals. Therefore, choice B will be the second most plausible choice.

Endocardium is most vulnerable to ischemia because it is subject to greatest intramural

pressure. Any pathophysiology which will change DP (hypertension, ventricular

hypertrophy, tachycardia with decrease in filling volume and pressure) and LVEDP (left

ventricular hypertrophy, changes in compliance) will affect the subendocardial perfusion

pressure.

1. Morgan GE, Mikhail MS, and Murray MJ, (eds.). Clinical Anesthesiology. 3rd edition. Lange Medical Books/ McGraw-Hill Medical Publishing Division. New York, NY. 2002: 376. 122. C Administer 2 mg edrophonium intravenously and recheck MIF This patient is 2 hours post-surgery. Continued muscle weakness as evident from high

PC02, low MIF strongly suggests weakness due to myasthenia gravis and not to the

continued presence of muscle relaxant (rocuronium). Although, myasthenic patients are

more sensitive to nondepolarizing muscle relaxants, > 25% of twitch response returns back

in 25 minutes following administration of rocuronium. Of the two choices B and C, choice

C is the most plausible choice. Edrophonium is a short acting anticholinesterase and

intravenous administration of this drug should produce transient improvement in muscle

strength. Neuromuscular blockade should be monitored with a nerve stimulator before

administration of edrophonium. This will help in understanding the etiology of continued

weakness of this patient and make additional plans for management (such as continued

ventilation or treatment with neostigmine). A peak inspiratory pressure of –20 cm H2O,

Page 160: 2004 Anesth Practice Ques eBook

Anesthesia Practice Questions: Book 1 – 2004

This material may not be duplicated without written permission from the publisher. Copyright 2004, All Rights Reserved Medtext Medical World, Inc.

160

adequate oxygenation, sustained head lift and vital capacity > 15 ml/kg, is crucial before

extubation.

1. Morgan GE, Mikhail MS, and Murray MJ (eds.). Clinical Anesthesiology. 3rd edition. Lange Medical Books/McGraw-Hill Medical Publishing Division. New York, NY. 2002: 754. 123. C Administration of recombinant factor VIII to increase factor VIII to 50% normal The most common inherited defect in secondary hemostasis is factor VIII deficiency. It is

inherited as x-linked abnormality. Patients with less than 5% of normal factor VIII activity

are symptomatic. Most clinicians recommend factor VIII levels to be 50% or greater prior

to surgery. Although fresh frozen plasma, cryoprecipitate can be used to replace factor

VIII deficiency, recombinant factor VIII is currently favored in order to avoid transmission

of hepatitis B or C, and HIV.

1. Morgan GE, Mikhail MS, and Murray MJ, (eds.). Clinical Anesthesiology. 3rd edition. Lange Medical Books/McGraw-Hill Publishing Division. New York, NY. 2002:722. 124. B Endotracheal intubation as soon as possible This is a case of a burn patient with possible inhalational injury. Singed eyebrows,

eyelashes or black soot around the nose should increase the suspicion of upper and lower

airway injury. The first priority in the management of these patients is administration of the

highest possible concentration of oxygen by a face mask. Early intubation is indicated if the

airway reflexes are depressed or there is suspicion of impending airway obstruction. Airway

humidification, bronchial toilet and the use of bronchodilator, if needed, is indicated. A

high FiO2 improves oxygenation and promotes elimination of carboxyhemoglobin. Normal

oxygen saturation from a pulse oximeter does not exclude the possibility of carbon

monoxide toxicity.

1. Morgan GE, Mikhail MS, Murray MJ, (eds). Clinical Anesthesiology. 3rd edition. Lange Medical Books/ McGraw-Hill Medical Publishing Division. New York, NY. 2002:801-803.

Page 161: 2004 Anesth Practice Ques eBook

Anesthesia Practice Questions: Book 1 – 2004

This material may not be duplicated without written permission from the publisher. Copyright 2004, All Rights Reserved Medtext Medical World, Inc.

161

125. A Physostigmine, scopolamine, carbon dioxide Physostigmine, neostigmine and edrophonium are cholinesterases. Edrophonium and

neostigmine have quaternary ammonium group which bind to the anionic site of

acetylcholinesterase. Physostigmine is a tertiary amine which crosses the blood-brain-

barrier. This limits its usefulness as a reversal agent for nondepolarizing blockade. Atropine,

scopolamine and glycopyrrolate are anticholinergics. Glycopyrrolate is a synthetics

quaternary ammonium containing compound and in contrast to scopolamine and atropine

does not cross the blood-brain-barrier. Scopolamine has greater CNS effects than atropine.

Since bicarbonate is a negatively charged molecule it can not cross blood-brain-barrier.

However it can combine with a proton to form carbonic acid which dissociates into carbon

dioxide and water. Carbon dioxide can diffuse freely through the blood-brain-barrier,

reform carbonic acid which then dissociates into bicarbonate and hydrogen ion.

1. Morgan GE, Mikhail MS, Murray MJ, (eds.). Clinical Anesthesiology. 3rd edition. Lange Medical Books/McGraw-Hill Publishing Division. New York, NY. 2002:210. 126. A Hyperkalemia Spironolactone is a K- sparing diuretic. It is a competitive inhibitor of aldosterone. It often

is co-administered with thiazide or loop diuretics in the treatment of hypertension and

edema. These combinations result in increased mobilization of edema fluid with fewer

changes in the redistribution of K ions. Spironolactone is also used in the treatment of

primary hyperaldosteronism, refractory edema associated with ascites, cirrhosis, cardiac

failure, nephritic syndrome. Its side effects include: hyperkalemia, metabolic acidosis in

cirrhotic patients, impotence, gynecomastia, hirsutism and menstrual irregularities.

Spironolactone does not produce hypercalcemia, hypoglycemia or hyperglycemia, although

it may produce hyponatremia.

1. Morgan GE, Mikhail MS, Murray MJ, (eds.) Clinical Anesthesiology. 3rd edition. Lange Medical Books/McGraw-Hill Publishing Division. New York, NY. 2002:676.

Page 162: 2004 Anesth Practice Ques eBook

Anesthesia Practice Questions: Book 1 – 2004

This material may not be duplicated without written permission from the publisher. Copyright 2004, All Rights Reserved Medtext Medical World, Inc.

162

127. A Intestinal obstruction, tremor and hepatic dysfunction Metoclopramide is a substituted benzamide which block D2 dopamine receptors and 5-HT3

receptors at higher concentrations. It enhances gastric emptying and transit of intestinal

contents from the duodenum to the ileocecal valve through action on smooth muscles of GI

wall. Therefore it is contraindicated in patients with intestinal obstruction. The

bioavailability is only 75% after oral administration due to hepatic first pass metabolism

(therefore not a desirable drug in patients with hepatic dysfunction). Because of its

antagonism at D2-dopamine receptors, its use is contraindicated in patients with

Parkinsonism. Other effects of metoclopramide include: increase lower gastroesophageal

tone, drowsiness, dizziness, depression (crosses BBB). It has no effect on gastric secretions

or colonic motility. It is useful as antiemetic agent in emesis during cancer therapy and

pregnancy. Doses of digoxin and insulin may need adjustment if metoclopramide is given

orally.

1. Hardman JG, Limbird LE, Molinoff PB, Ruddon RW, Gilman AG, (eds.). The Pharmacological basis of Therapeutics. 9th edition. McGraw-Hill. New York, NY. 1996:933. 128. B Maximum mid expiratory flow (MMEF) Spirometry provides useful information of expired volumes. For nonpulmonary surgery, it

rarely provides additional information which can not be obtained from history and physical

examination. Maximum breathing capacity (MBC), or maximum voluntary ventilation (MVV),

forced expiratory volume in one second, negative inspiratory pressure and peak inspiratory

flow rate, are all effort dependent and require patient’s motivation and cooperation. A low

value in these parameters may suggest poor functional capacity but in reality may only

reflect suboptimal effort. Maximal mid-expiratory flow and forced expiratory flow reflect

effort independent parameters. A decreased mid-expiratory-flow rate reflects collapse of

the small airways and is a sensitive indicator of early airway obstruction. Therefore, of the

choices given, choice B is the most appropriate choice.

1. Duke James, (eds.). Anesthesia Secrets. 2nd edition. Hanley and Belfus, Inc. Philadelphia, PA. 2000:431. 2. Barash PG, Cullen BF, Stoelting RK, (eds.). Clinical Anesthesia. 4th edition. Lippincott Williams & Wilkins. Philadelphia, PA. 2001:814.

Page 163: 2004 Anesth Practice Ques eBook

Anesthesia Practice Questions: Book 1 – 2004

This material may not be duplicated without written permission from the publisher. Copyright 2004, All Rights Reserved Medtext Medical World, Inc.

163

129. A Airway suctioning, intubation, mechanical ventilation with PEEP and observation in ICU This is a case of gastric aspiration. The most appropriate course of action would be airway

suction, intubation and mechanical ventilation especially in view of the fact that the patient

is desaturating. Trachea is suctioned before positive pressure ventilation. This avoids

disseminating aspirated material into distal airways. In the presence of hypoxemia,

increased airway resistance and pulmonary edema, the patient is ventilated with

supplemental oxygen, peep or CPAP. Use of steroids or prophylactic antibiotics use is not

recommended. If evidence of bacterial infection appears, antibiotics therapy should be

based on Gram stain and culture of sputum. Prophylactic use of antibiotics is not

recommended. Saline lavage is also not recommended. However bronchoscopy may be

necessary to relieve airway obstruction caused by inhaled food particles. .

1. Barash PG, Cullen BF, Stoelting RK, (eds.). Clinical Anesthesia. 4th edition. Lippincott and Wilkins. Philadelphia, PA. 2001:1392. 130. C Deepen the level of anesthesia with volatile anesthetic This is a case of foreign body aspiration in a pediatric patient. The foreign body may be

irritating or nonirritating to the airways. It should be removed within 24 hours as it may

cause residual pulmonary disease or pneumonia. Induction of anesthesia is individualized in

each case. Anesthesia is induced and maintained with volatile anesthetic agents in

spontaneously breathing patient. After an adequate depth of anesthesia direct

laryngoscopy is performed. However, if the patient has recently eaten, full stomach

precaution should be taken and anesthesia is induced intravenously by rapid sequence

induction and cricoid pressure. Use of nitrous oxide should be avoided to reduce airway

trapping distal to obstruction. In this particular patient, deepening of anesthesia with

sevoflurane will relax tracheal muscle. One can administer nebulized albuterol or

intravenous bronchodilator to relieve bronchospasm. Alternatively, skeletal muscle

paralysis produced with succinylcholine may be required to remove the foreign body

through the vocal cords.

1. Barash PG, Cullen BF, Stoelting RK, (eds.) Clinical Anesthesia. 4th edition. Lippincott and Wilkins. Philadelphia, PA. 2001:996.

Page 164: 2004 Anesth Practice Ques eBook

Anesthesia Practice Questions: Book 1 – 2004

This material may not be duplicated without written permission from the publisher. Copyright 2004, All Rights Reserved Medtext Medical World, Inc.

164

131. B Cancel the surgery and monitor the child Masseter muscle spasm (MMR) or trismus is a forceful contraction of jaw that prevents full

mouth opening. MMR occurs in 15 – 30% of the patients susceptible to malignant

hyperthermia (MH). Since the above surgery is an elective surgery, the safest course is to

postpone the surgery and observe the patient for signs and symptoms of MH. Serum

creatine kinase (CK) levels should be followed for 24 hours after an episode of MH. This is

not a sensitive test but elevation of CK indicates an underlying myopathy. The most reliable

test to diagnose MH is halothane-caffeine contracture test. Some anesthesiologist may

switch to non-triggering anesthetic agents for the maintenance of anesthesia and allow the

surgery to continue so long as no other signs of malignant hyperthermia develop.

Dantrolene administration is recommended only when there is a clear diagnosis of MH.

1. Morgan GE, Mikhail MS, Murray MJ, (eds.) Clinical Anesthesiology. 3rd edition. Lange Medical Books/ McGraw Hill Publishing Division, New York, NY. 2002:869. 132. A Topical anesthesia of airway followed by awake oral intubation with in line stabilization of the neck This is a post-trauma patient. Because of the emergent nature of the surgery the details of

the injuries are unknown. The most appropriate choice is A which is least traumatic to the

patient. Immobilization of the neck in the neutral position is indicated in unconscious or

conscious patients with cervical pain or tenderness. Nasotracheal intubation caries risk of

epistaxis and possible entry of the endotracheal tube into the cranial vault if there is

damage to cranial base or maxillofacial complex. LeFort 1 fracture involves the maxilla and

the maxillary sinus. LeFort 2 and 3 fractures involve the thick portion of the nasal septum.

In LeFort 2 fracture extends to the medial side of the orbit whereas LeFort 3 fracture

involves the orbit laterally and extends towards the temporal bone.

1. Barash PG, Cullen BF, Stoelting RK, (eds.) Clinical Anesthesia. 4th edition. Lippincott and Wilkins. Philadelphia, PA. 2001:1256-1260.

Page 165: 2004 Anesth Practice Ques eBook

Anesthesia Practice Questions: Book 1 – 2004

This material may not be duplicated without written permission from the publisher. Copyright 2004, All Rights Reserved Medtext Medical World, Inc.

165

133. A Diffusion barrier to oxygen The main features of ARDS are:

• diminished lung compliance

• refractory hypoxemia

• diffuse radiographic changes in lungs

• low or normal pulmonary capillary wedge pressure

• pulmonary edema

Hypoxemia is the result of right to left shunting of blood through collapsed or fluid filled

alveoli. Mechanical ventilatory support with PEEP is required to maintain adequate

oxygenation. The most significant improvement after recovery from ARDS is improvement

in oxygenation.

1. Barash PG, Cullen BF, Stoelting RK, (eds.). Clinical Anesthesia. 4th edition. Lippincott and Wilkins. Philadelphia, PA. 2001:1468. 134. C Cardioversion Severe narrowing of aortic valve results in chronic obstruction to left ventricular ejection

which leads to increased intraventricular systolic pressure and concentric ventricular

hypertrophy. This ultimately leads to decrease in ventricular contractility and stroke

volume. Because the ventricle is so stiff, atrial contraction is critical for maintaining

ventricular filling and stroke volume. This “atrial kick” may account up to 30-40% of left

ventricular end-diastolic volume. The enlarged muscle mass increases myocardial oxygen

requirement and myocardium may be susceptible to ischemia even in the absence of

coronary artery disease. This patient is post-aortic valve replacement. The pathophysiologic

changes in the heart muscle induced from stenosed aortic valve still exist. Atrial fibrillation

with tachycardia reduces ventricular filling and stroke volume even further resulting in

hypotension and reduced coronary perfusion. This leads to cardiac ischemia and congestive

Page 166: 2004 Anesth Practice Ques eBook

Anesthesia Practice Questions: Book 1 – 2004

This material may not be duplicated without written permission from the publisher. Copyright 2004, All Rights Reserved Medtext Medical World, Inc.

166

heart failure. The most appropriate immediate first step in the management of this patient

is conversion to sinus rhythm and reduction of the heart rate through cardioversion.

1. Morgan GE, Mikhail MS, Murray MJ, (eds.). Clinical Anesthesia. 3rd edition. McGraw-Hill New York, NY. 416-417. 135. B Administer nitroprusside Topical conjunctival application of phenylephrine produces papillary dilation and capillary

congestion. Systemic effects such as hypertension, headache, tachycardia and

tremulousness may occur. This is particularly important in patients with coronary artery

disease who may develop severe myocardial ischemia, cardiac dysrhythmias and

myocardial infarction. Phenylephrine is a pure alpha-agonist which increases venous

contraction more than arterial contraction, increases preload and to some extent afterload

as well. The net effect is increase in blood pressure and coronary flow. Lidocaine will not

antagonize the effects of phenylephrine. Propanolol is a beta blocking agent, which will

make the bradycardia worst. Atropine and glycopyrrolate because of antimuscarinic effects

will increase the heart rate but will have no effect on peripheral resistance. Therefore, the

most appropriate choice is B. Nitroprusside acts on smooth muscle, causes dilation of

arterial and venous vessels and decreases pre- and afterload. Normalization of blood

pressure will inhibit reflex bradycardia.

1. Barash PG, Cullen BF, Stoelting RK, (eds.). Clinical Anesthesia. 4th edition. Lippincott and Wilkins. Philadelphia, PA. 2001: 296, 975.

Page 167: 2004 Anesth Practice Ques eBook

Anesthesia Practice Questions: Book 1 – 2004

This material may not be duplicated without written permission from the publisher. Copyright 2004, All Rights Reserved Medtext Medical World, Inc.

167

136. C PaCO2 Cerebral blood flow is dependent upon cerebral perfusion pressure, respiratory gases

tension especially PaCO2, temperature, viscosity and autonomic influences. Cerebral

perfusion pressure is autoregulated between 50 – 150 mmHg. The most important factor

controlling blood flow to ischemic brain tissue is PaCO2. Increasing CO2 causes vasodilation

and increased blood flow to nonischemic areas because the focal ischemic areas are

already maximally dilated due to the metabolic demands of the ischemic tissue. Thus a high

PaCO2 would shunt blood to areas of less demand. Reducing blood CO2 with

hyperventilation or reducing (CMRO2) would reduce blood flow to most areas of brain due

to vasoconstriction and increase blood flow to the vessels supplying the ischemic areas.

This phenomena “inverse steal or Robin Hood effect” would have the effect of maximizing

blood flow to compromised areas.

1. Barash PG, Cullen BF, Stoelting RK, (eds.). Clinical Anesthesia. 4th edition. Lippincott and Wilkins. Philadelphia, PA. 2001:746. 137. C Increased drug metabolism and increased isoflurane MAC Hypothermia is defined as a body temperature of less than 36° C. Hypothermia has a

protective effect on cerebral and cardiac ischemia due to a reduction in metabolic oxygen

requirement. Hypothermia also decreases drug metabolism, the MAC of all the volatile

anesthetics. Therefore, choice C is incorrect. The deleterious effects of hypothermia

include:

• Cardiac dysrhythmias

• Increased peripheral vascular resistance

• Coagulopathy

• Altered mental status

• Increased stress response and increased protein catabolism

1. Morgan GE, Mikhail MS, Murray MJ, (eds.). Clinical Anesthesiology. 3rd edition. Lange Medical Books/McGraw-Hill Medical Publishing Division. New York, NY. 2002:117.

Page 168: 2004 Anesth Practice Ques eBook

Anesthesia Practice Questions: Book 1 – 2004

This material may not be duplicated without written permission from the publisher. Copyright 2004, All Rights Reserved Medtext Medical World, Inc.

168

138. A An 80-year-old man with a history of coronary artery disease, profoundly hypotensive scheduled for emergency surgery for ruptured appendix Ketamine one of the inducing agents used for general anesthesia. It produces a dissociative

anesthesia, that is, it dissociates the sensory inputs from the thalamus to the limbic system.

Its half life is 10 – 15 minutes due to rapid redistribution. Ketamine increases arterial blood

pressure, heart rate and cardiac output due to central stimulation of sympathetic nervous

system and inhibition of reuptake of norepinephrine. It also causes an increase in

pulmonary artery pressure. For these reasons ketamine should be avoided in patients with

coronary artery disease, congestive heart failure or severe hypertension. In severely

hypotensive patients (i.e., shock) ketamine should be avoided because these patients have

depleted stores of catecholamine and ketamine itself has cardio-depressant effect.

Ketamine is a useful inducing agent for asthmatic patients (because of its bronchodilation

effects) and in patients with cardiac tamponade. It is contraindicated in patients with

increased ICP because it increases cerebral oxygen consumption and intracranial pressure.

1. Morgan GE, Mikhail MS, Murray MJ, (eds.). Clinical Anesthesiology. 3rd edition. Lange Medical Books/McGraw-Hill Medical Publishing Division. New York, NY. 2002:171. 139. E All of the above

Pentazocine Nalbuphine Buprenorphine Naloxone

Pentazocine acts as a weak antagonist or partial agonist at mu opioid receptors.

Pentazocine does not antagonize the respiratory depressant effects of morphine. However,

it may precipitate withdrawal when given to patients dependent on morphine or heroin.

High doses of pentazocine itself may cause respiratory depression with associated

increased blood pressure and tachycardia.

Nalbuphine is a mixed opioid i.e., it is an agonist-antagonist opioid. The antagonist effect of

nalbuphine is speculated to occur at mu receptors. This antagonist property at mu-

receptors can be used to reverse ventilatory depressant effects of opioids agonists while

still maintaining analgesia. Buprenorphine is a mixed opioid with agonist and antagonist

Page 169: 2004 Anesth Practice Ques eBook

Anesthesia Practice Questions: Book 1 – 2004

This material may not be duplicated without written permission from the publisher. Copyright 2004, All Rights Reserved Medtext Medical World, Inc.

169

properties. It has high analgesic properties. However, because of antagonism at mu-

receptors it can cause an acute withdrawal in patients addicted to heroin and morphine.

Naloxone is a pure antagonist and can precipitate acute withdrawal in heroin addicted

individuals. 1. Stoelting RK, (eds.) Pharmacology and Physiology in Anesthetic Practice. 3rd edition. J.B. Lippincott Company. Philadelphia, PA. 1999:104. 140. A Hyperkalemia and Verapamil toxicity Calcium is no better than placebo in promoting resuscitation and survival from asystole and

pulseless electrical activity. Calcium is not recommended during CPR unless specific

indications exist. It should be infused slowly because rapid infusion can produce cardiac

arrhythmias. Calcium is useful if hyperkalemia, symptomatic hypocalcemia or calcium

channel blocker toxicity is present. For administration calcium chloride is recommended

because it produces higher and more consistent levels of ionized calcium levels. Calcium is

not used for digoxin or tricyclic antidepressant toxicity. 1. Barash PG, Cullen BF, Stoelting RK, (eds.). Clinical Anesthesia. 4th edition. Lippincott and Wilkins. Philadelphia, PA. 2001:1500. 141. A Ketorolac has little potential for renal toxicity when fluid balance is adequate Ketorolac is a member of NSAID group with significant analgesic effects. Ketorolac 30 mg

IM produces analgesia that is equivalent to 10 mg of morphine and 100 mg of meperidine.

It is renal toxic in patients with hypovolemia, congestive heart failure or hepatic cirrhosis

who depend upon prostaglandins for preventing renal arteriolar constriction. It inhibits

platelet aggregation by inhibiting platelet cyclooxygenase activity. It produces its

antipyretic and antiinflammatory effects by inhibiting prostaglandin synthetase. 60% is

excreted unchanged in urine while the rest 40% is conjugated with glucuronide.

1. Stoelting RK, (eds.) Pharmacology and Physiology in Anesthetic Practice. 3rd edition. J.B. Lippincott. Philadelphia, PA. 1999:255-256.

Page 170: 2004 Anesth Practice Ques eBook

Anesthesia Practice Questions: Book 1 – 2004

This material may not be duplicated without written permission from the publisher. Copyright 2004, All Rights Reserved Medtext Medical World, Inc.

170

142. A Three times more potent than diazepam Midazolam is water-soluble and almost three times as potent as diazepam. The onset is

0.9-5 minutes. It is rapidly metabolized in liver. Hepatic clearance of midazolam is 10 times

that of diazepam (i.e., has significantly shorter half time). The volume of distribution and

lipid solubility of both the drugs are similar. Midazolam produces respiratory depression

similar to that of diazepam but a greater decrease in blood pressure and an increase in

heart rate than diazepam. Diazepam is water-insoluble. Therefore, it is dissolved in

propylene-glycol. Unlike midazolam IM injection of diazepam is painful.

1. Stoelting RK (eds.). Pharmacology and Physiology in Anesthetic Practice. 3rd edition. J.B. Lippincott. Philadelphia, PA. 1999: 130-132. 143. B Increased muscle weakness Most common cause of hypophosphatemia in an ICU patient is inadequate phosphorus

supplementation during hyperalimentation. Other causes include: diabetic ketoacidosis,

alcohol abuse, prolonged respiratory alkalosis and abuse of aluminum or magnesium

containing antacids. Severe hypophosphatemia is associated with wide spread organ

dysfunction, such as: cardiomyopathy, hemolysis, impaired leukocyte function, skeletal

muscle weakness and myopathy, rhabdomyolysis, respiratory failure and hepatic

dysfunction. During anesthesia neuromuscular function must be monitored and respiratory

alkalosis and hyperglycemia should be avoided. 1. Morgan GE, Mikhail MS, Murray MJ, (eds.) Clinical Anesthesiology. 3rd edition. Lange Medical Books/McGraw-Hill Publishing Division. New York, NY. 2002:622. 144. A Upper airway obstruction and carbon dioxide retention Pickwickian syndrome include: obesity hypoventilation syndrome due to decreased

ventilatory response to CO2 and oxygen resulting in sleep apnea, hypoxemia, hypercarbia,

pulmonary hypertension, polycythemia and finally biventricular failure. These patients are

particularly vulnerable during the post-operative period if opioids or other sedative have

been used. Both obese and patients with Pickwickian syndrome have oxygen demand and

Page 171: 2004 Anesth Practice Ques eBook

Anesthesia Practice Questions: Book 1 – 2004

This material may not be duplicated without written permission from the publisher. Copyright 2004, All Rights Reserved Medtext Medical World, Inc.

171

increased CO2 production due to increased metabolic rate. Expiratory reserve volume,

functional residual capacity and inspiratory reserve volume is greatly decreased in both

conditions. Forced expiratory volume (FEV1) and midexpiratory flow rate (MMEF) are

normal. Preoperatively these patients are increased risk for aspiration. Post-operatively

respiratory failure is the major concern. 1. Morgan GE, Mikhail MS, Murray MJ, (eds.). Clinical Anesthesiology. 3rd edition. Lange Medical Books/McGraw-Hill Publishing Division. NewYork, NY. 2002:748. 145. B Microcytic hypochromic anemia Cyclosporine selectively inhibits helper T-lymphocytes but has no effect on B-lymphocytes.

Cyclosporine is used as an immunosuppressant agent in organ transplant. It is also used in

patients suffering from Crohn’s disease, uveitis, psoriasis, rheumatoid arthritis. The most

common side effect or toxicity of cyclosporine include: nephrotoxicity, hypertension, nerve

paresthesia, headache, seizures, increased liver enzymes, cholestasis, hirsutism, gingival

hyperplasia, and hyperglycemia. Prolonged use can produce myocardial hypertrophy with

ST changes. Cyclosporine promotes tumor growth and can lead to significant increase in

tumor size. Microcytic, hypochromic anemia is due to iron deficiency not cyclosporine. 1. Stoelting RK. Pharmacology and Physiology in Anesthetic Practice. 3rd edition. Lippincott-Raven Publishers. New York, NY. 1999:418. 146. D Resetting discharge from aortic and carotid bodies Propofol, a substituted isoprophylphenol is one of the most common anesthetics inducing

agent used today. It affects cardiovascular system at multiple sites. It produces a decrease

in systematic blood pressure and cardiac output. This decrease is greater than that

produced by thiopental and is due to inhibition of sympathetic vasoconstriction. The cardio-

depressant effect is due to decrease in intracellular Ca ions and is more pronounced in

hypovolumic patients. Propofol produces bradycardia due to vagal stimulation. Rarely

bradycardia may be severe enough to produce asystole. Although propofol may depress

Page 172: 2004 Anesth Practice Ques eBook

Anesthesia Practice Questions: Book 1 – 2004

This material may not be duplicated without written permission from the publisher. Copyright 2004, All Rights Reserved Medtext Medical World, Inc.

172

baroreceptors control of heart rate it does not reset it. Propofol has depressant effects on

ventilation and central nervous system.

1. Stoelting RK. Pharmacology and Physiology in Anesthetic Practice. 3rd edition. Lippincott-Raven Publishers. New York, NY. 1999:143. 147. D Administer thiopental 250 mg intravenously All the choices given can reduce intracranial pressure but D is the most appropriate choice

for an immediate action. Lowering PaCO2 any further than 23 will not decrease the

intracranial pressure. Thiopental is administered to decrease ICP that remains elevated

despite deliberate hyperventilation and drug-induced diuresis. Thiopental decreases

cerebral volume by cerebral vascular vasoconstriction which also increases perfusion

pressure. Lasix decreases intracranial pressure by inducing systematic diuresis, decreasing

cerebrospinal fluid production and resolving cerebral edema by improving cellular water

transport. Mannitol decreases intracranial pressure by increasing plasma osmolarity, which

draws water from the tissues, including brain. Mannitol also decreases the rate of formation

of CSF. It takes 10-15 minutes before mannitol begins to exert its effect which last about 2

hours. An intact blood-brain barrier is necessary otherwise mannitol can enter the brain,

draw fluid with it and can produce rebound hypertension. A combination of Lasix and

mannitol is more effective. However, because of the emergent nature of this case,

administration of thiopental is the best choice.

1. Stoelting RK. Pharmacology and Physiology in Anesthetic Practice. 3rd edition. Lippincott-Raven. New York, NY. 1999:116-118; 438-440.

Page 173: 2004 Anesth Practice Ques eBook

Anesthesia Practice Questions: Book 1 – 2004

This material may not be duplicated without written permission from the publisher. Copyright 2004, All Rights Reserved Medtext Medical World, Inc.

173

148. C Administer diphenhydramine, cimetidine and fluid This patient has carcinoid tumor which produce carcinoid syndrome, Carcinoid syndrome is

the complex of signs and symptoms caused by secretion of vasoactive substances

(serotonin, kallikrein, histamine) from these tumors. Since most of these tumors are

located in gastrointestinal tract they are destroyed in the liver before they cause systematic

effects. Manual manipulation of the tumor can cause release of vasoactive substances and

multi-system signs and symptoms. These include: cutaneous flushing, changes in blood

pressure, dysrhythmias, bronchospasm, hypo- or hypertension. Of the choices given,

administration of diphenhydramine (histamine antagonist) and cimetidine (H2 receptor

antagonist) with fluids is the most appropriate choice. In general preoperative preparation

of these patients with drugs that block effects of vasoactive substances is indicated. One

such drug is Octreotide. Pretreatment with Octreotide, a synthetic somatostatin analogue

inhibits the release of vasoactive substances.

1. Morgan GE, Mikhail MS, Murray MJ, (eds.). Clinical Anesthesiology. 3rd edition. Lange Medical Books/McGraw-Hill Medical Publishing Division. New York, NY. 2002:749- 750.

Page 174: 2004 Anesth Practice Ques eBook
Page 175: 2004 Anesth Practice Ques eBook

K-Type Answers

Page 176: 2004 Anesth Practice Ques eBook
Page 177: 2004 Anesth Practice Ques eBook

Anesthesia Practice Questions: Book 1 – 2004 A=1,2,3 B=1,3 C=2,4 D=4 E=All

This material may not be duplicated without written permission from the publisher. Copyright 2004, All Rights Reserved Medtext Medical World, Inc. .

177

149. A (1,2,3) Isoflurane Sevoflurane Desflurane Xenon and nitrous oxide are non-volatile gases with anesthetic properties. Xenon is a noble

gas that has characteristic of an ideal anesthetic. Xenon has a MAC of 71%, making it more

potent than nitrous oxide (MAC 104%). It has a blood:gas partition coefficient of 0.14

which allows for a more rapid emergence. Xenon is nonexplosive, non-pungent, and

odorless. It is unreactive and produces minimal cardiac depression. A key attribute is that

unlike other inhalation anesthetics, including nitrous oxide, it is not harmful to the

environment, especially the ozone. It drawback is the high cost of using xenon. Xenon has

not been shown to be a trigger for malignant hyperthermia in animal studies.

Volatile anesthetics such as isoflurane, sevoflurane and desflurane have all been shown to

be triggers for malignant hyperthermia.

1. Xenon Anaesthesia, Journal of the Royal Society of Medicine, volume 93, Oct 2000:513-517. 150. A (1,2,3) Phenoxybenzamine Phentolamine Prazosin Pheochromocytoma is a catecholamine producing tumor of chromaffin tissue origin. Less

than 0.1% of all cases of hypertension are caused by pheochromocytomas. These tumors

are usually found in the adrenal glands but may found anywhere. These tumors secrete

excessive catecholamines such as norepinephrine and epinephrine.

Symptoms and signs associated with pheochromocytoma include paroxysmal headaches,

excessive sweating, palpitations, hypertension, hyperglycemia, and weight loss.

Catecholamine induced cardiomyopathy may also be present. Laboratory tests for urine

vanillylmandelic acid and catecholamines showed widely varied sensitivity and specificity,

neither of which can diagnose pheochromocytomas 100% of the time. This is due to the

intermittent production and secretion of catecholamines. In fact, presence of the triad of

Page 178: 2004 Anesth Practice Ques eBook

Anesthesia Practice Questions: Book 1 – 2004 A=1,2,3 B=1,3 C=2,4 D=4 E=All

This material may not be duplicated without written permission from the publisher. Copyright 2004, All Rights Reserved Medtext Medical World, Inc. .

178

symptoms such as paroxysmal headaches, sweating and hypertension, are more specific

and sensitive indicators than any of the above laboratory tests.

Preoperative medical treatment before surgery is generally recommended. The drugs most

commonly used are alpha-adrenergic antagonist such as phenoxybenzamine or prazosin.

Phenoxybenzamine is a non-competitive alpha adrenergic antagonist with mostly alpha-1

(postsynaptic) blocking activity and with some alpha-2 (presynaptic) blocking activity.

Treatment with phenoxybenzamine allows control of hypertension and slow intravascular

volume expansion by reducing alpha mediated vasoconstriction. Because it binds covalently

with alpha receptors, it is relatively long acting (24-48 hours). Prazosin on the other hand is

a shorter acting competitive agent also with mostly alpha-1 (postsynaptic) blocking activity.

Phentolamine is an alpha antagonist administered intravenously. It is an often used

intraoperatively to control hypertension resulting from manipulation of a

pheochromocytoma.

Phenylephrine is a direct alpha agonist and would not be indicated for a patient with a

pheochromocytoma.

1. Barash P, Cullen B, Stoelting R. Clinical Anesthesia, 4th edition, J.B. Lippincott. Philadelphia. 2001:1130-1133. 151. B (1,3) Sensitivity to non-depolarizing muscle relaxants Resistant to depolarizing muscle relaxants Myasthenia gravis is a autoimmune disease involving the neuromuscular junction. The

classic presentation is muscle weakness with exercise of central muscle along with ptosis.

Patients often undergo thymectomy for improvement of this disorder. Treatment also

includes the use of anticholinesterases such as pyridostigmine for improvement of

symptoms.

Page 179: 2004 Anesth Practice Ques eBook

Anesthesia Practice Questions: Book 1 – 2004 A=1,2,3 B=1,3 C=2,4 D=4 E=All

This material may not be duplicated without written permission from the publisher. Copyright 2004, All Rights Reserved Medtext Medical World, Inc. .

179

The untreated myasthenic patient is extremely sensitive to nondepolarizing muscle

relaxants; therefore normal doses should be reduced. On the other hand, these patients

show resistance to depolarizing muscle relaxants such as succinylcholine.

1. Barash P, Cullen B, Stoelting R. Clinical Anesthesia, 4th edition, J.B. Lippincott. Philadelphia. 2001:433. 152. B (1,3) Droperidol Metoclopramide Parkinson’s disease is a degenerative disease of the central nervous system characterized

by destruction of dopamine containing nerve cells in the substantia nigra of the basal

ganglia. The etiology of this destruction is most commonly idiopathic. The most

characteristic clinical features of Parkinson’s disease are resting tremor, cogwheel rigidity of

the extremities, bradykinesia, shuffling gait, stooped posture, and facial immobility. All are

secondary to diminished inhibition of the extrapyramidal motor system.

The vomiting center of the brain is located in the dorsal part of the lateral reticular

formation in the medulla. The vomiting center receives afferents from many different parts

of the body including the chemoreceptor trigger zone (CTZ). The CTZ has receptors for

serotonin, histamine, muscarinic compounds, and dopamine. Treatment for nausea and

vomiting is based on antagonizing the CTZ receptors sites. One of these receptors response

to dopamine and therefore is antagonized with an antidopaminergic medication such as

droperidol and metoclopramide. These drugs also have extrapyramidal side effects due to

this antidopaminergic property and therefore should be avoided in patients with Parkinson’s

disease as they could worsen this condition.

1. Stoelting, RK. Pharmacology and Physiology in Anesthetic Practice. 3rd edition. Lippincott-Raven. Philadelphia.1999:449-450,373-374.

Page 180: 2004 Anesth Practice Ques eBook

Anesthesia Practice Questions: Book 1 – 2004 A=1,2,3 B=1,3 C=2,4 D=4 E=All

This material may not be duplicated without written permission from the publisher. Copyright 2004, All Rights Reserved Medtext Medical World, Inc. .

180

153. A (1,2,3) Propofol Atropine Succinylcholine

Porphyria is inherited disorder involving enzymatic defects in the synthesis of heme. A key

step in heme synthesis is the conversion of succinyl-CoA with glycine to form aminolevulinic

acid (ALA). The enzyme responsible for this reaction is aminolevulinic acid synthetase (ALA

synthetase). The different forms of porphyrias involve enzymatic defects after this step.

Therefore, an excess of ALA in the urine is often helpful in diagnosis of porphyria.

Some drugs can induce ALA synthetase cause an excerebration of symptoms. These drugs

include barbiturates, phenytoin, glutethimide, ethanol, and ergot preparations. Therefore,

barbiturates such as sodium thiopental and methohexital are absolutely contraindicated in

these patients. All the other medications mentioned are safe to administer in patients with

porphyria.

1. Barash P, Cullen B, Stoelting R. Clinical Anesthesia, 4th edition, J.B. Lippincott. Philadelphia. 2001:540-541. 154. A (1,2,3) Hyperventilate Administer glucose plus insulin Administer sodium bicarbonate Hyperkalemia can occur because of increase intake (banked blood transfusion or rapid

infusion of potassium ion), cellular redistribution (acidosis, tissue necrosis, or

succinylcholine use), or decrease renal secretion (acute renal failure). Administering

glucose with insulin and sodium bicarbonate (or hyperventilation) causes the shift of

potassium from the extracellular fluid to the intracellular fluid. Calcium chloride although

used acutely to antagonize the effects of high levels of potassium on the myocardium, it

will not cause the shift of potassium away from the extracellular fluid.

1. Gravenstein N, Kirby RR. Complications in Anesthesiology. 2nd edition. Lippincott. Philadelphia. 1996:473-475.

Page 181: 2004 Anesth Practice Ques eBook

Anesthesia Practice Questions: Book 1 – 2004 A=1,2,3 B=1,3 C=2,4 D=4 E=All

This material may not be duplicated without written permission from the publisher. Copyright 2004, All Rights Reserved Medtext Medical World, Inc. .

181

155. A (1,2,3) Expiratory reserve volume Functional residual capacity Residual volume Pregnancy is associated with many physiologic changes. The most important respiratory

changes is a change in the functional residual capacity (FRC) which can be reduced by 20%

compared to non pregnant patients. This reduction is due to the elevation of the diaphragm

by the enlarging uterus into the abdominal cavity. The volumes that comprise FRC,

expiratory reserve volume and residual volume, are therefore reduced. Vital capacity

however remains unchanged throughout pregnancy.

1. Chestnut DH, Obstetric Anesthesia, Principal and Practice. 2nd edition. Mosby. St. Louis. 1997:18-19. 156. D (4) Osteoporosis in late stages Complex Regional Pain Syndrome Type I (CRPS I) is formally known as reflex sympathetic

dystrophy. CRPS I usually follow a noxious event that does not involve nerve injury or

follow a characteristic peripheral nerve distribution. CRPS II or causalgia has the same

signs and symptoms but occurs after a know nerve injury.

CRPS I presents with a triad of sensory, autonomic and motor signs and symptoms.

Sensory symptoms include pain described as burning and aching. Allodynia and

hyperalgesia is often present. Autonomic signs and symptoms almost always include

edema, skin color changes (erythema or cyanotic), and even changes in skin temperature

(higher or lower) when compared to the other limb. Motor signs and symptoms include

muscle weakness, spasm, and decrease range of motion. Other associated signs can

include trophic changes like increase or decrease nail or hair growth. Osteoporosis is

characteristic of late stages of the disease.

1. Srinivasa RN, et.al., Complex Regional Pain Syndrome I (Reflex Sympathetic Dystrophy), Anesthesiology, 2002:96:1254-1260.

Page 182: 2004 Anesth Practice Ques eBook

Anesthesia Practice Questions: Book 1 – 2004 A=1,2,3 B=1,3 C=2,4 D=4 E=All

This material may not be duplicated without written permission from the publisher. Copyright 2004, All Rights Reserved Medtext Medical World, Inc. .

182

157. B (1,3) Generalized pain in 3 or more sites for 3 months or longer Reproducible tenderness in 11 out of 18 pre specified sites Fibromyalgia is one of the most common causes of musculoskeletal pain. Its etiology is

unknown and it also includes a spectrum of other clinical symptoms such as sleep

disturbances, migraine headaches, irritable bowel syndrome and diffuse pain. Diagnosis is

base on the following criteria:

1. Generalized pain in 3 or more sites for 3 months or longer

2. Exclusion of other conditions that may cause similar symptoms

3. Reproducible tenderness in 11 out of 18 pre specified sites

Fibromyalgia does not have a sympathetic pain component and therefore a stellate

ganglion block or phentolamine infusion would not decrease the pain associated with

fibromyalgia. 1. Warfield, CA, Bajwa, ZH. Principles and Practice of Pain Medicine, 2nd edition, McGraw-Hill. New York NY. 2004:514-515. 158. A (1,2,3) Paroxysm of pain Unilateral facial involvement Trigger zones may be present Trigeminal neuralgia is a severe, usually unilateral facial pain involving the distribution of

cranial nerve V (most often divisions V2 and V3). Trigeminal neuralgia represents the most

frequent neuralgia seen in the elderly. The pain is often describes as sharp or stabbing pain

occurring is paroxysms and separated by pain free intervals. Trigger zones may be present

and light touching of these sites may elicit a paroxysm of pain.

1. Warfield, CA, Bajwa, ZH. Principles and Practice of Pain Medicine, 2nd edition, McGraw-Hill. New York NY. 2004:246-247.

Page 183: 2004 Anesth Practice Ques eBook

Anesthesia Practice Questions: Book 1 – 2004 A=1,2,3 B=1,3 C=2,4 D=4 E=All

This material may not be duplicated without written permission from the publisher. Copyright 2004, All Rights Reserved Medtext Medical World, Inc. .

183

159. A (1, 2, 3) Myosis Ptosis Nasal congestion The sympathetic innervation of the head, neck and arms originate from ganglia formed

from preganglionic nerve fibers emerging from T1and below. The ganglia of the head and

neck consist of three ganglia (the superior, middle and inferior cervical ganglion). All fibers

from to the superior and middle must pass through the inferior cervical ganglion (also

called the stellate ganglion). The stellate ganglion lies along the lateral aspect of the

seventh vertebrae just anterior to its transverse process. This places it behind the carotid

sheath and the vertebral artery, just above the pleura, and in close proximity to the

recurrent laryngeal nerve. Indication for a sympathetic block involving the stellate ganglion

is in sympathetically maintained pain such as complex regional pain syndrome esp. of the

upper extremity.

Signs of successful stellate ganglion block include Horner’s syndrome (ptosis, miosis, and

anhidrosis), injection of the conjunctiva, nasal stuffiness, vasodilation, and increased skin

temperature especially in the upper extremity.

Complications include block of the brachial plexus and recurrent laryngeal nerve or superior

laryngeal nerves, hematoma formation (from the carotid artery), intravascular injection

resulting in convulsions (injecting of even small amounts of local anesthetics via the

vertebral artery), and epidural and subarachnoid injections. Hoarseness would result with

blockade of the recurrent laryngeal nerve.

1. Warfield, CA, Bajwa, ZH. Principles and Practice of Pain Medicine, 2nd edition, McGraw-Hill. New York NY. 2004:696-698.

Page 184: 2004 Anesth Practice Ques eBook

Anesthesia Practice Questions: Book 1 – 2004 A=1,2,3 B=1,3 C=2,4 D=4 E=All

This material may not be duplicated without written permission from the publisher. Copyright 2004, All Rights Reserved Medtext Medical World, Inc. .

184

160. B (1,3) Cricoid cartilage Chassaignac’s tubercle Indication for a sympathetic block involving the stellate ganglion is in sympathetically

maintained pain such as complex regional pain syndrome esp. of the upper extremity. The

stellate ganglion lies along the lateral aspect of the seventh vertebrae just anterior to its

transverse process. This places it behind the carotid sheath and the vertebral artery, just

above the pleura, and in close proximity to the recurrent laryngeal nerve.

The anterior paratracheal approach is performed with the patient in the supine position.

The cricoid cartilage is palpated and then the fingers slide laterally in the groove of the

trachea and sternocleidomastoid muscle. The anterior tubercle of the transverse process of

C-6 (Chassaignac’s tubercle) is then located to perform the block.

The greater cornu of the hyoid bone is the landmark use to perform a superior laryngeal

block while C-3 transverse process is the landmark for a deep cervical plexus block.

1. Warfield, CA, Bajwa, ZH. Principles and Practice of Pain Medicine, 2nd edition, McGraw-Hill. New York NY. 2004:696-698. 161. A (1,2,3) 12th ribs T-12 spinous processes L-1 spinous processes Celiac block is indicated in patients with pain arising from the abdominal viscera, especially

in pancreatic cancer. Relief of pain is almost immediate. Celiac plexus block performed with

a neurolytic agent can provide relief of pain for up to 4 months. The celiac plexus is

generally clustered at the level of the body of L1, posterior to the vena cava on the right,

just lateral to the aorta on the left, and posterior to the pancreas.

The classic approach involves identifying the 12th ribs, and T-12 and L-1 spinous processes.

The sacral hiatus is a landmark for performing the sciatic nerve block.

1. Miller R. Anesthesia. 5th edition. Churchill Livingstone. New York NY. 2000:1545-1546.

Page 185: 2004 Anesth Practice Ques eBook

Anesthesia Practice Questions: Book 1 – 2004 A=1,2,3 B=1,3 C=2,4 D=4 E=All

This material may not be duplicated without written permission from the publisher. Copyright 2004, All Rights Reserved Medtext Medical World, Inc. .

185

162. C (2,4) Intercostobrachial nerve Musculocutaneous nerve The axillary approach to the brachial plexus is the most popular because of ease of block,

reliability of hand and forearm anesthesia, and safety. This block is ideally suited for

outpatients. At the level of the axillary, the musculocutaneous nerve has already left the

brachial plexus and travels within the coracobrachialis muscle. Supplementation of this

nerve is important since it provides sensory to the lateral portion of the forearm. The

intercostobrachial nerve is a branch from T-2 intercostal nerve and provides sensory to the

upper medial arm where if not supplemented will elicit pain when a tourniquet is used.

1. Miller R. Anesthesia. 5th edition. Churchill Livingstone. New York NY. 2000:1524-1527. 163. B (1,3) Spondylolysis Ankylosing spondylitis

Epidural steroid injection appears to be most therapeutic in patients experiencing low back,

neck or extremity pain secondary to nerve root irritation (radicular pain). The mechanism

appears to be from the steroid anti-inflammatory properties. Epidural steroid injection has

been found to be ineffective in spondylolysis and ankylosis spondylitis.

1. Warfield, CA, Bajwa, ZH. Principles and Practice of Pain Medicine, 2nd edition, McGraw-Hill. New York NY. 2004:655-660. 164. E All Clonidine Amitriptyline Gabapentin Morphine Complex Regional Pain Syndrome I (CRPS I) is the new term replacing reflex sympathetic

dystrophy (RSD) while CRPS II replaces the term for the syndrome causalgia. CRPS I

usually follow a noxious event that does not involve nerve injury or follow a characteristic

peripheral nerve distribution. CRPS II has the same clinical signs and symptoms but the

history is significant for a nerve injury.

Page 186: 2004 Anesth Practice Ques eBook

Anesthesia Practice Questions: Book 1 – 2004 A=1,2,3 B=1,3 C=2,4 D=4 E=All

This material may not be duplicated without written permission from the publisher. Copyright 2004, All Rights Reserved Medtext Medical World, Inc. .

186

Pharmacologic management include clonidine (α2 agonist) has been shown to be effective

especially when given by epidural or intrathecal. Amitriptyline (a serotonin and

norepinephrine reuptake blocking agent) is very effective in the treatment of neuropathic

pain such as CRPS. Gabapentin a selective voltage-gated Na2+ channel blocker has shown

efficacy in the treatment of CRPS as well. And finally, opioids such as morphine do have

role in neuropathic pain although they are most effective in nociceptive pain.

1. Warfield, CA, Bajwa, ZH. Principles and Practice of Pain Medicine, 2nd edition, McGraw-Hill. New York NY. 2004:411-412. 165. E All Hypotension Hypothermia Bradycardia Cardiac arrest Hypotension and bradycardia represents the most common serious complications

associated with spinal anesthesia. Blockade of the sympathetic efferents is the mechanism

for these cardiovascular effects. Hypotension is the result vasodilation and redistribution of

central blood volume to the periphery. This results in a drop in venous return and therefore

a decrease in cardiac output. The severity of hypotension is most times (not always)

associated with block height (T5 or greater). Other risk factors include age > 40 years,

starting baseline systolic blood pressure less than 120 mmHg, and spinal puncture above

L3 - L4.

Bradycardia, although less common than hypotension, is also the result of sympathetic

blockade and the risk also associated with block height (T5 or greater). Other risk factors

include a baseline heart rate less than 60 beats/min, ASA classification of I, use of beta

blockers, and pre-existing first degree block.

Hypotension and bradycardia can lead in rare instances to cardiac arrest. Lack of vigilance

and lack of early intervention has often been sighted as reasons contributing to cardiac

arrest.

Page 187: 2004 Anesth Practice Ques eBook

Anesthesia Practice Questions: Book 1 – 2004 A=1,2,3 B=1,3 C=2,4 D=4 E=All

This material may not be duplicated without written permission from the publisher. Copyright 2004, All Rights Reserved Medtext Medical World, Inc. .

187

Both general anesthesia and regional anesthesia result in impairment of thermoregulation.

Hypothermia is the most common thermoregulatory disturbance occurring with spinal

anesthesia. Again the mechanism associated with sympathetic blockade.

1. Spencer SL, et. Al. Current Issues in Spinal Anesthesia, Review Article. Anesthesiology. 2001:94:888-906. 2. Barash P, Cullen B, Stoelting R. Clinical Anesthesia, 4th edition, J.B. Lippincott. Philadelphia. 2001:704-706. 166. D (4) Baricity of local anesthetic It is the cephalad spread of local anesthetic in the cerebrospinal fluid (CSF) that determines

the height of spinal block. Many factors have been suggested to play a role in this spread,

but only baricity and patient position appear to be the most important. Hyperbaric local

anesthetic solutions tend to flow to depended areas, but by changing the position of the

patient, can influence the spread of the solution. Hypobaric solutions on the other hand

tend to spread more readily through the CSF. Increasing the dose and volume of

hyperbaric appears to be unimportant in predicting the spread of either hyperbaric or

isobaric solutions. Patient’s height is a poor predictor of the height of the block.

1. Barash P, Cullen B, Stoelting R. Clinical Anesthesia, 4th edition, J.B. Lippincott. Philadelphia. 2001:697-700. 167. E All Use of adrenergic agonist Type of local anesthetic Drug dose Block height The single most important factor determining the duration of block is the type of local

anesthetic used. Procaine has the shortest duration. Local anesthetics with intermediate

duration include lidocaine and mepivacaine, while those with the longest duration include

bupivacaine and tetracaine. Increasing the dose of local anesthetic also influences the

duration. Interesting the higher the height of the spinal block, the shorter the duration of

the block. The explanation for this is that for a given concentration, the greater the

cephalad spread, the lower the concentration of drug in the CSF and spinal nerve roots.

Page 188: 2004 Anesth Practice Ques eBook

Anesthesia Practice Questions: Book 1 – 2004 A=1,2,3 B=1,3 C=2,4 D=4 E=All

This material may not be duplicated without written permission from the publisher. Copyright 2004, All Rights Reserved Medtext Medical World, Inc. .

188

Finally, the use of adrenergic agonist such as epinephrine and phenylephrine are known to

increase the duration of the spinal block. The mechanism by which adrenergic agonists

prolong spinal block is not clear.

1. Barash P, Cullen B, Stoelting R. Clinical Anesthesia, 4th edition, J.B. Lippincott. Philadelphia. 2001:700-701. 168. A (1,2,3) Sex Age Use of opioids The ability to identify those at risk for postoperative nausea and vomiting narrows the pool

of those needing prophylactic antiemetic therapy. The incidence of PONV in patients with

no risk factors is approximately 10% while those with greater than three risk factors, can

increase the incidence to > 60%.

Patient related risk factors include female gender, history of PONV (or motion sickness) and

nonsmoking status. Anesthetic risk factors include the use of volatile anesthetics, nitrous

oxide and opioids perioperatively. Surgical risk factors are the length of the surgery and in

some cases the type of surgery (i.e., laparoscopy, neurosurgery, ear surgery, etc.).

A smoking history has been shown to be protective against PONV. 1. Gan TT, et. al., Consensus Guideline for Managing Postoperative Nausea and Vomiting. Anesthesia and Analgesia. 2003: 97: 62-71. 169. E All Droperidol Dolasetron Dexamethasone Ondansetron There are several antiemetic medications available for the prophylaxis treatment of PONV.

Among them are the serotonin antagonists such as dolasetron, ondansetron, granisetron

and tropisetron. All are most effective when given at the end of the surgery preferably 30

minutes before the end. Droperidol is also effective and also best given 30 minutes before

Page 189: 2004 Anesth Practice Ques eBook

Anesthesia Practice Questions: Book 1 – 2004 A=1,2,3 B=1,3 C=2,4 D=4 E=All

This material may not be duplicated without written permission from the publisher. Copyright 2004, All Rights Reserved Medtext Medical World, Inc. .

189

the end of the surgery. Dexamethasone (whose mechanism of action against PONV is

unknown) is also a very effective prophylactic antiemetic and is most effective when

administered at the beginning of the case rather than the end.

1. Gan TT, et. al., Consensus Guideline for Managing Postoperative Nausea and Vomiting. Anesthesia and Analgesia. 2003: 97: 62-71. 170. A (1,2,3) Gradual metabolism Redistribution Administration of cholinesterase inhibitors

With the exception of mivacurium (which is metabolized significantly by

pseudocholinesterase), reversal of blockade by nondepolarizing agents depends on

redistribution, gradual metabolism and excretion by the body, or administration of

cholinesterase inhibitors.

Duration of action of nondepolarizing drugs is determined by the time required for plasma

concentration to decrease below a critical level. As is the case for other drugs used in

anesthesia, the elimination half-life of neuromuscular blocking agents does not always

correlate with duration of action because termination of action sometimes depend on

redistribution instead of elimination.

The pharmacokinetic variables of vecuronium are similar to those of pancuronium. The

reason for vecuronium’s intermediate-duration of action is that the plasma concentration

decreases through the effective range far more rapidly so that duration and recovery

depend more on distribution than on elimination. However after prolonged infusion of

vecuronium (in excess of 6 h), the peripheral storage sites have become saturated and a

decrease in plasma concentration is then dependent upon metabolism and excretion not

upon redistribution.

Recovery after reversal is dependent on the rate of spontaneous recovery as well as the

acceleration induced by the reversal agent. 1. Neuromuscular blocking agents. In Morgan, Jr. GE, Mikhail MS, Murray MJ, Larson, Jr. CP (eds.). Clinical Anesthesiology. 3rd edition McGraw-Hill, 2002:182. 2. Bevan DR, Donati F. Muscle relaxants. In Barash PG, Cullen BF, Stoelting RK (eds.). Clinical Anesthesia. 4th edition Lippincott Williams & Wilkins, Philadelphia, PA. 2001:431,440.

Page 190: 2004 Anesth Practice Ques eBook

Anesthesia Practice Questions: Book 1 – 2004 A=1,2,3 B=1,3 C=2,4 D=4 E=All

This material may not be duplicated without written permission from the publisher. Copyright 2004, All Rights Reserved Medtext Medical World, Inc. .

190

171. A (1,2,3) The combination of Rocuronium and Cis-atracurium are synergistic Pre curarizing dose of D-Tubocurarine will shorten the duration of action of succinylcholine The potency of Rocuronium is enhanced when administered after succinylcholine

The duration of suxamethonium neuromuscular blockade following pancuronium

pretreatment has shown to be significantly prolonged (not shorter) due to the

cholinesterase inhibition caused by pancuronium.

Combinations of similar drugs - pancuronium/vecuronium (amino steroids),

atracurium/mivacurium (benzyl isoquinoline) - have additive effects. Other combinations

tend to show potentiation. Combinations of rocuronium (aminosteroid) and cisatracurium

(benzyl isoquinolinium) are synergistic.

In general, the depolarizing and non-depolarizing relaxants are mutually antagonistic and it

has been suggested that the dose of suxamethonium should be increased after

precurarization.

The duration of suxamethonium neuromuscular blockade following pancuronium

pretreatment has shown to be significantly prolonged due to the cholinesterase inhibition

caused by pancuronium. This effect is more marked with the smaller than the larger doses

of pancuronium, because at larger doses antagonistic action predominates.

Conversely, the potency of nondepolarizing drugs is enhanced when they are administered

after succinylcholine. The reason for this potentiation is not clear.

The response to a small dose of succinylcholine at the end of an anesthetic in which a

nondepolarizing agent has been used is difficult to predict. It may either antagonize or

potentiate the blockade, depending on the degree of nondepolarizing block. 1. Bevan DR, Donati F. Muscle relaxants. In Barash PG, Cullen BF, Stoelting RK (eds.). Clinical Anesthesia, 4th edition Lippincott Williams & Wilkins, Philadelphia, PA. 2001:432 2. Freund FG, Rubin AP. The need for additional succinylcholine after d-tubocurarine. Anesthesiology 1972;36:185-7 3. Stovner J, Oftedal N, Holmboe J. The inhibition of cholinesterases by pancuronium. Br. j. Anaesth. 1975;47:949-54 4. Krieg N, Hendrickx HHL, Crul JF: Influence of suxamethonium on the potency of ORG NC 45 in anesthetized patients. Br J Anaesth 1981;53:259

Page 191: 2004 Anesth Practice Ques eBook

Anesthesia Practice Questions: Book 1 – 2004 A=1,2,3 B=1,3 C=2,4 D=4 E=All

This material may not be duplicated without written permission from the publisher. Copyright 2004, All Rights Reserved Medtext Medical World, Inc. .

191

172. A (1,2,3) The response of the orbicularis oculi over the eyelid is similar to that of the adductor pollicis The response of the eyebrow (corrugator supercilii) is similar to that of the laryngeal adductors The onset of neuromuscular blockade occurs earlier at the diaphragm The dose response curve for the diaphragm is shifted to the right.

Muscles around the eye vary in their response to rocuronium. The response of the

superciliary arch (corrugator supercilii) reflects blockade of laryngeal adductor muscles.

However, the eyelid (orbicularis oculi) and thumb (adductor pollicis) have similar

sensitivities.

The dose required for 90% blockade has been reported to be 40-100% greater for the

diaphragm than for the adductor pollicis. The decreased sensitivity probably explains the

earlier recovery of the diaphragm. However, onset of neuromuscular blockade occurs

earlier at the diaphragm. This is believed to be due to the greater muscle blood flow

allowing earlier distribution and higher peak concentration of muscle relaxants at the

neuromuscular junction of the diaphragm as compared to the adductor pollicis.

1. Plaud B, Debaene B, Donati F. The corrugator supercilii,not the orbicularis oculi, reflects rocuronium neuromuscular blockade at the laryngeal muscles. Anesthesiology. 2001;95(1);96-101 2. Bevan DR, Donati F. Muscle relaxants. In Barash PG, Cullen BF, Stoelting RK (eds.). Clinical Anesthesia 4th edition Lippincott Williams & Wilkins, Philadelphia, PA. 2001:436 3. Lebreault C, Chauvin M, Guirimand F, Duvaldestin P: Relative potency of vecuronium on the diaphragm and the adductor pollicis. Br J Anaesth 1989; 63:389-392 173. E All Protection of the airway against regurgitation can be impaired Most patients complain of visual symptoms In the majority of patients, sustained bite is not present Grip strength can be decreased At TOF ratio of 0.7 most patients have signs and symptoms of significant paralysis.

In one study involving volunteers, at a TOF ratio of 0.70 (at the adductor pollicis) none of

the subjects found airway maintenance a problem, but all agreed that they were not “street

Page 192: 2004 Anesth Practice Ques eBook

Anesthesia Practice Questions: Book 1 – 2004 A=1,2,3 B=1,3 C=2,4 D=4 E=All

This material may not be duplicated without written permission from the publisher. Copyright 2004, All Rights Reserved Medtext Medical World, Inc. .

192

ready”. Although symptoms varied from subject to subject, TOFs in the range of 0.70-0.75

were associated with all of the following symptoms:

1. Diplopia and various visual disturbances (Most patients complain of visual

symptoms until TOF ratio was greater than 0.9)

2. Decreased grip strength

3. Inability to maintain incisor teeth apposition (In the majority of patients,

sustained bite does not return until the TOF ratio exceeds 0.85)

4. Inability to sit up without assistance

5. Severe facial weakness including an inability to make an airtight seal around a

drinking straw with the lips, speaking describes as a major effort

6. Overall weakness and tiredness

The results of these investigations indicate that normal respiratory and upper airway

function does not return to normal unless the TOF ratio at the adductor pollicis is 0.9 or

more.

1. Donati F, Meistelman C, Plaud B. Relationship of the TOF fade ratio to clinical signs and symptoms of residual paralysis in awake volunteers. Anesthesiology 1997;86:765-71 2. Bevan DR, Donati F. Muscle relaxants. In Barash PG, Cullen BF, Stoelting RK (eds.). Clinical Anesthesia. 4th edition Lippincott Williams & Wilkins, Philadelphia, PA. 2001:437 174. C (2,4) The post tetanic twitch count should not be repeated more often than every 5 minutes The most sensitive test for detecting residual paralysis is the ability to maintain sustained contractions to 100Hz of tetanus for 5 seconds TOF stimulation can be repeated after a pause of 10 seconds, not every 5 sec. The

presence of small number of impulses eliminates the problem of post-tetanic facilitation

and stimulation can be repeated after a pause of 10 seconds.

When assessing tactile/visual muscular activity in response to nerve stimulation, TOF and

double-burst stimulation can reliably detect fade when the TOF is <0.4 and 0.6,

Page 193: 2004 Anesth Practice Ques eBook

Anesthesia Practice Questions: Book 1 – 2004 A=1,2,3 B=1,3 C=2,4 D=4 E=All

This material may not be duplicated without written permission from the publisher. Copyright 2004, All Rights Reserved Medtext Medical World, Inc. .

193

respectively. On the other hand, studies have shown that at a TOF ratio of 0.7, most

subjects had significant signs and symptoms of residual paralysis.

It has been shown that 100Hz, 5–s tetanic nerve stimulation is a very good reference test

for detecting clinical fading at the adductor pollicis (AP), because a high level TOF ratio can

be detected. The absence of visual fading at the AP, elicited in anesthetized patents by

100-Hz, 5-s tetanus, is compatible with a TOF ratio > 0.85.

1. Bevan DR, Donati F. Muscle relaxants. In Barash PG, Cullen BF, Stoelting RK (eds.) Clinical Anesthesia. 4th edition Lippincott Williams & Wilkins, Philadelphia, PA. 2001:434 2. Viby-Mogensen J, Jensen NH, Engbaek J, et al. Tactile and visual evaluation of the response to train-of-four nerve stimulation. Anesthesiology 1985;63:440-3 3. Drenck NE, Ueda N, Olsen NV, et al. Manual evaluation of residual curarization using double-burst stimulation: a comparison with train-of-four. Anesthesiology1989;70:578-81 4. Kopman AF, Yee PS, Neuman GG. Relationship of the train-of-four fade ratio to clinical signs and symptoms of residual paralysis in awake volunteers. Anesthesiology 1997;86:765-71. 5. Baurain MJ, Hennart DA, Godschalx A, et al. Visual evaluation of residual curarization in anesthetized patients using one hundred-Hertz, five-second tetanic stimulation at the adductor pollicis muscle. Anesth analg 1998;87:185-9 175. B (1,3) May develop cardiac arrest following the administration of a depolarizing muscle relaxant Has a less intense block on the affected limb Monitoring of the affected side (not the unaffected) shows that the block is less intense and

recovery is more rapid than the unaffected side.

The patients with hemiplegia or quadriplegia as a result of central nervous system lesion

show an abnormal response to muscle relaxants. Hyperkalemia and cardiac arrest have

been described after succinylcholine, and probably related to potassium loss via a

proliferation of extra junctional receptors.

The hemiplegic patients are resistant to nondepolarizing muscle relaxants. Monitoring of

the affected side shows that the block is less intense and recovery is more rapid than the

Page 194: 2004 Anesth Practice Ques eBook

Anesthesia Practice Questions: Book 1 – 2004 A=1,2,3 B=1,3 C=2,4 D=4 E=All

This material may not be duplicated without written permission from the publisher. Copyright 2004, All Rights Reserved Medtext Medical World, Inc. .

194

unaffected side. Therefore, monitoring of the involved limb tends to underestimate the

block and the noninvolved limb should be used.

1. Bevan DR, Donati F. Muscle relaxants. In Barash PG, Cullen BF, Stoelting RK (eds): Clinical Anesthesia. 4th edition. Lippincott Williams & Wilkins, Philadelphia, PA. 2001:433, 437 176. B (1,3) Is functionally similar to Mapleson E and F systems Requires high fresh gas flows to prevent rebreathing of carbon dioxide Mapleson has a T-piece near the patient end, and is more efficient during controlled

ventilation. Mapleson A is the most efficient system during spontaneous ventilation.

The Mapleson D, E and F have a T piece near the patient end and they function similarly.

Mapleson D is the most efficient Mapleson system during controlled ventilation. To prevent

rebreathing CO2, the DEF systems require a fresh gas inflow rate of 2.5 times the minute

ventilation.

Dorsch JA, Dorsch SE. Understanding anesthesia equipment. 4th edition Williams & Wilkins, Baltimore, MD. 1999:211 177. B (1,3) Is located downstream from the nitrous oxide supply source Is located upstream of the flow control valves The fail-safe valve shuts interrupts the supply of nitrous oxide if the oxygen supply

pressure (not the oxygen flow) decreases. This device does not offer total protection

against a hypoxic mixture being delivered.

Fail-safe device is located downstream from the nitrous oxide supply and upstream of the

flow control valves. This valve shuts off or proportionately decreases and ultimately shuts

off nitrous oxide and other gases if the oxygen supply pressure (not the oxygen flow)

decreases. This device does not offer total protection against a hypoxic mixture being

delivered. The device aid in preventing hypoxia caused by some problems occurring

upstream of flow control valves only. These include disconnected oxygen hose and

Page 195: 2004 Anesth Practice Ques eBook

Anesthesia Practice Questions: Book 1 – 2004 A=1,2,3 B=1,3 C=2,4 D=4 E=All

This material may not be duplicated without written permission from the publisher. Copyright 2004, All Rights Reserved Medtext Medical World, Inc. .

195

depletion of oxygen cylinders. They do not guard against accidental crossovers in the

pipeline system. Use of an oxygen analyzer near the patient end is essential.

1. Andrews JJ, Brockwell RC. Delivery systems for inhaled anesthetics. In Barash PG, Cullen BF, Stoelting RK (eds.). Clinical Anesthesia. 4th edition. Lippincott Williams & Wilkins, Philadelphia, PA. 2001:567 2. Dorsch JA, Dorsch SE. Understanding anesthesia equipment. 4th edition. Williams & Wilkins. Baltimore, MD. 1999:95,96 178. E All Allergic reactions to local anesthetics remain uncommon despite their frequent use Muscle relaxants are the most frequently involved drugs Anaphylactoid reactions are considered to activate only basophils Increased concentration of tryptase (greater than 25 µg/L) suggest an anaphylactic reaction Among the drugs and other agents involved in anaphylaxis, muscle relaxants are still most

frequently involved. Rocuronium seems to be the most frequently involved agent (43.1%),

whereas the number of cases involving succinylcholine remains relatively stable (22.6%).

Anaphylaxis involves the activation of basophils and mast cell, whereas non-immune

mediated anaphylactoid reactions are considered to activate only basophils. Mast cells

activated during an IgE-mediated hypersensitivity reaction release proteases such as

tryptase, pre-stored histamine, and newly generated vasoactive mediators. Therefore,

increased concentration of tryptase greater than 25 µg/L is considered as a highly sensitive

indicator of anaphylactic reaction during anesthesia, although elevated tryptase can be

observed in other situation.

1. Laxenaire M, Mertes PM. Anaphylaxis during anesthesia: Results of a 2 year survey in France. Br J Anaesth 2001;21:549-58 2. Mertes PM, Laxenaire M, Alla F. Anaphylactic and anaphylactoid reactions occurring during anesthesia in France in 1999-2000. Anesthesiology 2003;99:536-45

Page 196: 2004 Anesth Practice Ques eBook

Anesthesia Practice Questions: Book 1 – 2004 A=1,2,3 B=1,3 C=2,4 D=4 E=All

This material may not be duplicated without written permission from the publisher. Copyright 2004, All Rights Reserved Medtext Medical World, Inc. .

196

179. B (1,3) Unidirectional valves should be close to the patient The pressure relief valve should be placed immediately before the absorber Unidirectional valves should be close to the patient to prevent back flow into the inspiratory

limb if a circuit leak develops.

The fresh gas inlet should be placed between the absorber and the inspiratory valve.

Positioning it downstream from the inspiratory valve would allow fresh gas to bypass the

patient during exhalation and be wasted. Fresh gas introduced between the expiration

valve and the absorber would be diluted by recirculating gas and also may be absorbed by

soda lime, thus slowing induction and emergence.

The pressure-relief valve should be placed immediately before the absorber to conserve

absorption capacity. The resistance to exhalation is decreased by locating the breathing

bag in the expiratory limb.

1. Breathing systems. In Morgan, Jr. GE, Mikhail MS, Murray MJ, Larson, Jr. CP (eds): Clinical Anesthesiology.3rd edition McGraw-Hill, 2002 ;35 180. B (1,3) Bronchospasm Increased inspiratory gas flow rate Peak inspiratory pressure (PIP) is the highest circuit pressure generated during an

inspiratory cycle, and provides an indication of dynamic compliance. Plateau pressure

(PP) is the pressure measured during an inspiratory pause (a time of no gas flow), and

mirrors static compliance. An increase in both PIP and PP implies an increase in tidal

volume or decrease in pulmonary compliance (i.e. pulmonary edema, pleural effusion,

tension pneumothorax, ascites)

An increase in PIP without any change in PP signals an increase in airway resistance

(bronchospasm, secretions) or inspiratory gas flow rate.

1. The anesthesia machine. In Morgan, Jr. GE, Mikhail MS, Murray MJ, Larson, Jr. CP (eds): Clinical Anesthesiology.3rd edition McGraw-Hill, 2002 ;47

Page 197: 2004 Anesth Practice Ques eBook

Anesthesia Practice Questions: Book 1 – 2004 A=1,2,3 B=1,3 C=2,4 D=4 E=All

This material may not be duplicated without written permission from the publisher. Copyright 2004, All Rights Reserved Medtext Medical World, Inc. .

197

181. A (1,2,3) Laryngospasm Laryngeal edema Foreign body aspiration The differential diagnosis of acute onset, post-operative inspiratory stridor includes:

1. Laryngospasm - involuntary spasm of the laryngeal musculature triggered by

blood or secretions stimulating the superior laryngeal nerve.

2. Laryngeal edema caused by an allergic reaction to drugs or traumatic intubation

3. Vocal cord dysfunction due to residual muscle relaxant effect

4. Hypocalcemic tetany

5. Vocal cord dysfunction due to bilateral recurrent laryngeal nerve palsy

Superior laryngeal nerve provides motor innervation to cricothyroid muscle (adductors of

the vocal cords) only and bilateral palsy results in hoarseness or easy tiring of the voice,

but airway control is not jeopardized.

Assuming an intact superior laryngeal nerve, acute bilateral recurrent laryngeal nerve palsy

can result in stridor and respiratory distress. This is due to the unopposed tension of the

cricothyroid muscle.

1. Anesthesia for otorhinolaryngologic surgery. In Morgan, Jr. GE, Mikhail MS, Murray MJ, Larson, Jr. CP (eds): Clinical Anesthesiology.3rd edition McGraw-Hill, 2002:780. 182. E All Incidence of radial artery thrombosis can be minimized by avoiding tapered catheters Arterial sampling is an important source of bacterial contamination of transducer systems Cerebral air embolism is a possible complication Systolic pressure reading at the radial artery exceeds that of the aortic root Arterial cannulation is regarded as an invasive procedure with a documented morbidity.

Ischemia, hemorrhage, thrombosis, embolism, cerebral embolism (retrograde flow

associated with flushing) aneurysm formation, arteriovenous fistula formation, skin

necrosis, and infection have occurred as a direct result of arterial cannulation, arterial blood

Page 198: 2004 Anesth Practice Ques eBook

Anesthesia Practice Questions: Book 1 – 2004 A=1,2,3 B=1,3 C=2,4 D=4 E=All

This material may not be duplicated without written permission from the publisher. Copyright 2004, All Rights Reserved Medtext Medical World, Inc. .

198

sampling, or high pressure flushing. Incidence of radial artery thrombosis can be minimized

by avoiding tapered catheters.

Because retrograde arterial embolism is possible whenever forceful flushing of a peripheral

arterial catheter is performed, special care should be exercised whenever more centrally

located arterial catheters are used.

Arterial sampling was found to be an important source of bacterial contamination.

Compared with central aortic pressure, peripheral waveform have higher systolic, lower

diastolic and wide pulse pressure. Despite this the MAP in the aorta is only slightly greater

than that in the radial artery. 1. Murphy GS, Vender JS Monitoring of the anesthetized patient. In Barash PG, Cullen BF, Stoelting RK (eds): Clinical Anesthesia. 4th edition. Lippincott Williams & Wilkins, Philadelphia, PA. 2001:674 2. Mark JB, Slaughter TF, Reves JG Cardiovascular Monitoring. In Miller RD (ed.): Anesthesia. 5th edition. Churchill Livingstone, Philadelphia, PA. 2000:1131 3. Shinozaki T, Dean RS, Mazuzan JE et al: Bacterial contamination of arterial lines: A prospective study. JAMA 1983;249:233 183. E All Are used intraoperatively to evaluate the functional integrity of sensory and motor pathways. Of brain stem origin (BAEP) are less vulnerable to anesthetic agents than the evoked potentials of cortical origin (cortical component of SSEP or VEP). Are minimally affected by opioids. Are minimally affected by N2O at concentrations of less than 50%. The sensitivity of evoked potentials to drug effects varies with the sensory modality being

monitored. Evoked potentials of cortical origin (i.e., the cortical component of the

somatosensory evoked potentials-SSEP and visual evoked potentials –VEP) are more

vulnerable to anesthetic influences than brain stem potentials (e.g., brain stem auditory

evoked potentials – BAEP and the subcortical components of the SSEP).

Because opioids preserve SEP recordings even in relatively high doses, they are

recommended for use as infusions during intraoperative monitoring.

Page 199: 2004 Anesth Practice Ques eBook

Anesthesia Practice Questions: Book 1 – 2004 A=1,2,3 B=1,3 C=2,4 D=4 E=All

This material may not be duplicated without written permission from the publisher. Copyright 2004, All Rights Reserved Medtext Medical World, Inc. .

199

Motor evoked potentials (MEPs) are extremely sensitive to depression by anesthetics.

However, N2O concentration of 50% is reported to cause minimal response alteration and

appears to be compatible with monitoring.

1. Bendo AA, Kass IS, Hartung J, Cottrell JE. Anesthesia for Neurosurgery. In Barash PG, Cullen BF, Stoelting RK (eds): Clinical Anesthesia. 4th edition. Lippincott Williams & Wilkins, Philadelphia, PA. 2001:754,757. 184. A (1,2,3) Is a synthetic opioid Is a selective µ-opioid receptor agonist Has an elimination half-life of about 9 minutes Remifentanil is a selective mu opioid agonist with an analgesic potency similar to that of

fentanyl. Although chemically related to the fentanyl family of short acting phenylpiperidine

derivatives, remifentanil is structurally unique because of its ester linkage. This ester

structure of remifentanil renders it susceptible to hydrolysis by nonspecific plasma and

tissue esterases to inactive metabolites. Because pseudocholinesterase does not appear to

metabolize remifentanil, plasma cholinesterase deficiency and anticholinergic administration

are not expected to affect remifentanil clearance.

1. Coda BA. Opioids. In Barash PG, Cullen BF, Stoelting RK (eds): Clinical Anesthesia. 4th edition. Lippincott Williams & Wilkins, Philadelphia, PA. 2001:366 2. Manullang J, Egan TD: Remifentanil’s effect is not prolonged in a patient with pseudocholinesterase deficiency. Anesth Analg 1999;89:529-530 3. Opioid Agonists and Antagonists. In Stoelting RK (ed) Pharmacology and Physiology in Anesthetic Practice. 3rd edition. Lippincott Williams & Wilkins, Philadelphia, PA. 1999;100. 185. A (1,2,3) Does not suppress myocardial contractility at clinical doses

When administered orally, morphine-6-glucuronide (M6G) is the primary active compound

Has a bioavailability of <30% when administered orally At clinically relevant doses, morphine does not suppress myocardial contractility.

Morphine induced histamine release is not prevented by pretreatment with naloxone,

suggesting that histamine release is not mediated by opioid receptors.

Page 200: 2004 Anesth Practice Ques eBook

Anesthesia Practice Questions: Book 1 – 2004 A=1,2,3 B=1,3 C=2,4 D=4 E=All

This material may not be duplicated without written permission from the publisher. Copyright 2004, All Rights Reserved Medtext Medical World, Inc. .

200

Morphine-3-glucuronide (M3G) is the major metabolite of morphine, but it does not bind to

opioid receptors and has little or no analgesic activity. Morphine-6-glucuronide (M6G),

which accounts for nearly 10% of the morphine’s metabolism, is a more potent mu

receptor agonist than morphine with a similar duration of action. M6G contributes

substantially to the analgesic effects of morphine even in patients with normal renal

function and appears to have a more favorable side effect profile than morphine.

The bioavailability of orally administered morphine is significantly lower (20-30%) than

after IM or subcutaneous injection, because of its high hepatic extraction ratio. The high

hepatic first pass effect also result in substantial level of M6G when morphine is

administered orally. It appears that M6G is in fact the primary active compound when

morphine is administered orally.

1. Coda BA. Opioids. In Barash PG, Cullen BF, Stoelting RK (eds): Clinical Anesthesia. 4th edition. Lippincott Williams & Wilkins, Philadelphia, PA. 2001:351. 2. Bailey PL, Egan TD, Stanley TH. Intravenous Opioid Anesthesia. In Miller RD (ed.): Anesthesia. 5th edition. Churchill Livingstone, Philadelphia, PA. 2000:313. 3. Hanna MH, Peat SJ, Knibb AA et al: Disposition of morphine-6-glucuronide and morphine in healthy volunteers. Br J Anaesth. 1991;66:103-7. 186. C (2,4) Can be decreased or prevented by pretreatment with muscle relaxants Is mu receptor mediated Although the precise mechanism of opioid induced muscle rigidity is not clearly understood,

it is not due to a direct action on muscle fibers. It can be decreased or prevented by

pretreatment with muscle relaxants and is not associated with increase in creatinine kinase.

This suggests that there is little or no muscle damage. Mu receptor agonists induced

rigidity in the rat, whereas delta and kappa agonists did not.

1. Bailey PL, Egan TD, Stanley TH. Intravenous Opioid Anesthesia. In Miller RD (ed.): Anesthesia. 5th edition. Churchill Livingstone, Philadelphia, PA

Page 201: 2004 Anesth Practice Ques eBook

Anesthesia Practice Questions: Book 1 – 2004 A=1,2,3 B=1,3 C=2,4 D=4 E=All

This material may not be duplicated without written permission from the publisher. Copyright 2004, All Rights Reserved Medtext Medical World, Inc. .

201

187. B (1,3) Is a more potent mu receptor agonist than morphine Has duration of action similar to that of morphine Morphine-3-glucuronide (M3G) is the major metabolite of morphine, but it does not bind to

opioid receptors and has little or no analgesic activity. Morphine-6-glucuronide (M6G),

which accounts for nearly 10% of the morphine’s metabolism, is a more potent mu

receptor agonist than morphine with a similar duration of action.

M6G contributes substantially to the analgesic effects of morphine even in patients with

normal renal function and appears to have a more favorable side effect profile than

morphine. It appears that M6G is in fact the primary active compound when morphine is

administered orally.

1. Bailey PL, Egan TD, Stanley TH. Intravenous Opioid Anesthesia. In Miller RD (ed.): Anesthesia. 5th edition. Churchill Livingstone, Philadelphia, PA. 2000:313 2. Hanna MH, Peat SJ, Knibb AA et al: Disposition of morphine-6-glucuronide and morphine in healthy volunteers. Br J Anaesth. 1991;66:103-7 188. A (1,2,3) Is a selective antagonist at 5-HT3 receptors May cause prolongation of the QT interval Is effective in the treatment of perioperative nausea and vomiting Ondansetron is structurally related to serotonin and posses specific serotonin (5-HT3)

subtype receptor antagonist properties, without altering dopamine, histamine, adrenergic,

or cholinergic receptor activity. Ondansetron, 4 to 8 mg IV (administered over 2 to 5

minutes immediately before induction of anesthesia), is highly effective in decreasing the

incidence of postoperative nausea and vomiting in a susceptible patient population

(ambulatory gynecologic surgery, middle ear surgery). In addition to prophylaxis,

ondansetron, 1 to 8 mg IV is highly effective in the treatment of postoperative nausea and

vomiting.

Page 202: 2004 Anesth Practice Ques eBook

Anesthesia Practice Questions: Book 1 – 2004 A=1,2,3 B=1,3 C=2,4 D=4 E=All

This material may not be duplicated without written permission from the publisher. Copyright 2004, All Rights Reserved Medtext Medical World, Inc. .

202

All the 5 HT3 receptor antagonists can prolong the QT interval and can produce

arrhythmias. Unlike metoclopramide, these agents do not affect GI motility or lower

esophageal sphincter tone.

1. White PF, Droperidol: a cost-effective antiemetic for over thirty years. Anesth Analg 2002;95:789-90 2. Adjuncts to anesthesia. In Morgan, Jr. GE, Mikhail MS, Murray MJ, Larson, Jr. CP (eds): Clinical Anesthesiology.3rd edition McGraw-Hill, 2002 ;247 3. Non barbiturate induction drugs. In Stoelting RK (ed) Pharmacology and Physiology in Anesthetic Practice. 3rd edition. Lippincott Williams & Wilkins, Philadelphia, PA. 1999;406. 189. A (1,2,3) Is a highly selective alpha-2 –adrenergic agonist Frequently causes dry mouth Causes bradycardia Medetomidine is a highly selective alpha-2 – adrenergic agonist and dexmedetomidine is its

specific stereoisomer.

A frequently reported side effect of dexmedetomidine has been a dry mouth, which is due

a decrease in saliva production.

The basic side effects of alpha-2 – adrenergic agonists on the cardiovascular system are

decreased heart rate, decreased systemic vascular resistance and indirectly decreased

myocardial contractility, cardiac output, and systemic blood pressure. Intravenous

dexmedetomidine has a biphasic effect on blood pressure. There is an initial increase in

blood pressure due to its effect on peripheral alpha-2 receptors.

Dexmedetomidine reduces the MAC of the potent volatile anesthetics by 30-50%.

1. Reves JG, Glass PSA, Lubarsky DA. Nonbarbiturate Intravenous Anesthetics. In Miller RD (ed.): Anesthesia. 5th edition. Churchill Livingstone, Philadelphia, PA. 2000:259-261.

Page 203: 2004 Anesth Practice Ques eBook

Anesthesia Practice Questions: Book 1 – 2004 A=1,2,3 B=1,3 C=2,4 D=4 E=All

This material may not be duplicated without written permission from the publisher. Copyright 2004, All Rights Reserved Medtext Medical World, Inc. .

203

190. A (1,2,3) Has minimal intrinsic activity When used to reverse Diazepam, the potential for re-sedation exists Can precipitate withdrawal symptoms in patients physically dependent on benzodiazepines According to the pharmacological studies, flumazenil is a benzodiazepine receptor ligand

with high affinity, great specificity, by definition minimal intrinsic activity.

Flumazenil, because of its high-affinity to benzodiazepine receptor, will replace relatively

weak agonists such as diazepam as long as it is given in sufficient dose. Since flumazenil is

cleared relatively rapidly, (plasma half life one hour) the receptors occupied by agonist will

increase overtime, and thus potential for resedation exist. This is less likely with

midazolam, since it has a rapid clearance also.

Flumazenil can precipitate withdrawal symptoms in humans physically dependent on a

benzodiazepine receptor agonist.

1. Reves JG, Glass PSA, Lubarsky DA Nonbarbiturate Intravenous Anesthetics. In Miller RD (ed.): Anesthesia. 5th edition. Churchill Livingstone, Philadelphia, PA. 2000:237-239 191. B (1,3) Is a synthetic colloid solution Is degraded by amylase Hydroxyethyl starch (HES) compounds are a group of polydispersed synthetic colloids that

resemble glycogen structurally. HESs are synthesized from amylopectin, a waxy starch

derived from maize or sorghum. Amylopectin is a D-glucose polymer with a branching

structure. Reaction with ethylene oxide in the presence of an alkaline catalyst results in

hydroxyethyl substitution. The unsubstituted starch is rapidly hydrolyzed by nonspecific

alpha-amylase in the plasma, and substitution with hydroxyethyl groups substantially slows

this process.

Dextran, (not Hydroxyethyl starch) interferes with the cross matching due to rouleaux

formation.

Page 204: 2004 Anesth Practice Ques eBook

Anesthesia Practice Questions: Book 1 – 2004 A=1,2,3 B=1,3 C=2,4 D=4 E=All

This material may not be duplicated without written permission from the publisher. Copyright 2004, All Rights Reserved Medtext Medical World, Inc. .

204

There is minimal interference with subsequent cross matching with hydroxyethyl and

allergic reactions are very rare.

1. McKinlay S, Gan TJ. Intraoperative Fluid Management and Choice of Fluids. ASA Refresher Course. Lippincott Williams & Wilkins, Philadelphia, PA. 2003:130-131. 2. Kaye AD, Grogono AW. Fluid and Electrolyte Physiology. In Miller RD (ed.): Anesthesia. 5th edition. Churchill Livingstone, Philadelphia, PA. 2000:1604 192. A (1,2,3) High concentration of volatile anesthetics may virtually obliterate the hypercarbia-induced increase in ventilatory drive. All volatile anesthetics decrease tidal volume. Most volatile agents depress the ventilatory response to hypoxia at subanesthetic concentrations. All inhaled anesthetics dose dependently depress the ventilatory response to hypercarbia

and high concentrations may virtually obliterate the response.

In general, all volatile agents decrease tidal volume. However, the resultant depression of

minute ventilation may be partially offset by a concomitant increase in respiratory rate.

Subanesthetic concentrations of most volatile anesthetics depress the ventilatory response

to hypoxia in normocapnic, quiet resting subjects, but desflurane does appear to be an

exception. However, it has been shown that subanesthetic concentrations of desflurane do

decrease hypoxic sensitivity during concomitant hypercapnia, suggesting an effect at the

peripheral chemoreceptors.

All volatile anesthetics significantly reduces respiratory system resistance, equi-MAC

sevoflurane decreased resistance as much as halothane and more so than isoflurane.

1. Farber NE, Pagel PS, Warltier DC.. Pulmonary Pharmacology. In Miller RD (ed.): Anesthesia. 5th edition. Churchill Livingstone, Philadelphia, PA. 2000:128-142 2. Dahan A, Sarton E et al: Ventilatory response to hypoxia in humans: influences of subanesthetic desflurane. Anesthesiology 1996;85:60-68

Page 205: 2004 Anesth Practice Ques eBook

Anesthesia Practice Questions: Book 1 – 2004 A=1,2,3 B=1,3 C=2,4 D=4 E=All

This material may not be duplicated without written permission from the publisher. Copyright 2004, All Rights Reserved Medtext Medical World, Inc. .

205

193. C (2,4) Causes subacute combined degeneration of the spinal cord only after several months of daily exposure. Causes very low methionine synthetase activity after a few hours of anesthesia. Nitrous oxide (N2O) is the only anesthetic reported to produce hematological and

neurological toxicity with long-term administration. Both effects are due to interaction of

N2O with vitamin B12 and irreversible oxidation of the cobalt in vitamin B12 by a

physicochemical reaction. Methionine synthetase and thymidylate synthetase are vitamin

B12 dependent enzymes and require vitamin B12 in the completely reduced form to act as

its coenzyme.

These two enzymes are involved in the formation of myelin and the formation of DNA and

therefore, inhibition of these manifests as neurologic disturbances and depression of bone

marrow function.

The half time for (irreversible) inactivation of methionine synthetase is shown to be 46 min

when 70% N2O is administered to patients.

In healthy patients megaloblastic changes are seen after about 12 hour exposure to 50%

N2O. The neurological disease (subacute combined degeneration of the spinal cord)

develops only after several months of daily exposure to N2O.

Experimental data suggest that there is a threshold concentration of about 1,000 ppm

(0.1%) below which N2O has no biochemical effect.

1. Baden JM, Rice SA Metabolism and toxicity of inhaled Anesthetics. In Miller RD (ed.): Anesthesia. 5th edition. Churchill Livingstone, Philadelphia, PA. 2000:162-163 2. Ebert TJ, Schmid PG in Inhalation Anesthesia. In Barash PG, Cullen BF, Stoelting RK (eds): Clinical Anesthesia. 4th edition. Lippincott Williams & Wilkins, Philadelphia, PA. 2001:402-403 3. Royston BD, Nunn JF, Weinbren HK et al: Rate of inactivation of human and rodent hepatic methionine synthase by nitrous oxide. Anesthesiology1988;68:213-216 4. Sharer NM, Nunn JF, Royston JP et al: Effects of chronic exposure to nitrous oxide on methionine synthetase activity. Br j Anaesth 1983;5:693-701

Page 206: 2004 Anesth Practice Ques eBook

Anesthesia Practice Questions: Book 1 – 2004 A=1,2,3 B=1,3 C=2,4 D=4 E=All

This material may not be duplicated without written permission from the publisher. Copyright 2004, All Rights Reserved Medtext Medical World, Inc. .

206

194. D (4) Expansion of air filled spaces Volatile anesthetics produce a dose-dependent decrease in uterine smooth muscle

contractility, but nitrous oxide has no such effect.

There is a small increase in pulmonary vascular resistance with nitrous oxide, which is more

pronounced in patients with pulmonary hypertension.

There is increase in sympathetic nervous system activity with nitrous oxide, which offset

the direct negative inotropic action on the heart.

Nitrous oxide expands air filled spaces because of its greater solubility in blood compared

to nitrogen (i.e., 34 times more soluble in blood than nitrogen). Therefore, nitrous oxide

tends to diffuse into air-containing cavities more rapidly than nitrogen is absorbed by the

blood stream.

1. Pagel PS, Farber NE, Warltier DC. Cardiovascular Pharmacology. In Miller RD (ed.): Anesthesia. 5th edition. Churchill Livingstone, Philadelphia, PA. 2000:96-116 2. Ebert TJ, Schmid PG in Inhalation Anesthesia. In Barash PG, Cullen BF, Stoelting RK (eds): Clinical Anesthesia. 4th edition. Lippincott Williams & Wilkins, Philadelphia, PA. 2001:402-403 195. A (1,2,3) Dryness of the carbon dioxide absorbant High temperature of the carbon dioxide absorbant Prolonged high fresh gas flows Carbon monoxide formation occurs with degradation of volatile agents containing a CHF2

moiety by the strong bases in carbon dioxide absorbants. Carbon monoxide production is

influenced by:

a) Dryness of the absorbent with hydration preventing formation

b) High temperature of the absorbant such as during low fresh gas flow and

increased metabolic production of carbon dioxide

c) Prolonged high fresh gas flow causing dryness of the absorbant

d) Type of absorbent (Baralyme>soda lime)

1. Inhaled Anesthetics in Stoelting RK (editor) Physiology & Pharmacology in Anesthetic Practice. Third edition. Lippincott Williams & Wilkins, Philadelphia, PA. 1999:71

Page 207: 2004 Anesth Practice Ques eBook

Anesthesia Practice Questions: Book 1 – 2004 A=1,2,3 B=1,3 C=2,4 D=4 E=All

This material may not be duplicated without written permission from the publisher. Copyright 2004, All Rights Reserved Medtext Medical World, Inc. .

207

196. B (1,3) Causes formation of organic and inorganic fluoride metabolites. Produces peak plasma fluoride concentrations that are higher than after comparable doses of enflurane. 5% of absorbed sevoflurane undergoes oxidative metabolism to form organic and inorganic

fluoride metabolites.

Sevoflurane does not undergo metabolism to acetyl halide that could result in formation of

trifluoroacetylated liver proteins. Therefore, sevoflurane cannot stimulate the formation of

antitrifluoroacetylated protein antibodies leading to hepatotoxicity by this mechanism.

Peak plasma concentrations are higher than after comparable dose of enflurane. Because

of the limited exposure of renal tubules to fluoride due to rapid elimination of sevoflurane,

there is a less of a nephrotoxic risk. Furthermore hepatic production of fluoride from

sevoflurane may be less of a nephrotoxic risk than is intrarenal production of fluoride from

enflurane.

Carbon monoxide formation reflects the degradation of volatile agents containing a CHF2-

moiety- namely desflurane, enflurane and isoflurane, (not sevoflurane) by strong bases

present in carbon dioxide absorbents.

1. Inhaled Anesthetics in Stoelting RK (editor) Physiology & Pharmacology in Anesthetic Practice. Third edition. Lippincott Williams & Wilkins, Philadelphia, PA. 1999:71 197. A (1,2,3) Is primarily a result of inferior vena caval occlusion May reduce the blood flow to the uterus by as much as 45% Causes approximately 1 in 10 pregnant women to become symptomatic Supine hypotension syndrome is primarily a result of inferior vena caval occlusion, but

aortic compression also contributes to it in some. Flow to the uterus (and lower limbs) may

be reduced by as much as 45% without systemic signs or evidence of aortic compression.

The effect on the venous return depends on the efficiency of the collateral azygous and

intervertebral venous plexuses. Approximately 1 in 10 pregnant women become

symptomatic, with pallor, sweating, nausea and hypotension, accompanied by bradycardia

Page 208: 2004 Anesth Practice Ques eBook

Anesthesia Practice Questions: Book 1 – 2004 A=1,2,3 B=1,3 C=2,4 D=4 E=All

This material may not be duplicated without written permission from the publisher. Copyright 2004, All Rights Reserved Medtext Medical World, Inc. .

208

and fall in cerebral blood flow and unconsciousness. The Trendelenburg position may

actually worsen aortocaval compression.

1. Kerr MG, Scott DB, Samuel E. Studies of the inferior vena cava in late pregnancy: BMJ 1964;1:532-3 2. Kinsella SM, Lee A, Spencer JA: Maternal and fetal effects of the supine and pelvic tit positions in late pregnancy. Eur J Obstet Gynecol Reprod Biol 1990;36:11 3. Ross A. Physiological changes of pregnancy. In Birnbach DJ, Gatt SP, Datta S (eds): Textbook of Obstetric Anesthesia .Churchill Livingstone Philadelphia, PA. 2000:35 198. C (2,4) Decrease in the colloid oncotic pressure Threefold elevation of alkaline phosphatase Physiological changes of pregnancy do not cause changes in the central venous pressure.

Hemoglobin increases at a slower rate than plasma volume and accounts for the relative

anemia of pregnancy.

There is three-fold elevation of alkaline phosphatase (mainly placental origin).

Colloid osmotic pressure decreases in pregnancy, chiefly following a decrease in albumin

level. The precise etiology remains obscure Albumin level falls to a minimum immediately

after delivery. Although the incidence of pulmonary edema remains low after both cesarean

and vaginal delivery, excess intravenous hydration is poorly tolerated.

1. Ross A. Physiological Changes of Pregnancy. In Birnbach DJ, Gatt SP, Datta S.(eds) Text book of Obstetric Anesthesia. Churchill Livingstone, Philadelphia, PA. 2000:34-39.

Page 209: 2004 Anesth Practice Ques eBook

Anesthesia Practice Questions: Book 1 – 2004 A=1,2,3 B=1,3 C=2,4 D=4 E=All

This material may not be duplicated without written permission from the publisher. Copyright 2004, All Rights Reserved Medtext Medical World, Inc. .

209

199. C (2,4) Resolves spontaneously within several days Has higher incidence with lidocaine spinal and lithotomy position Prospective, randomized, controlled studies reveal up to a 40% incidence of pain radiating

from the lower back to the buttocks or lower extremities after lidocaine spinal anesthesia.

These symptoms have been labeled transient neurological symptoms (TNS) and also have

been reported with other local anesthetics, but it is clear that the incidence of TNS is

greater after lidocaine spinal anesthesia. Patients typically manifest symptoms of TNS

within 12 to 24 h after surgery. The pain is characterized by back pain radiating to the legs

without sensory or motor deficits, occurring after the resolution of spinal block and

resolving spontaneously within several days. Clinical studies have identified the use of

lidocaine, lithotomy position, ambulatory surgical status, arthroscopic knee surgery, and

obesity are factors that place patients at increased risk for development of TNS.

1. Salinas FV, Liu SL, Schloz AM. Analgesics. In Evers AS, Maze M (Eds) Anesthetic pharmacology. Churchill Livingstone. Philadelphia, PA. 2004:526-7. 200. E All Clopidogrel should be discontinued for 7 days before a neuraxial anesthesia. Aspirin may be safely used in patients having epidural or spinal injections. Subcutaneous heparin does not appear to increase the risk of spinal hematoma. Post surgical prophylactic dose of low molecular weight heparin (LMWH) may be started 2-4 hours after removal of epidural catheter. It is now recommended that clopidogrel be discontinued for 7 days before a neuraxial

anesthesia or analgesia. (one case reports of epidural hematoma following clopidogrel)

Aspirin can be safely used with neuraxial injections, but caution should be observed in

patients on multiple antiplatelet medications.

Subcutaneous heparin does not appear to increase the risk of spinal hematoma, and the

risk may be further reduced by administering heparin at least one hour before the block.

Page 210: 2004 Anesth Practice Ques eBook

Anesthesia Practice Questions: Book 1 – 2004 A=1,2,3 B=1,3 C=2,4 D=4 E=All

This material may not be duplicated without written permission from the publisher. Copyright 2004, All Rights Reserved Medtext Medical World, Inc. .

210

When prophylactic doses (30 mg BID) of low molecular weight heparin is employed

(LMWH), neuraxial blocks or catheter insertion or removal should not be performed for 12

hours. This period is increased to 24 hours when therapeutic doses of LMWH (1 mg/kg) are

employed. LMWH may be started 2-4 hours after epidural analgesia is discontinued.

1. Benzon HT< Wong HY, Siddiqui T, et al: Caution in performing epidural injections in patients on several antiplatelet drugs. Anesthesiology 1999;91:1558-1559. 2. The American Society of Regional Anesthesia and pain Medicine Consensus Conference. Regional anesthesia in the anticoagulated patient-defining the risks.2002; www.asra.com. 201. A (1,2,3) May be difficult to differentiate from chickenpox during the first 2-3 days May cause death due to toxemia and overwhelming pneumonia Vaccine consists of live attenuated vaccinia virus Differentiation between chickenpox and smallpox can be difficult during the first 2-3 days.

The most reliable differentiation is that the vesicles of smallpox are all at the same stage on

all parts of the body, while they are at different stages on different parts of the body in

chickenpox. Death may occur for hypotension, toxemia and severe pneumonia. Vaccination

within 3 days of exposure will prevent or significantly lessen the severity of the disease.

The vaccine, which consist of live attenuated vaccinia virus, may cause life threatening side

effects in 14 to 52 people per million. Vaccine offers elevated protective immunity for 5

years and diminishing effects over the next 15 years.

1. Breman JG, Henderson DA. Diagnosis and management of smallpox. N Engl J Med.2002;346:1300-1308 2. Centers of Disease Control and Prevention. Emergency preparedness and response: smallpox. Available at: http://www.bt.cdc.gov/agent/smallpox/index.asp Accessed October 20, 2003

Page 211: 2004 Anesth Practice Ques eBook

Anesthesia Practice Questions: Book 1 – 2004 A=1,2,3 B=1,3 C=2,4 D=4 E=All

This material may not be duplicated without written permission from the publisher. Copyright 2004, All Rights Reserved Medtext Medical World, Inc. .

211

202. B (1,3) May cause prolongation of QT interval in the ECG Has a major metabolite with potent serotonin (5HT3) receptor agonist activity

Both Dolasetron Mesylate and its major metabolite, hydrodolasetron, are potent and highly

selective serotonin (5HT3) receptor antagonists. Dolasetron Mesylate is rapidly eliminated

(t1/2 < 10 min) and completely metabolized to hydrodolasetron by a ubiquitous enzyme,

carbonyl reductase.

Dolasetron Mesylate has multiple routes of elimination- both renal and hepatic. Therefore,

there is no need for dosage adjustment in patients with impaired renal and/or hepatic

function. The majority of hydrodolasetron is excreted in the urine unchanged.

Dolasetron Mesylate can cause ECG interval changes (PR, QT, JT prolongation and QRS

widening). These changes are self-limiting with declining blood levels.

1. Hahne W, Pharmacology and metabolism of Dolasetron Mesylate. Eur Hosp. Pharmacy. 1996;2(Suppl 1):S12-S14. 2. Kuryshev YA, Brown AM, Wang L, Benedict CR, Rampe D. Interactions of the 5-hydroxytryptamine 3 antagonist class of antiemetic drugs with human cardiac ion channels. J Pharmacol Exp Ther. 2000 Nov;295(2):614-20. 203. A (1,2,3) Is an indirectly acting sympathomimetic amine Has four stereoisomers Is superior to other vasopressors in preserving uteroplacental blood flow Ephedrine is an indirectly acting sympathomimetic amine with some degree of direct action

on adrenoceptors. The major mechanism of its indirect action is considered to be release of

norepinephrine from peripheral sympathetic neurons and, possibly, inhibition of neuronal

norepinephrine reuptake, rather than a centrally mediated action.

Because ephedrine contains two asymmetrical carbon atoms, four stereoisomers of

ephedrine are present l-ephedrine, d-ephedrine, l-pseudoephedrine, and d-

pseudoephedrine. Among these only d-ephedrine is used as a vasopressor and d-

pseudoephedrine is used as a nasal decongestant.

Page 212: 2004 Anesth Practice Ques eBook

Anesthesia Practice Questions: Book 1 – 2004 A=1,2,3 B=1,3 C=2,4 D=4 E=All

This material may not be duplicated without written permission from the publisher. Copyright 2004, All Rights Reserved Medtext Medical World, Inc. .

212

Ephedrine is used as the first-line vasopressor to treat hypotension associated with regional

anesthesia in obstetrics. This is because of its ability to preserve utero-placental blood flow

compared with other vasopressors. Uteroplacental circulation lacks (or has reduced)

sympathetic innervation. Since ephedrine exerts its pressor response via norephinephrine

release from sympathetic nerve terminals, it does not produce any contraction in human

umbilical artery or vein.

1. Kobayashi S, Endou M, Sakuraya F et al. The sympathomimetic action of l-ephedrine and d-pseudoephedrine: Direct receptor activation or norepinephrine release? Anesthe Analg 2003;97:1239-45 204. A (1,2,3) Provides extended period of slow drug administration In febrile patients may increase fentanyl absorption Achieves therapeutic levels within 6-8 hours Transdermal fentanyl patch is especially useful in chronic pain patients since it can provide

extended slow drug administration similar to that provided with a continuous intravenous

infusion.

In adults, uptake of fentanyl begins within 1 hour, achieving therapeutic level within 6-8

hours and peak levels at 18-24 hours. The skin acts as a reservoir, and, even after removal,

uptake continues for some hours. The fentanyl uptake is markedly affected by skin blood

flow, and alteration in skin blood flow, such as those caused by fever, may increase

absorption.

1. Cote CJ, Lugo RA, Ward RM.Pharmacokinetics and pharmacology of drugs in children. In Corte CJ, Tordes ID, Ryan JF, Goudsouzian NG (Eds)A Practice of Anesthesia for Infants and Children. 3rd ED. W.B Saunders, Philadelphia, PA. 2001:149

Page 213: 2004 Anesth Practice Ques eBook

Anesthesia Practice Questions: Book 1 – 2004 A=1,2,3 B=1,3 C=2,4 D=4 E=All

This material may not be duplicated without written permission from the publisher. Copyright 2004, All Rights Reserved Medtext Medical World, Inc. .

213

205. A (1,2,3) Supplemental oxygen Positive end-expiratory pressure (PEEP) Inhalation of beta-2-agonists Hypoxemia is the most common clinical manifestation of aspiration pneumonitis.

The most effective treatment is supplemental oxygen and institution of PEEP. Beta-2

agonists effectively relieve bronchospasm. Gastric fluid is rapidly distributed to the

periphery of the lung and lavage with large volumes of fluid could exaggerate this process.

1. Restrictive Lung Disease. Stoelting RK, Dierdorf SF.(eds) Anesthesia and Co-existing Disease. 4th edition. Churchill Livingstone, Philadelphia, PA. 2002:208. 206. D (4) Can be measured using body plethysmography The FRC is defined as the volume of gas in the lung at the end of a normal expiration when

there is no airflow and alveolar pressure equals ambient pressure.

FRC equals the residual volume plus the expiratory reserve volume (not inspiratory reserve

volume). Residual volume cannot be measured by simple spirometry. Therefore the three

methods used for measurement of FRC are Nitrogen washout, Helium washing and body

plethysmography.

1. Benumof JL. Respiratory physiology and respiratory function during anesthesia. In Miller RD (ed.): Anesthesia. 5th edition. Churchill Livingstone, Philadelphia, PA. 2000:590

Page 214: 2004 Anesth Practice Ques eBook

Anesthesia Practice Questions: Book 1 – 2004 A=1,2,3 B=1,3 C=2,4 D=4 E=All

This material may not be duplicated without written permission from the publisher. Copyright 2004, All Rights Reserved Medtext Medical World, Inc. .

214

207. B (1,3) Is a sensitive test of early small airway disease Is measured by the use of a tracer gas such as Xenon-133 (Xe133) Closing volume (CV) is the volume at which smaller airways begin to close during expiration

and CV plus the residual volume is known as the closing capacity (CC). Smoking, obesity,

aging, and the supine position increase the CC.

Closing capacity is a sensitive test of early small airway disease. Measurement of CC is

done by the use of a tracer gas such as xenon-133.

The relationship between FRC and CC is far more important than consideration of the FRC

or CC alone because it is this relationship that determines whether a given respiratory unit

is normal or atelectatic or has a low V/Q ratio. When the volume of lung at which some

airways close is greater than the whole of the tidal volume, lung volume never increases

enough during tidal inspiration to open any of these airways. Thus, these airways stay

closed during the entire tidal breathing. Airways that are closed all the time are equivalent

to atelectasis.

1. Benumof JL. Respiratory physiology and respiratory function during anesthesia. In Miller RD (ed.): Anesthesia. 5th edition. Churchill Livingstone, Philadelphia, PA. 2000:592-3 208. A (1,2,3) Decreases significantly with the induction of general anesthesia. Reduction correlates with an increase in alveolar-arterial PO2 gradient during anesthesia (A-a gradient). Remains decreased into the postoperative period. Induction of general anesthesia is consistently accompanied by a significant (15-20%)

decrease in FRC. The maximum decrease in FRC appears to occur within the first few

minutes of anesthesia and in the absence of any other complicating factors does not seem

to decrease progressively during anesthesia. The reduction of FRC continues into the

postoperative period. For individual patients, this reduction correlates well with an increase

in A-a gradient during anesthesia.

1. Benumof JL. Respiratory physiology and respiratory function during anesthesia. In Miller RD (ed.): Anesthesia. 5th edition. Churchill Livingstone, Philadelphia, PA. 2000:604

Page 215: 2004 Anesth Practice Ques eBook

Anesthesia Practice Questions: Book 1 – 2004 A=1,2,3 B=1,3 C=2,4 D=4 E=All

This material may not be duplicated without written permission from the publisher. Copyright 2004, All Rights Reserved Medtext Medical World, Inc. .

215

209. E All Deliberate hypotension Positive end expiratory pressure (PEEP) Pulmonary embolism Increase in airway pressure Little or no zone 1 exists in the lung under normal conditions because the normal

pulmonary artery pressure (Ppa = 30 cm H2O) is greater than the height of a water column

between the heart and top the lung (about 15 cm); but the amount of zone 1 lung may be

greatly increased if Ppa (pulmonary artery pressure) is reduced or alveolar pressure is

increased.

A decrease in pulmonary artery pressure, as during deliberate hypotension during

anesthesia, increase in alveolar pressure as with PEEP, or pulmonary embolus may increase

the amount of lung that is ventilated but under perfused (increase in Zone 1).

1. Benumof JL. Respiratory physiology and respiratory function during anesthesia. In Miller RD (ed.): Anesthesia. 5th edition. Churchill Livingstone, Philadelphia, PA. 2000:610 2. Powell FL. Structure and Function of the Respiratory System. In Johnson LR (Ed) Essential Medical Physiology. Lippincott Williams & Wilkins, Philadelphia, PA.1998:250 210. A (1,2,3) Increased QT interval Rise in plasma potassium Increased myocardial oxygen demand The effects of CO2 on the cardiovascular system are as complex as those of hypoxia. Like

hypoxemia, hypercapnia appears to cause direct depression of both the cardiac muscle and

vascular smooth muscle, but at the same time it causes reflex stimulation of the

sympathoadrenal system. With moderate to severe hypercapnia, a hyperkinetic circulation

results with increased QT and systemic blood pressure. Thus hypercapnia like hypoxia may

cause increased myocardial O2 demand (tachycardia, early hypertension) and decreased

myocardial O2 supply (tachycardia, late hypotension). Hypercapnia is a potent pulmonary

vasoconstrictor.

Page 216: 2004 Anesth Practice Ques eBook

Anesthesia Practice Questions: Book 1 – 2004 A=1,2,3 B=1,3 C=2,4 D=4 E=All

This material may not be duplicated without written permission from the publisher. Copyright 2004, All Rights Reserved Medtext Medical World, Inc. .

216

Hypercapnia is accompanied by a leakage of potassium from the cells into the plasma.

Much of the potassium comes from the liver, probably from glucose release and

mobilization, which occur in response to the rise in plasma catecholamine levels. Because

the plasma potassium levels take an appreciable time to return to normal, repeated bouts

of hypercapnia at short intervals result in a stepwise rise in plasma potassium.

1. Benumof JL. Respiratory physiology and respiratory function during anesthesia. In Miller RD (ed.): Anesthesia. 5th edition. Churchill Livingstone, Philadelphia, PA. 2000:613-4. 211. E All Malignancy is the underlying cause in 90% cases Dyspnea and headache are the most common symptoms Patient may present with changes in mental status

Changes in voice or stridor may occur Malignancy is the underlying cause in 90% of cases of superior vena cava syndrome (SVC

syndrome). In most, causative extrinsic disease begins in the right paratracheal space or

right pulmonary hilum. Upper body venous hypertension impedes lymphatic drainage, often

leading to lymphedema or chylothorax. Dyspnea and headaches are the most common

symptoms. Changes in mental status herald the onset of cerebral edema; changes in voice

or stridor suggest laryngeal edema or recurrent laryngeal nerve injury.

1. Narang S, Harte BH, Body SC. Anesthesia for patients with a mediastinal mass. Anesthesiol Clin North America. 2001 Sep;19(3):559-79 212. E All Describes the position of the Oxy-hemoglobin curve. When lower, may require higher than normal tissue perfusion to produce the normal amount of oxygen unloading. Is lower in carbon monoxide poisoning. Is lower in methemoglobinemia. The position of the oxy-hemoglobin curve is best described by the PO2 level at which

hemoglobin is 50% saturated (P50). The normal adult P50 is 26.7 mmHg. A P50 lower than

27 mmHg describes a left-shifted oxy-Hb curve and Hb has a higher affinity for O2. This

lower P50 may require higher than normal tissue perfusion to produce the normal amount of

oxygen unloading. The causes of a left-shifted oxy-hemoglobin curve are alkalosis,

Page 217: 2004 Anesth Practice Ques eBook

Anesthesia Practice Questions: Book 1 – 2004 A=1,2,3 B=1,3 C=2,4 D=4 E=All

This material may not be duplicated without written permission from the publisher. Copyright 2004, All Rights Reserved Medtext Medical World, Inc. .

217

hypothermia, abnormal and fetal hemoglobin, carboxyhemoglobin, methemoglobin, and

decreased RBC 2,3-diphosphoglycerate (2,3-DPG) content.

1. Benumof JL. Respiratory physiology and respiratory function during anesthesia. In Miller RD (ed.): Anesthesia. 5th edition. Churchill Livingstone, Philadelphia, PA. 2000:596 213. E All Is stimulated by increases in intravascular volume Causes increased heart rate Sends afferent impulses through vagus nerve Has its receptors within the right atrial wall Bainbridge reflex responds to changes in right atrial or central venous pressure via stretch

receptors present within the right atrial wall and cavoatrial junction. Increases in

intravascular volume or right-sided filling pressures stimulate these receptors and send

their impulses through vagal afferent to inhibit parasympathetic activity and to increase

heart rate. In addition there is also a direct stretching effect on the SA node, which also

leads to enhanced automaticity and increase heart rate.

1. Blanck TJ, Lee DL. Cardiac physiology. In Miller RD (ed.): Anesthesia. 5th edition. Churchill Livingstone, Philadelphia, PA. 2000:641 214. A (1,2,3) Responds to changes in blood pressure via stretch receptors present in the carotid sinus and aortic arch. Sends impulses along the afferent limbs of glossopharyngeal and vagus Typically begins to respond at pressures in excess of 170 mmHg. The baroreceptor reflex (carotid sinus reflex) responds to changes in blood pressure

(typically >170 mmHg increase) via circumferential and longitudinal stretch receptors

present in the carotid sinus and aortic arch. The impulses are sent along the afferent limbs

of glossopharyngeal and vagus nerve. The response is decreased sympathetic activity-

decreased contractility, heart rate, and vascular tone, and increased parasympathetic

activity. Decreases in blood pressure have the reverse effect but at pressures lower than

50-60 mmHg baroreceptors lose much of their functional capacity.

1. Blanck TJ, Lee DL. Cardiac physiology. In Miller RD (ed.): Anesthesia. 5th edition. Churchill Livingstone, Philadelphia, PA. 2000:641

Page 218: 2004 Anesth Practice Ques eBook

Anesthesia Practice Questions: Book 1 – 2004 A=1,2,3 B=1,3 C=2,4 D=4 E=All

This material may not be duplicated without written permission from the publisher. Copyright 2004, All Rights Reserved Medtext Medical World, Inc. .

218

215. B (1,3) Is principally driven by mean arterial pressure Exhibits some degree of autoregulation Coronary blood flow (CBF), like blood flow in other vessels, is principally driven by mean

arterial pressure. CBF through the left side is maximal during early diastole, corresponding

to the period of isovolumic relaxation and minimum extravascular compression. CBF

through the right side is maximal during peak systole, because developed pressure and

consequently extravascular compression within the RV are considerably less than in the LV.

Coronary arteries exhibit some degree of autoregulation like the renal and carotid arteries

and maintain CBF within tightly controlled limits over a perfusion pressure range of 0 to

140 mmHg.

1. Blanck TJ, Lee DL. Cardiac physiology. In Miller RD (ed.): Anesthesia. 5th edition. Churchill Livingstone, Philadelphia, PA. 2000:642-3 216. B (1,3) A VVIR pacemaker senses electrical activity only in the ventricle An AOO pacemaker is a fixed rate pace maker The pacemaker code consists of 5 letters. The first three letters describes the anti

bradycardic function, the fourth addresses the rate responsive feature, and the fifth

describes the antitachycardia function.

First letter: Identifies which chamber(s) of the heart are paced by the pace maker-A

(atrium), V (ventricle), and D (both)

Second letter: Refers to which chamber is sensed for electrical activity. (A/V/D or O for no

sensing)

Third letter: Describes the pacemaker’s response to sensed electrical activity. O for no

response; I refers to inhibited; T to triggered; D for dual triggered and inhibited.

Page 219: 2004 Anesth Practice Ques eBook

Anesthesia Practice Questions: Book 1 – 2004 A=1,2,3 B=1,3 C=2,4 D=4 E=All

This material may not be duplicated without written permission from the publisher. Copyright 2004, All Rights Reserved Medtext Medical World, Inc. .

219

Fourth letter: R is used to indicate the presence of a rate responsive feature or ability to

increase heart rate based on the body’s metabolic demand.

Fifth letter: Is rarely used and indicates the response of the pacemaker to detected

tachycardia.

VVIR pacemaker senses electrical activity only in the ventricle. R indicates that the

pacemaker has a rate responsive capability.

VVD pacemaker can only pace in the ventricle; it can sense electrical activity in both the

atrium and the ventricle.

AOO is a fixed rate pacemaker and cannot serve any intrinsic electrical activity.

When AAI pacemaker senses an intrinsic electrical beat in the atrium, it inhibits the

generator from firing.

1. Moses H, Miller B, Moulton K, Schneider J. A Practical Guide to Cardiac Pacing. 5th edition Philadelphia: Lippincott Williams & Wilkins. 2000. 217. E All Battery failure Hyperkalemia Myocardial ischemia Severe hyperglycemia Failure to capture may be caused by

a) An increase in fibrotic tissue at the site of lead implantation

b) Fracture of the pacemaker leads

c) Disconnection between the pacemaker lead and the generator

d) Battery failure

e) Metabolic changes that change the effective threshold for capture-

hyperkalemia, severe hyperglycemia, hypercarbia or hypoxia

f) Medications- beta blockers, amiodarone, procainamide

g) Myocardial ischemia and myocardial infarction 1. Sarko J, Tiffany B. Cardiac pacemakers: Evaluation and management of malfunctions. Am J Emerg Med 2000;18(4):435-440.

Page 220: 2004 Anesth Practice Ques eBook

Anesthesia Practice Questions: Book 1 – 2004 A=1,2,3 B=1,3 C=2,4 D=4 E=All

This material may not be duplicated without written permission from the publisher. Copyright 2004, All Rights Reserved Medtext Medical World, Inc. .

220

218. E All Causes platelet dysfunction Directly impairs enzymes of the coagulation cascade Impairs immune function Decreases wound oxygen delivery Coagulation is impaired by mild hypothermia. The most important factor appears to be a

cold-induced defect in platelet function. Hypothermia also directly impairs enzymes of the

coagulation cascade. This may not be apparent during routine coagulation screening

because the tests are performed at 37oC.

Hypothermia can contribute to wound infection both by directly impairing immune function

and by triggering thermoregulatory vasoconstriction that, in turn, decreases wound oxygen

delivery.

1. Sessler DI. Temperature Monitoring. In Miller RD (ed.): Anesthesia. 5th edition. Churchill Livingstone, Philadelphia, PA. 2000:1376-77. 219. E All Increases oxygen consumption about 100% Increases intraocular pressure Can be treated by skin surface warming Increases intracranial pressure Postanesthetic shivering is a potentially serious complication, increasing oxygen

consumption roughly 100%, in proportion to intraoperative heat loss. In addition to

increasing intraocular and intracranial pressures, postoperative shivering also likely

aggravates wound pain by stretching incisions.

Postanesthetic shivering can be treated by skin surface warming because the regulatory

system tolerates more core hypothermia when cutaneous warm input is augmented. This

technique is effective in most patients with core temperatures exceeding 35oC.

1. Sessler DI. Temperature Monitoring. In Miller RD (ed.): Anesthesia. 5th edition. Churchill Livingstone, Philadelphia, PA. 2000:1378.

Page 221: 2004 Anesth Practice Ques eBook

Anesthesia Practice Questions: Book 1 – 2004 A=1,2,3 B=1,3 C=2,4 D=4 E=All

This material may not be duplicated without written permission from the publisher. Copyright 2004, All Rights Reserved Medtext Medical World, Inc. .

221

220. A (1,2,3) Operating room temperature is the most crucial factor influencing heat loss One cotton blanket placed on the patient reduces heat loss by about 30% The most effective perianesthetic warming system is forced air warming blanket Operating room temperature is the most crucial factor influencing heat loss because it

determines the rate at which metabolic heat is lost by radiation and convection from the

skin and by evaporation from within surgical incisions.. However room temperature > 23oC

in adults and > 26oC in infants generally are required to maintain normothermia. The

easiest method of decreasing cutaneous heat loss is to apply passive insulation to the skin.

A single layer of cotton blankets, surgical drapes and plastic sheeting reduces heat loss

approximately 30%.

Studies consistently report that circulating-water mattresses are nearly ineffective.

Furthermore, the combination of heat and decreased local perfusion (due to patient’s

weight reducing capillary blood flow) increases the propensity for burns even when water

temperature does not exceed 40o C.

The most effective perianesthetic warming system is forced air and has been shown to

maintain normothermia even during the largest operations.

1. Sessler DI. Temperature Monitoring. In Miller RD (ed.): Anesthesia. 5th edition. Churchill Livingstone, Philadelphia, PA. 2000:1380-81 221. B (1,3) Is employed to reduce the need for allogenic red blood cells May improve tissue perfusion The term acute normovolemic hemodilution (ANH) refers to the removal of blood from the

surgical patient immediately before or just after induction of anesthesia, replacement with

asanguinous fluid, and later reinfusion of the withdrawn blood. ANH is employed to reduce

the need for allogenic red blood cells and to avoid potential transfusion associated

complications. An additional potential advantage of ANH is improvement in tissue perfusion

as a result of decreased viscosity.

Page 222: 2004 Anesth Practice Ques eBook

Anesthesia Practice Questions: Book 1 – 2004 A=1,2,3 B=1,3 C=2,4 D=4 E=All

This material may not be duplicated without written permission from the publisher. Copyright 2004, All Rights Reserved Medtext Medical World, Inc. .

222

The presence of malignancy or wound infection may contraindicate blood recovery during

surgery, but not ANH.

1. Stehling L. Autologous Transfusion. In Miller RD (ed.): Anesthesia. 5th edition. Churchill Livingstone, Philadelphia, PA. 2000:1649-50 222. E All Air and fat embolism Rh isoimmunization during cesarean deliveries Renal dysfunction secondary to lysis of red blood cells Disseminated intravascular coagulation Potential complications associated with use of the cell processing devices include air and fat

embolism, pulmonary dysfunction secondary to infusion of debris in recovered blood,

coagulopathy, renal dysfunction, sepsis and dissemination of malignant cells. Lysis of red

blood cells can occur as a result of high vacuum suction levels or aspiration techniques that

cause turbulence during blood collection. Two cases of renal dysfunction requiring dialysis

have been reported.

Processed blood is depleted of coagulation proteins and functional platelets. There is a

report of two cases of disseminated intravascular coagulation in patients undergoing spine

surgery during which recovered blood was reinfused.

Transfusion of red blood cells salvaged during cesarean section can result in the

administration of a substantial additional load of fetal erythrocytes. Antigens present on

fetal erythrocytes but absent on maternal erythrocytes can result in alloimmunization

(especially Rh isoimmunization). Therefore, immunoglobulin should be administered as

necessary.

1. Stehling L. Autologous Transfusion. In Miller RD (ed.): Anesthesia. 5th edition. Churchill Livingstone, Philadelphia, PA. 2000:1654-5 2. Weiskopf RB. Erythrocyte salvage during cesarean section. Anesthesiology. 2000;92:1519-22

Page 223: 2004 Anesth Practice Ques eBook

Anesthesia Practice Questions: Book 1 – 2004 A=1,2,3 B=1,3 C=2,4 D=4 E=All

This material may not be duplicated without written permission from the publisher. Copyright 2004, All Rights Reserved Medtext Medical World, Inc. .

223

223. A (1,2,3) Hemorrhage accounts for more than 25% of maternal deaths. In the United States, maternal deaths due to anesthesia related Complications have decreased within the last twenty years. Maternal case-fatality risk ratio for general anesthesia has shown to be 16 Times that of regional anesthesia after 1985. Maternal hemorrhage, the leading cause of maternal mortality, accounts for 25% of

maternal mortality in the developing world, just as it does in the developed world.

In the United States, the anesthesia related maternal mortality rate (deaths per million live

births) has decreased in each subsequent 3 year period from 4.3 per million in 1979-1981

to 1.7 per million in 1988 - 1990. Since 1984, the number of deaths involving general

anesthesia have remained stable, but the number of regional anesthesia –related deaths

have decreased. The case-fatality risk ratio for general anesthesia was 2.3 times that for

regional anesthesia before 1985, but increased to 16.7 times that after 1985.

1. Thomas T. Maternal Mortality. In Birnbach DJ, Gatt SP, Datta s, editors. Textbook of Obstetric anesthesia 2000. Churchill Livingstone Philadelphia, PA. 2000:35 2. Hawkins JL, Koonin LM, Palmer SK, Gibbs CP. Anesthesia-related deaths during obstetrics delivery in the United States, 1979-1990. Anesthesiology 1997;86:277-84 224. A (1,2,3) Developmentally, is a paired structure May have midline gaps due to lack of fusion in the mid-line Is thinnest at the cervical and high thoracic levels The ligamentum flavum is a structure composed of elastic fibers. In contrast to the

ligamentum flavum, supraspinous and interspinous ligaments are composed of collagenous

fibers.

Developmentally, the ligamentum flavum is a paired structure and gaps due to lack of

fusion in the midline has been demonstrated. Absence of continuous ligamentum flavum

may imply that actual loss of resistance is brought about by supraspinous and interspinous

ligaments. Therefore, distinct elastic resistant offered by the ligamentum flavum may be

blunted or even absent, in these situations. These gaps are shown to be frequent at the

cervical and high thoracic levels but become rare at the T3/T4 level and below.

1. Lirk P, Kolbitsch C, Putz G, et al. Cervical and high thoracic ligamentum flavum frequently fails to fuse in the midline. Anesthesiology 2003;99:1387-90.

Page 224: 2004 Anesth Practice Ques eBook

Anesthesia Practice Questions: Book 1 – 2004 A=1,2,3 B=1,3 C=2,4 D=4 E=All

This material may not be duplicated without written permission from the publisher. Copyright 2004, All Rights Reserved Medtext Medical World, Inc. .

224

225. D (4) Total uterine blood flow approaches 800 ml/min The total uterine blood flow increases during pregnancy and approaches 800 ml/min; 80%

of which reaches the intervillous space for interfacing with the fetal circulation, while the

remaining 20% supplies the uterine myometrium. The uterine vascular bed is almost

maximally dilated under normal conditions. The uterine blood flow is not autoregulated and

is proportional to the mean perfusion pressure.

Uterine blood flow = uterine arterial pressure - uterine venous pressure / uterine vascular

resistance

The sensitivity to alpha-adrenergic agonists in the uterine circulation is maintained and in

fact, uterine vessels become more sensitive to alpha agonists than are the systemic

vessels. Accordingly, excessive administration of vasopressors with predominantly alpha-

adrenergic action may impair uterine blood flow.

1. Alahuhta S. Uteroplacental blood flow. In Birnbach DJ, Gatt SP, Datta s, editors. Textbook of Obstetric anesthesia 2000. Churchill Livingstone Philadelphia, PA. 2000:62 226. B (1,3) Acts by increasing cyclic guanosine monophosphate May be potentiated by Sildenafil, phosphodiesterase-5 inhibitor Nitric oxide used as an inhaled gas is a selective pulmonary vasodilator and there is no

systemic vasodilatation and hypotension. It acts by stimulating soluble guanylate cyclase

and increasing cyclic guanosine monophosphate. The effect of inhaled nitric oxide may be

potentiated by phophodiesterase-5 inhibitors (e.g. dipyridamole and sildenafil) which

specifically suppress cyclic guanosine monophosphate catabolism and prolong its action.

Sildenafil has been described in case reports as a therapy for severe pulmonary

hypertension.

1. Blaise G, Langleben D, Hubert B. Pulmonary arterial hypertension, pathophysiology and anesthetic approach. Review article. Anesthesiology 2003;99:1415-32 2. Prasad S, Wilkinson J, Gatzoulis MA. Sildenafil in primary pulmonary hypertension (letter). N Eng J Med 2000;343:1342

Page 225: 2004 Anesth Practice Ques eBook

Anesthesia Practice Questions: Book 1 – 2004 A=1,2,3 B=1,3 C=2,4 D=4 E=All

This material may not be duplicated without written permission from the publisher. Copyright 2004, All Rights Reserved Medtext Medical World, Inc. .

225

227. A (1,2,3) Metabolic acidosis Respiratory acidosis Decreased extracellular concentration of potassium Both metabolic and respiratory acidosis may augment a nondepolarizing neuromuscular

blockade. Low extracellular concentration of potassium (especially acute decrease)

increases the transmembrane potential, causing hyperpolarization of cell membranes. This

manifests as resistance to the effects of depolarizing drugs and increased sensitivity to

nondepolarizing neuromuscular blocking drugs. Hyperkalemia has the opposite effects,

increasing the effects of depolarizing drugs and opposing the action of nondepolarizers.

Administration of antibiotics particularly aminoglycosides and polypeptide classes may

enhance the blockade. Antibiotics devoid of neuromuscular blocking effects are the

penicillins and cephalosporins.

1. Savarese JJ, Caldwell JE, Lien CA, Miller RD. Pharmacology of muscle relaxants and their antagonists. In Miller RD (ed.): Anesthesia. 5th edition. Churchill Livingstone, Philadelphia, PA. 2000:468-9. 2. Neuromuscular Blocking drugs In Stoelting RK (editor) Physiology & Pharmacology in Anesthetic Practice. Third edition. Lippincott Williams & Wilkins, Philadelphia, PA. 1999:197. 228. E All Respiratory acidosis (PaCO2 greater than 50 mmHg) Hypothermia Low extracellular potassium concentration Magnesium sulfate Antagonism of neuromuscular blockade by anticholinesterase drugs may be inhibited or

even prevented by (a) certain antibiotics, (b) hypothermia, (c) respiratory acidosis

associated with a PaCO2 of > 50 mmHg, (d) magnesium sulfate, or (e) hypokalemia and

metabolic acidosis.

Magnesium sulfate, given for the treatment of preeclampsia, enhances the neuromuscular

blocking properties of both non depolarizers and depolarizers. Magnesium decreases the

amount of Ach released from the motor nerve terminal, the depolarizing action of Ach on

the postjunctional membrane, the excitability of the muscle fiber itself, and the amplitude

Page 226: 2004 Anesth Practice Ques eBook

Anesthesia Practice Questions: Book 1 – 2004 A=1,2,3 B=1,3 C=2,4 D=4 E=All

This material may not be duplicated without written permission from the publisher. Copyright 2004, All Rights Reserved Medtext Medical World, Inc. .

226

of the action potential. Therefore, there is reduced Ach output and reduced sensitivity of

the postjunctional membrane.

1. Neuromuscular blocking drugs. In Stoelting RK (editor) Physiology & Pharmacology in Anesthetic Practice. Third edition. Lippincott Williams & Wilkins, Philadelphia, PA. 1999:197. 2. Savarese JJ, Caldwell JE, Lien CA, Miller RD. Pharmacology of muscle relaxants and their antagonists. In Miller RD (ed.): Anesthesia. 5th edition. Churchill Livingstone, Philadelphia, PA. 2000:463.. 229. E All Has less muscarinic effects Has a quicker onset of action Has fewer side effects Is not broken down by acetylcholinesterase In patients with normal renal and hepatic function, there are no significant pharmacokinetic

differences among the anticholinesterase drugs. Controlled studies in anesthetized patients

have documented that the duration of action of edrophonium does not differ from that of

neostigmine. Elimination half-life of edrophonium (110 min) is similar to that of

pyridostigmine (113 min) and neostigmine (77 min). Edrophonium has a quicker onset of

action and fewer side effects. The muscarinic effects of edrophonium are mild compared

with neostigmine.

Neostigmine inhibits the breakdown of acetylcholine by virtue of it being hydrolyzed by

acetylcholinesterase. In this process, acetylcholinesterase is carbamylated, and its ability to

hydrolyze acetylcholine is decreased. In contrast to neostigmine, acetylcholinesterase does

not break down edrophonium but rather edrophonium forms a reversible electrostatic

attachment to the enzyme.

1. Savarese JJ, Caldwell JE, Lien CA, Miller RD. Pharmacology of muscle relaxants and their antagonists. In Miller RD (ed.): Anesthesia. 5th edition. Churchill Livingstone, Philadelphia, PA. 2000:470. 2. Neuromuscular Blocking drugs In Stoelting RK (editor) Physiology & Pharmacology in Anesthetic Practice. Third edition. Lippincott Williams & Wilkins, Philadelphia, PA. 1999:226-8..

Page 227: 2004 Anesth Practice Ques eBook

Anesthesia Practice Questions: Book 1 – 2004 A=1,2,3 B=1,3 C=2,4 D=4 E=All

This material may not be duplicated without written permission from the publisher. Copyright 2004, All Rights Reserved Medtext Medical World, Inc. .

227

230. E All The sensitivity to non depolarizers may be diminished The response to succinylcholine may be lengthened The reversal of residual block at the end of the case may be ineffective Often surgical relaxation can be provided using a potent inhaled anesthetic only Pyridostigmine will modify the response to relaxants and the sensitivity to non-depolarizers

will be diminished. The response to succinylcholine may be lengthened because therapeutic

levels of pyridostigmine may partially inhibit pseudocholinesterase. The reversal of non-

depolarizers may be ineffective because much acetylcholinesterase inhibition already exist.

Therefore, it is probably safer to allow spontaneous recovery postoperatively while

continuing mechanical ventilation.

1. Savarese JJ, Caldwell JE, Lien CA, Miller RD. Pharmacology of muscle relaxants and their antagonists. In Miller RD (ed.): Anesthesia. 5th edition. Churchill Livingstone, Philadelphia, PA. 2000:471 231. B (1,3) The duration of mivacurium may be lengthened due to decrease in the plasma cholinesterase activity The duration of action of atracurium is not affected by renal failure Renal failure influence the pharmacology of nondepolarizing muscle relaxants by producing

either decreased elimination of the drug or its metabolites via the kidney, or decreased

activity of enzymes that metabolize the drug (e.g., mivacurium). Renal failure can decrease

plasma cholinesterase activity, and mivacurium is metabolized by plasma cholinesterase at

approximately 70% of the rate of succinylcholine.

The pharmacokinetics and the duration of action of atracurium are unaffected by renal

failure, because atracurium undergoes spontaneous chemical degradation and ester

hydrolysis. The elimination half-life of laudanosine, the principal metabolite of atracurium,

increases significantly in renal failure.

Although the plasma clearance is unchanged in renal failure, the volume of distribution of

rocuronium is increased by 28% and elimination half life is prolonged by 37%.

1. Savarese JJ, Caldwell JE, Lien CA, Miller RD. Pharmacology of muscle relaxants and their antagonists. In Miller RD (ed.): Anesthesia. 5th edition. Churchill Livingstone, Philadelphia, PA. 2000:458

Page 228: 2004 Anesth Practice Ques eBook

Anesthesia Practice Questions: Book 1 – 2004 A=1,2,3 B=1,3 C=2,4 D=4 E=All

This material may not be duplicated without written permission from the publisher. Copyright 2004, All Rights Reserved Medtext Medical World, Inc. .

228

232. B (1,3) Decreasing the amount of acetylcholine (Ach) released from the motor nerve terminal Decreasing the excitability of the muscle fiber itself Magnesium decreases the amount of acetylcholine (Ach) released from the nerve terminal,

the depolarizing action of the Ach on the postjunctional membrane, the excitability of the

muscle fiber itself and the amplitude of the end-plate potential.

A prolonged block should be anticipated when a muscle relaxant is administered to a

patient receiving magnesium.

1. Savarese JJ, Caldwell JE, Lien CA, Miller RD. Pharmacology of muscle relaxants and their antagonists. In Miller RD (ed.): Anesthesia. 5th edition. Churchill Livingstone, Philadelphia, PA. 2000:463 233. B (1,3) Local anesthetics Dantrolene In large doses and given intravenously, most local anesthetics block neuromuscular

transmission; in smaller doses, they enhance the neuromuscular block from both

nondepolarizing and depolarizing muscle relaxants.

Dantrolene depresses skeletal muscle directly and also blocks excitation-contraction

coupling; the effects of nondepolarizing muscle relaxants are enhanced.

Chronic phenytoin or carbamazepine therapy accelerates recovery from nondepolarizing

muscle relaxants; the mechanism of this relative resistance is not clear.

1. Savarese JJ, Caldwell JE, Lien CA, Miller RD. Pharmacology of muscle relaxants and their antagonists. In Miller RD (ed.): Anesthesia. 5th edition. Churchill Livingstone, Philadelphia, PA. 2000:464

Page 229: 2004 Anesth Practice Ques eBook

Anesthesia Practice Questions: Book 1 – 2004 A=1,2,3 B=1,3 C=2,4 D=4 E=All

This material may not be duplicated without written permission from the publisher. Copyright 2004, All Rights Reserved Medtext Medical World, Inc. .

229

234. B (1,3) Disruption of sodium channel activation process Blockade of the ion-conducting pores Local anesthetics do not alter the resting membrane potential or the threshold level, but it

slows the rate of depolarization by inhibition of the sodium channels

There are 2 fundamental mechanisms of inhibition:

(1) Na+ currents are reduced by the local anesthetics primarily because the drug-

bound channels fail to open; channel activation process is disrupted by local

anesthetics. A sodium channel inhibited by local anesthetic is functionally similar

to an inactivated channel.

Local anesthetics bind more rapidly and with higher affinity to activated

channels (open or preceding the open state) than to resting channels.

Regardless of the channel state, by its very binding the local anesthetic

stabilizes that state.

(2) Blockade of ion-conducting pore, contribution from this action seems minor.

1. Berde CB, Strichartz GR. Local Anesthetics In Miller RD (ed.): Anesthesia. 5th edition. Churchill Livingstone, Philadelphia, PA. 2000:498-50 235. E All Ester local anesthetics are relatively unstable in solution Amide compounds undergo enzymatic degradation in the liver Reports of allergic reactions to amide compounds are extremely rare Amide local anesthetics are extremely stable in solution The ester and amide local anesthetics differ in their chemical stability, locus of

biotransformation, and allergic potentials.

Amino esters are hydrolyzed in plasma by the cholinesterase enzymes, except cocaine. P-

aminobenzoic acid is one of the metabolites of ester type compounds that can induce

allergic-type reactions in a small percentage of patients.

1. Berde CB, Strichartz GR. Local Anesthetics In Miller RD (ed.): Anesthesia. 5th edition. Churchill Livingstone, Philadelphia, PA. 2000:501-2.

Page 230: 2004 Anesth Practice Ques eBook

Anesthesia Practice Questions: Book 1 – 2004 A=1,2,3 B=1,3 C=2,4 D=4 E=All

This material may not be duplicated without written permission from the publisher. Copyright 2004, All Rights Reserved Medtext Medical World, Inc. .

230

236. E All Rate of injection Hypercarbia Metabolic acidosis Site of injection The rate of injection and the rapidity with which a particular blood level is achieved

(vascularity of the site of injection) alter the toxicity of local anesthetic drugs. Respiratory

and metabolic acidosis increases the risks of local anesthetic toxicity. An elevation of PaCO2

enhances cerebral blood flow and anesthetic is delivered more rapidly to the brain. In

addition, diffusion of carbon dioxide into the neuron cells decreases the intracellular pH, so

that ion trapping occurs. Hypercarbia and/or acidosis also decrease the plasma protein

binding of local anesthetic drugs. This increases the free drug available for diffusion into

the brain.

1. Berde CB, Strichartz GR. Local Anesthetics In Miller RD (ed.): Anesthesia. 5th edition. Churchill Livingstone, Philadelphia, PA. 2000:511. 237. B (1,3) Venodilatation Hypotension which is greater in hypertensive patient than in normotensive patients The predominant cardiovascular effect of barbiturate induction is venodilatation, followed

by pooling of blood in the periphery. Myocardial contractility is depressed, but not to the

extent seen after volatile anesthetics.

Both thiopental and methohexital cause an increased heart rate. Therefore, must be used

cautiously if at all in conditions in which an increased heart rate or a decrease in preload

could be detrimental (pericardial tamponade, ischemic heart disease, hypovolemia,

congestive heart failure). Heart rate increases more after methohexital than after

equivalent dose of thiopental.

Hypotension from a given dose is greater in both treated and untreated hypertensive

patients than in normotensive patients.

1. Fragen RJ, Avram MJ, Barbiturates. In Miller RD (ed.): Anesthesia. 5th edition. Churchill Livingstone, Philadelphia, PA. 2000:509.

Page 231: 2004 Anesth Practice Ques eBook

Anesthesia Practice Questions: Book 1 – 2004 A=1,2,3 B=1,3 C=2,4 D=4 E=All

This material may not be duplicated without written permission from the publisher. Copyright 2004, All Rights Reserved Medtext Medical World, Inc. .

231

238. E All Vecuronium Atracurium Sufentanil Midazolam The pH of thiopental solution is 10-11. A decrease in the alkalinity of the barbiturate

solution can result in their precipitation as free acids. The drugs that are in more acidic

solutions when co-administered with thiopental have the potential to precipitate and

occlude the intravenous line. These include pancuronium, vecuronium, atracurium,

sufentanil and midazolam. Such precipitation can be prevented by allowing a 30-second

delay between the injection of thiopental and the injection of other drugs.

1. Fragen RJ, Avram MJ, Barbiturates. In Miller RD (ed.): Anesthesia. 5th edition. Churchill Livingstone, Philadelphia, PA. 2000:509. 239. C (2,4) May cause hypotension After discontinuation most drug effects are eliminated within 5 minutes Esmolol is a potent selective beta-1-receptor antagonist. It may cause hypotension through

both vasodilatation and negative inotropic effect. Esmolol is rapidly hydrolyzed by esterases

in the cytosol of red blood cells (not plasma esterases) resulting in an elimination half life of

9 minutes.

1. Balser JR, Butterworth J. Cardiovascular drugs. In Hensley FA,jr, Martin DE, Gravlee GP (eds) A Practical Approach to Cardiac Anesthesia. 3rd edition. Lippincott Williams & Wilkins, Philadelphia, PA. 2003:93. 240. B (1,3) Tachyphylaxis Metabolic acidosis Nitroprusside has a rapid onset of action and a short duration of effect because of the rapid

breakdown break down of the unstable nitroprusside radical to produce cyanide. Free

cyanide ions are usually converted to thiocyanate in the liver and kidney. Free cyanide not

converted to thiocyanate can bind with high affinity to cytochrome oxidase, interfere with

electron transport, prevent cellular aerobic respiration, and produces tissue hypoxia.

Prolonged administration of nitroprusside at moderate doses may result in cyanide and

Page 232: 2004 Anesth Practice Ques eBook

Anesthesia Practice Questions: Book 1 – 2004 A=1,2,3 B=1,3 C=2,4 D=4 E=All

This material may not be duplicated without written permission from the publisher. Copyright 2004, All Rights Reserved Medtext Medical World, Inc. .

232

thiocyanate toxicity. During nitroprusside toxicity, there is metabolic acidosis and elevated

mixed venous PO2 (due to decreased cellular O2 utilization). No cyanosis is seen because

cells cannot utilize O2, therefore O2 saturation remains high. The recommended maximum

dose of 1.5 mg/kg for acute administration or 0.5 mg/kg/h (8 µg/kg/min) for chronic

administration appear to be safe.

1. Balser JR, Butterworth J. Cardiovascular drugs. In Hensley FA,jr, Martin DE, Gravlee GP (eds) A Practical Approach to Cardiac Anesthesia. 3rd edition. Lippincott Williams & Wilkins, Philadelphia, PA. 2003:93. 2. Aken HV, Miller ED. Deliberate Hypotension. In Miller RD (ed.): Anesthesia. 5th edition. Churchill Livingstone, Philadelphia, PA. 2000:1476. 241. C (2,4) Reflex bradycardic response is absent after carotid sinus massage Response to laryngoscopy may be delayed or blunted The cardiac autonomic plexus is transected during orthotopic cardiac transplantation,

leaving the transplanted heart completely denervated. Partial efferent sympathetic neuronal

reinnervation commences within 12 months of cardiac transplantation. Parasympathetic

reinnervation is much less extensive, resulting in a characteristic high resting heart rate

(HR 90-110 bpm) and absent reflex bradycardic response after carotid massage. The

transplanted heart increases cardiac output (CO) primarily by increasing stroke volume

(SV), in contrast to the normal heart. As acute changes in CO, HR, and SV in the

transplanted heart are mediated mainly by adrenal secretion and circulation of direct acting

catecholamines, the response to stressful stimuli may be delayed or blunted.

1. Rother AL, Collard CD. Anesthetic management for cardiac transplantation. In Hensley FA,jr, Martin DE, Gravlee GP (eds) A Practical Approach to Cardiac Anesthesia. 3rd edition. Lippincott Williams & Wilkins, Philadelphia, PA. 2003:442-3.

Page 233: 2004 Anesth Practice Ques eBook

Anesthesia Practice Questions: Book 1 – 2004 A=1,2,3 B=1,3 C=2,4 D=4 E=All

This material may not be duplicated without written permission from the publisher. Copyright 2004, All Rights Reserved Medtext Medical World, Inc. .

233

242. E All No change in heart rate with administration of atropine and glycopyrrolate Increased response to epinephrine and norepinephrine Absent reflex bradycardia or tachycardia in response to changes in systemic arterial blood pressure Reduced response to alpha-adrenergic agents Autonomic denervation of the transplanted heart significantly alters the pharmacodynamic

activity of many drugs. Drugs that mediate their actions through the autonomic nervous

system are generally ineffective in altering heart rate and contractility, although partial

reinnervation may explain individual variation. Directly acting catecholamines, such as

norepinephrine and epinephrine on the other hand, show an increased response. Response

to alpha-adrenergic agents may be reduced. Reflex bradycardia or tachycardia in response

to changes in systemic blood pressure is absent. Atropine and glycopyrrolate will not alter

heart rate, although their peripheral ani cholinergic activity remains unaffected.

1. Rother AL, Collard CD. Anesthetic management for cardiac transplantation. In Hensley FA,jr, Martin DE, Gravlee GP (eds) A Practical Approach to Cardiac Anesthesia. 3rd edition. Lippincott Williams & Wilkins, Philadelphia, PA. 2003:442-3. 243. E All Is a direct vasoconstrictor Enhances the pre junctional release of norepinephrine from the adrenergic nerve ending Increases efferent sympathetic nerve activity Stimulates the secretion of aldosterone The juxtaglomerular cells of the renal cortex secrete rennin, which cleaves angiotensinogen

(produced in the liver) to form angiotensin I. Angiotensin I is converted almost immediately

to angiotensin II by ACE (angiotensin converting enzyme). ACE is found predominantly in

the endothelial cells of the lung. In addition to its direct vasoconstrictive activity,

angiotensin II enhances the pre junctional release of norepinephrine from the adrenergic

nerve ending and increases efferent sympathetic nerve activity. Angiotensin II also affects

sodium water homeostasis by increasing ADH and stimulating the secretion of aldosterone.

1. Moss J, Renz CL. The Autonomic Nervous System. In Miller RD (ed.): Anesthesia. 5th edition. Churchill Livingstone, Philadelphia, PA. 2000:562.

Page 234: 2004 Anesth Practice Ques eBook

Anesthesia Practice Questions: Book 1 – 2004 A=1,2,3 B=1,3 C=2,4 D=4 E=All

This material may not be duplicated without written permission from the publisher. Copyright 2004, All Rights Reserved Medtext Medical World, Inc. .

234

244. A (1,2,3) Bradycardia with changes in position Hyperthermia easily Hypothermia easily In patients with high spinal cord transection, the only intact efferent component of

baroreflex pathway is the vagus. Therefore, bradycardia occurs with changes in position,

Valsalva maneuvers or increased thoracic pressure.

There is an increase in sensitivity to exogenously administered angiotensin as well as

catecholamines, possibly due to the impairment of descending inhibitory pathways.

Hypothermia may occur readily from cutaneous vasodilatation and the inability to shiver.

Similarly, hyperthermia can occur because the normal sweating mechanisms can be

impaired.

1. Moss J, Renz CL. The Autonomic Nervous System. In Miller RD (ed.): Anesthesia. 5th edition. Churchill Livingstone, Philadelphia, PA. 2000:568-9. 245. E All Inadequate corticosteroid coverage can cause death Administration of supraphysiologic doses of steroid for a short time perioperatively causes no discernible complications. Even topical application of steroid can suppress normal adrenal response for as long as 9-12 months. Plasma cortisol concentration of more than 25 µg/ml, measured during acute stress, indicates normal pituitary–adrenal responsiveness Inadequate corticosteroid coverage can lead to addisonian crisis and death. Administration

of supraphysiologic doses of steroid for a short time perioperatively causes no discernible

complications. Therefore, if in doubt supplementation should be provided to any patient

who has received steroids within a year. Even topical application of steroid can suppress

normal adrenal response for as long as 9-12 months.

The morning plasma cortisol level does not reveal whether the adrenal cortex has

recovered sufficiently. If the plasma cortisol concentration is measured during acute stress,

a value of more than 25 µg/ml indicates normal pituitary –adrenal responsiveness 1. Roizen MF .Anesthetic implications of concurrent diseases. In Miller RD (ed.): Anesthesia. 5th edition. Churchill Livingstone, Philadelphia, PA. 2000:921-2.

Page 235: 2004 Anesth Practice Ques eBook

Anesthesia Practice Questions: Book 1 – 2004 A=1,2,3 B=1,3 C=2,4 D=4 E=All

This material may not be duplicated without written permission from the publisher. Copyright 2004, All Rights Reserved Medtext Medical World, Inc. .

235

246. E All Is an imidazole sedative-hypnotic Causes compromised adrenal reserves for at least 24 hours after a single induction dose Causes minimal cardiovascular depression Inhibits two essential adrenocortical

Etomidate is an imidazole sedative-hypnotic. Adrenal reserve is compromised for at least 24

hours after a single induction dose. It inhibits two essential adrenocortical hormones. There

is a dose dependent reversible inhibition of the enzyme 11-beta-hydroxylase, and a

relatively minor effect on 17-alpha-hydroxylase. It has been shown that the issue of

temporary adrenocortical suppression following induction dose of etomidate is not clinically

significant.

Etomidate causes minimal cardiovascular depression even in compromised patients. The

hemodynamic stability seen with etomidate may be due in part to its unique lack of effect

both on the sympathetic nervous system and on baroreceptor function.

1. Roizen MF .Anesthetic implications of concurrent diseases. In Miller RD (ed.): Anesthesia. 5th edition. Churchill Livingstone, Philadelphia, PA. 2000:922.

247. E All Serotonin Cholinergic Dopamine Histamine

The vomiting center is located in the lateral reticular formation of the medulla. This center

receives afferent signals from (a) the chemoreceptor trigger zone, (b) the vestibular center,

(c) higher brain stem and cortical structures, and (d) from visceral afferents such as the GI

tract. Various receptors are implicated in the stimulation of the chemoreceptor trigger zone.

These different receptors form the basis by which most current antiemetic pharmacology is

base on. These receptors are serotonin, dopamine, cholinergic (specifically muscarinic) and

histamine receptors. Antiemetic drugs such as ondansetron and dolasetron antagonize

serotonin receptors. Antidopaminergic drugs include metoclopramide and droperidol.

Antihistamine medications with antiemetic properties include promethazine and

dimenhydrinate. Finally, scopolamine represents the current anticholinergic drug in use as a

prophylactic antiemetic.

1. Evers AS, Maze M. Anesthetic Pharmacology: Physiologic Principles and Clinical Practice, Churchill Livingstone. New York NY. 2004:778-787.

Page 236: 2004 Anesth Practice Ques eBook

Anesthesia Practice Questions: Book 1 – 2004 A=1,2,3 B=1,3 C=2,4 D=4 E=All

This material may not be duplicated without written permission from the publisher. Copyright 2004, All Rights Reserved Medtext Medical World, Inc. .

236

248. C 2,4 A - delta fiber Unmyelinated C fiber

Normal thermoregulation of body temperature is dependent on three components:

1. Afferent input from temperature sensing receptors throughout the body

2. Central regulation or control by the hypothalamus

3. Efferent responses in the form of (a) autonomic defenses such as sweating and

vasodilation for temperature increases and vasoconstriction and shivering for

decreases in temperature, and (b) behavior responses such as adding more

clothing when cold, or modifying the temperature of the environment. Of the two

responses, behavior responses are the most effective and important response.

Afferent temperature nerve fiber are primarily of two type, A - delta fibers transmit cold

sensation while unmyelinated C fibers transmit warm signals.

Below is summation of nerve fiber classification and their physiologic properties:

Type Fiber Diameter (µ) Myelination Function

A - alpha 12 - 20 Yes Proprioception, Motor

A - beta 5 - 12 Yes Proprioception, Motor

A - gamma 3-6 Yes Muscle Tone

A - delta 2 – 5 Yes Pain, Temperature, Touch

B < 3 Yes Preganglionic Autonomic

C 0.3 – 1.2 NO Autonomic, Pain, Temperature, Touch

1. Miller R. Anesthesia. 5th edition. Churchill Livingstone. New York NY. 2000:1367,494-495.

Page 237: 2004 Anesth Practice Ques eBook

Anesthesia Practice Questions: Book 1 – 2004 A=1,2,3 B=1,3 C=2,4 D=4 E=All

This material may not be duplicated without written permission from the publisher. Copyright 2004, All Rights Reserved Medtext Medical World, Inc. .

237

249. B 1,3 Low molecular weight Low protein binding

Passive diffusion is the primary means by which medications administered to a mother

enter the fetal circulation. Factors that help promote this passive diffusion include:

1. Low molecular weight (less than 500 daltons)

2. Low protein binding

3. High lipid solubility

4. Low degree of ionization

Almost all medications used to produce anesthesia, analgesia or sedation share many of

these characteristics and readily cross the placenta. Neuromuscular relaxants however,

have low lipid solubility and high degree of ionization and therefore do not readily cross the

placenta.

1. Miller R. Anesthesia. 5th edition. Churchill Livingstone. New York NY. 2000:2031-2032.

Page 238: 2004 Anesth Practice Ques eBook

Anesthesia Practice Questions: Book 1 – 2004 A=1,2,3 B=1,3 C=2,4 D=4 E=All

This material may not be duplicated without written permission from the publisher. Copyright 2004, All Rights Reserved Medtext Medical World, Inc. .

238

250. D 4 Usually affects children ages 2 to 6 years Epiglottitis is a life-threatening inflammation of the epiglottis secondary Haemophilus

influenzae Type b. However, since the use of a vaccine for H. influenzae type B, the

incidence of epiglottitis has decreased dramatically. Epiglottitis requires differentiation from

laryngotracheobronchitis (viral croup). At no time (in the emergency room or x-ray suite)

should direct visualization of the epiglottis be attempted in the unanesthetized patient

when epiglottitis is suspected since this may lead to complete obstruction of the airway.

Characteristics of Laryngotracheobronchitis vs. Epiglottitis

Laryngotracheobronchitis Epiglottitis Etiology Parainfluenza virus, type 1

(viral) Haemophilus influenzae, type b (bacterial)

Age Infancy 2 months-2years 2-6 years

Onset Subacute (insidious) onset Acute onset

Temperature Low grade fever High fever

Course Usually mild, stridor May worsen at night

Rapid progress of symptoms

Symptomatology Barking (croupy) cough stridor

Dysphagia, sore throat, Respiratory distress, Dysphonia (The 4 D’s – Dysphonia, Dysphagia, Drooling, Respiratory Distress)

1. Cote JC, et. al., A Practice of Anesthesia for Infants and Children. 3rd edition. Sauders. Philadelphia PA. 2001:319-321. 2. Barash P, Cullen B, Stoelting R. Clinical Anesthesia, 4th edition, J.B. Lippincott. Philadelphia PA. 2001:996-997.

Page 239: 2004 Anesth Practice Ques eBook

Let us know your test results! We love hearing from our customers. Send a message to [email protected]

Page 240: 2004 Anesth Practice Ques eBook

ISBN 1-889344-65-6 www.medtext.net